You are on page 1of 312

SAT

The SAT

1 1
SAT

Revolution Prep
SAT Workbook

This workbook is for the exclusive use of Revolution Prep, LLC and its students and is not legal for resale.
© 2015 Revolution Prep, LLC. All rights reserved. Version 12-15.

While every effort has been made to ensure the accuracy and correctness of the content in this manual,
Revolution Prep assumes no responsibility for errors or omissions, or for damages resulting from the use of
the information contained herein.

This product is not endorsed by the College Board., nor any other agency or school. Any similarity to other
printed material is purely coincidental.

2 2
SAT Table of Contents

Table of Contents

3 3
SAT Table of Contents

Introduction to the SAT: Test Overview (Page 7)


Introduction to the SAT: Content Overview (Page 13)
The Test Zone (Page 23)

II

SAT Reading: Determining Central Ideas, Themes, and Meaning (Page 27)
SAT Writing: Punctuation (Page 39)
SAT Math: Foundations of Arithmetic and Algebra (Page 47)

III

SAT Writing: Passage Analysis (Page 53)


SAT Math: Plug and Chug & Backsolving (Page 61)
SAT Math: Translating Word Problems (Page 65)

IV

SAT Reading: Command of Evidence (Page 73)


SAT Math: Solving Non-Linear Equations (Page 79)
SAT Writing: Wordiness and Redundancy (Page 85)

The SAT Essay (Page 89)


SAT Writing: Transitions (Page 113)
SAT Math: Solving Linear Equations (Page 117)
SAT Math: Ratio, Proportion, and Unit Conversion (Page 123)

VI

SAT Reading and Writing: Words in Context (Page 129)


SAT Writing: Parallelism (Page 135)
SAT Math: Average, Median, and Mode (Page 139)
SAT Math: Functions (Page 145)

4 4
SAT Table of Contents

VII

SAT Reading and Writing: Evaluating Quantitative Information (Page 151)


SAT Writing: Pronouns (Page 157)
SAT Writing: Verbs (Page 165)
SAT Math: Quadratics and Polynomials (Page 169)

VIII

SAT Reading: Analyzing Multiple Texts (Page 177)


SAT Writing: Prepositions (Page 185)
SAT Writing: Adjectives and Adverbs (Page 189)
SAT Math: Circles (Page 193)
SAT Math: Triangles (Page 199)
SAT Math: Quadrilaterals and Polygons (Page 207)
SAT Math: Lines and Angles (Page 213)

IX

SAT Writing: Word Choice (Page 217)


SAT Math: Statistics (Page 221)
SAT Math: Graphing (Page 229)
SAT Math: Interpreting Equations (Page 233)

SAT Writing: Advanced Grammar (Page 237)


SAT Math: Geometry in Three Dimensions (Page 245)
SAT Math: Advanced Concepts (Page 253)

Extra Content

Practice Reading Passages (Page 261)


Practice Writing Passages (Page 277)

Appendix

Appendix: Answers (Page 293)


Appendix: Common Math Formulas (Page 305)
Appendix: Table of Contents (Page 307)

5 5
6 6
SAT Introduction to the SAT: Test Overview

Introduction to the SAT:


Test Overview

7 7
SAT College Admissions

Why Does the SAT Matter?

What are the main factors in college admission?

Big impact on admissions


relative to the time investment!
Application:
• Essays 25% 25%
• Recommendations Application SAT
• Extra-curricular activities
• Sports/Clubs
• Volunteer work
• Jobs
50% Academic Transcript:
Academic Transcript • Grade Point Average (GPA)
• Class rigor

What is a good SAT score?

An AVERAGE SCORE is roughly 500 points per section.


A GOOD SCORE depends on what colleges you want to apply to.

TARGET UNIVERSITIES AVERAGE SAT SCORE

Harvard, Yale, Stanford, Princeton, Univ. of Chicago, Columbia, Northwestern, Elite


Washington Univ. in St. Louis 1500-1600

Georgetown, Cornell, Emory, UCLA, USC, UC Berkeley, Rice, NYU, Notre Dame, Most Competitive
Grinnell, Univ. of Michigan 1400-1500

Univ. of Virginia, Syracuse, Univ. of Wisconsin, Highly Competitive


Univ. of Illinois, Ohio State Univ., Case Western, UC Irvine 1300-1400

Univ. of Colorado, SUNY Schools, Florida State, Loyola, Very Competitive


Univ. of Washington, UC Davis, Rutgers 1200-1300

CUNY Schools, Univ. of Oregon, UC Riverside, Competitive


Univ. of Arizona, Arizona State, Ball State, Temple 1100-1200

Indiana State, Cleveland State, Cal State Long Beach, Less Competitive
San Diego State, Coppin State, Indiana Univ. of Penn. 1000-1100

8 8
SAT Structure and Scoring

SAT LENGTH

Test 1: Reading = 65 Minutes


Test 2: Writing and Language = 35 Minutes

Test 3 and Test 4: Math = 80 Minutes


o Test 3: No calculator math section = 25 Minutes
o Test 4: Calculator permitted math section = 55 Minutes

Test 5: Essay (Optional) = 50 Minutes

Total with Essay = 230 Minutes (3 hours and 50 minutes)

SAT ORDER OF DIFFICULTY

Math questions generally increase in difficulty as you move through a section.


o “Obvious” answers are usually correct earlier in a section and incorrect later
in a section.

Reading and Writing questions are of randomly varying difficulty throughout each
section.

SAT SCORING

The Math and the combined Reading and Writing tests are each scored from 200 to
800.

Your COMPOSITE SCORE is the total of these two scores (400 to 1600).

The Essay is scored from 6 to 24. The Essay DOES NOT factor into your composite
score.

9 9
SAT Content

READING

If you are reading this, then it’s already clear you can read boring
material. Now, you just need to learn how to answer the questions
correctly.

This has a lot less to do with knowledge than with your ability to survive
mind-numbing reading passages.

WRITING AND LANGUAGE

What’s scary: it measures your understanding of standard written


English.

What’s not-so-scary: there’s no spelling, no vocabulary, and no complicated


terminology.

MATH

All the math you were trying to forget: Pre-Algebra, Algebra, Coordinate &
Plane Geometry, and Trigonometry.

**THE BEST NEWS**


You will learn what the test will be like EVERY TIME.
There will be no surprises on test day.

10 10
SAT Guessing and Pacing

GUESSING ON THE SAT

There is NO PENALTY for wrong answers on the SAT and every question is
worth the SAME POINT VALUE.

• FIRST: Answer all the questions you KNOW HOW TO DO.


• NEXT: Make educated guesses on questions for which you can ELIMINATE ANSWERS.
• LAST: Guess randomly on questions about which you HAVE NO CLUE.

PACING ON THE SAT

DON’T RUSH to answer every question.

• If you’re regularly making careless mistakes, you must SLOW DOWN.


• Spend test time working on the problems you KNOW HOW TO DO.
• Spend homework time working on the problems you DON’T KNOW HOW TO DO.

DON’T HESITATE to cut loose.

• If you’re unsure whether you’ve wasted “a lot” of time on a question, you already have!
o If you stare at a question and CAN’T figure out anything, MOVE ON!
• The LONGER a question takes, the LESS it can help your score.

DON’T WORRY about timing for every single question.

• GOOD MOVE: Staying aware of how much time is left in a test.


• BAD MOVE: Calculating how much time is left per question in a test.
• Develop a sense of test pacing from your practice tests and homework.
• Do the math on the test, not on the clock.

11 11
12 12
SAT Introduction to the SAT: Content Overview

Introduction to the SAT:


Content Overview

13 13
SAT Reading Introduction

Test 1: Reading

Section Structure

65 Minutes for 52 Questions – Five passages


• Four single passages, one paired passage.
• Passages from fiction, current events, and the “Great Global
Conversation.”

SAT Reading Concepts

• Understanding main ideas and purpose


• Applying quantitative information
• Defining words in context
• Utilizing textual evidence

Quick Reading Tips


• Read for main ideas.
• Save time by skimming details.
• Always anticipate your answer.
• Always eliminate choices to find the correct answer.
• Don’t spend too much time on any one question.
• Don’t make stupid mistakes – stay focused.

14 14
SAT Reading Introduction

Test 1: Reading

Reading Strategy
The Passage

1. Read – Read the introductory blurb and the passage using your Two-Track
Mind.
2. Ask – Ask yourself questions that help you focus on the author’s argument. Do
this for each sentence and for each paragraph.
3. Reflect – Pause before answering the questions and try to BRIEFLY describe
the author, the subject of the passage, and the main points.

The Questions – Take CARE

1. Cover – Cover the answer choices with your hand, then read the question.
2. Anticipate – Figure out how YOU would answer the question. Use any line
references that may be provided to help.
3. Reveal – Once you have your answer in mind, uncover the answer choices.
4. Eliminate – Compare their answer choices with your answer, and
ELIMINATE WRONG ANSWERS.

Key Points for Reading


• Read for “What’s it about?” NOT “What happens?”

• ANTICIPATE your own answers before looking at the answer choices.

• ELIMINATE wrong answers, especially “Out of Scope” ones.

15 15
SAT Writing and Language Introduction

Test 2: Writing and Language

Section Structure

35 Minutes for 44 Questions – Four passages, 11 questions each.

SAT Writing and Language Concepts

• Basic Grammar and Syntax


o Prepositions, punctuation, verbs, etc.
• Main Idea Analysis
• Quantitative Analysis
• Words in Context

Quick Writing Tips


• Trust your grammar ear!
• Stay confident – you know all the grammar you need to know.
• Identify the grammatical issue that is being tested.
• Don’t make a problem harder than it is.
• Don’t make stupid mistakes – stay focused.

16 16
SAT Writing and Language Introduction

Test 2: Writing and Language

Writing and Language Strategy

1. READ period to period.


Even if there isn’t an underlined portion in a sentence, READ THAT SENTENCE
ANYWAY.

2. USE your grammar ear.


 If the underlined part is CLEARLY RIGHT, mark answer choice “A.”
 If the underlined part is CLEARLY WRONG, anticipate a correction, then find the
answer choice that most closely matches your anticipation.
 If you CAN’T TELL whether the underlined part is right or wrong,
IMMEDIATELY move on to the next question.

3. GO BACK to the questions you skipped.


After you’ve done ALL the questions in the section that you KNOW HOW TO DO, go
back to the questions you skipped and plug in answer choices to find the right one.

Key Points for Writing

• Don’t stop reading when you see an underlined word or phrase – READ TO THE END OF
THE SENTENCE.

• Use your GRAMMAR EAR to identify errors.

• The SIMPLEST AND MOST DIRECT phrasing of a sentence is usually the best answer
choice.

17 17
SAT Math Introduction

Tests 3 and 4: Math

Section Structure

80 Minutes for 58 Questions

Test 3: Math (No Calculator)


• 25 minutes for 20 questions
• Multiple choice questions and student-produced responses

Test 4: Math (Calculator)


• 55 minutes for 38 questions
• Multiple choice questions and student-produced responses

SAT Math Concepts

• Algebra
o Formulas, expressions, and equations
• Data analysis
o Ratios, proportions, and percentages
• Advanced math
o Quadratics and polynomials
• Additional topics
o Geometry and trigonometry

Quick Math Tips


• Stay confident – you know all the math you need to know!
• Identify the concept that is being tested.
• Don’t make a problem harder than it is.
• Don’t make stupid mistakes – stay focused.

18 18
SAT Math Introduction

Tests 3 and 4: Math

Math Strategy – ISME

1. Identify – ALWAYS begin by determining the problem type.


The most important step in solving a math problem on the SAT does NOT involve
your calculator.

2. Set Up - WRITE DOWN WHAT YOU KNOW IS TRUE


If there’s a diagram, label anything you can. Determine which equation(s) you will
need to do the math.

3. Make Sure - What is the SPECIFIC answer they are asking for? Make sure you’re
not falling for one of the SAT’s tricks.

4. Execute - Plug in the information from the problem, carefully write out ALL
STEPS, and solve the problem. After you’ve got an answer, check your work, and
make sure your answer makes sense.

ISME Application

The average (arithmetic mean) score of six teams is 80 points. If the average score of four
of the teams is 75 points, then what is the average number of points scored by the other
two teams?

A) 82.5
B) 85
C) 88.3
D) 90

19 19
SAT Essay Introduction

Test 5: The Essay

Section Structure

50 Minutes for 1 Prompt

SAT Essay Concepts

• Reading Comprehension
• Analyzing Arguments
• Effective Writing Communication

Quick Essay Tips


• Make sure your essay structure includes introduction, body,
and conclusion paragraphs.
• Make sure your essay content includes a thesis and
supporting evidence.
• Be familiar with common argumentative techniques.
• Take time to brainstorm your ideas before writing your essay.

20 20
SAT Essay Introduction

Test 5: The Essay

Essay Strategy
1. Read the essay prompt – The prompt is the same for each essay. The provided text
will change.

2. Read and notate the provided text – Analyze the text for the author’s thesis and
how the author supports that thesis through the use of argumentative techniques.

3. Write an I-B-C - This is your brief outline. It only needs to make sense to you.

I: Introduction – What is the author’s thesis? What are the major argumentative
techniques used to support the thesis?

B: Body – This is the meat of your essay. Jot down two or three argumentative
techniques and how the author used those techniques to support the thesis.

C: Conclusion – A BRIEF wrap-up. Jot down a phrase that ties everything together

4. Write Your Essay – Let your I-B-C notes be your guide. Follow traditional essay
structure.

Key Points

• Before you start writing your essay, take three minutes to BRAINSTORM & ORGANIZE.

• Make sure your ESSAY STRUCTURE includes Intro, Body, and Conclusion paragraphs.

• Make sure your ESSAY CONTENT includes a Thesis and Supporting Evidence.

21 21
22 22
SAT The Test Zone

The Test Zone

23 23
SAT The Test Zone

THE DIFFERENCE
BETWEEN A
“GOOD DAY”
AND A
“BAD DAY”
CAN BE 200 PTS
THE TEST ZONE IS ABOUT MAKING SURE
YOU HAVE A GOOD DAY


24 24
SAT The Test Zone

What is the TEST ZONE?

The TEST ZONE is a state in which you are in complete control of yourself and the exam, so you
can MAKE THE TEST WORK FOR YOU, not the test makers.

It’s easy to stop caring about the


test (it’s BORING!). You need to
remind yourself why you have to
care about every question, every
Losing focus on one
time (COLLEGE!).
question can throw you off FOCUS
for the next few questions. MOTIVATION
You need to work through
the test one question at
time and forget about
everything else!

TEST
ZONE

The SAT is a long test! Four hours of


sitting and pretending to care about
pronouns, probability, and passages.
You need to get your brain used to
THINKING for the entire time, so
ENDURANCE that you have the necessary energy
until the last question.

Success on the SAT is as much about HEART and ATTITUDE as


intelligence
It doesn’t matter how much you know if you are not mentally and physically prepared to take
the test. Being prepared is about being in the TEST ZONE.

**TEST ZONE…YOUR NEW BEST FRIEND**


Apply it on the SAT and every other test you take!

25 25
SAT The Test Zone

ENDURANCE
Warning signs:

• Your mind starts wandering.


• You start thinking about what you’re going to do after the exam.

Helpful remedies:

• Make the effort to stay focused for the entirety of every class.
• Take your practice tests seriously. Bring the intensity that you would bring to the official test.
• Do your homework assignments in one sitting.

MOTIVATION
Warning signs:

• You miss a word or two in a question and miss the question.


• You fall into careless and easy traps.
• You don’t try questions that you don’t immediately know how to solve.

Helpful remedies:

• Attack the exam aggressively, as though you were battling a mortal enemy.
• Use the strategies and do not take shortcuts.
• Always give it a shot, write things down, get your pencil moving.

FOCUS
Warning signs:

• Your mind starts wandering.


• You start thinking about what you’re going to do after the exam.
• You have a song lyric that you can’t get out of your head.

Helpful remedies:

• Practice outside of class in a public place to get used to dealing with distractions.
• Develop a routine and follow it for the practice exams.

26 26
SAT Determining Central Ideas, Themes, and Meaning

SAT Reading:
Determining Central Ideas,
Themes, and Meaning

Answers on Page 294

27 27
SAT Determining Central Ideas, Themes, and Meaning: Reading for Main Ideas

Reading for Main Ideas

When reading on the SAT, it is important to read for an understanding of what the text is
about (as opposed to what happens).

We understand what a text is about through its main ideas:

• Why did the author write this sentence, paragraph, or passage?


• If I had to quickly describe this sentence, paragraph, or passage to someone
else, what would I say?

Facts, lists, and details are unimportant to your initial understanding of a text.
Think big picture!

Paragraph Structure: Where Do Main Ideas Live?

Main ideas are usually located at the beginning and the end of a paragraph. Details
tend to be found in the middle of a paragraph.

Thesis/Topic Sentence: Main ideas are most likely to live here.


Students must stop after the opening sentence of a
paragraph and actively note the idea/purpose.

Middle of Paragraph: This is where facts, details, and


reiterations tend to be found. Readers oftentimes get bogged down
in this part of the paragraph.

Closing Sentence(s): Main ideas/paragraph summaries are


frequently found here. Students must not neglect the ends of
paragraphs when they read.

28 28
SAT Determining Central Ideas, Themes, and Meaning: Effective Reading

Effective Reading: The Two-Track Mind

Whenever you read, there are two parts of your brain at work:

1) The part of your brain that reads the words on the page.
2) The part of your brain that thinks about other stuff.

These two parts of your brain do not necessarily work together!

To ensure focus, periodically stop and ask yourself questions while you read!

• At the end of each paragraph, STOP! What is the main idea?


• At the end of the passage, STOP!
o Who was the author? Was the author part of the story or just telling the
story?
o What was the author’s overall purpose in writing the passage?
o How did the author feel about this topic? What is the tone of the
passage?

Effective Reading: The Two-Speed Transmission

Once we know how to identify main ideas, we can focus our time and attention on the most
important parts of a text.

When to accelerate when reading a text:

• When the author lists facts to support a main idea.


• When the author repeats a point that has already been made.
• When the author uses unfamiliar terms/phrases that are unrelated to a
main idea.

When to slow down when reading a text:

• At the beginning and the end of a paragraph.


• Whenever the author introduces a new idea.
o Watch out for transitional words/phrases that indicate a new or
opposing idea (however, therefore, for example, etc.).
• When the author expresses his or her opinion.

29 29
SAT Determining Central Ideas, Themes, and Meaning: Effective Reading

Effective Reading: Sample Paragraph

Critics argue that since the Apollo


missions, little substantial progress has
been made in space exploration. NASA
Line missions have been criticized as ineffective
5 and costly, and more than one respected
pundit has suggested that the U.S.
government drastically reduce funding for
the International Space Station and other
NASA ventures. Against this backdrop of
10 hostility, policymakers would do well to
remember that trailblazing scientific
exploration has been the subject of
misplaced scrutiny throughout history.
Centuries before the founding of NASA,
15 Christopher Columbus faced similar
skepticism in funding his maiden – and in
retrospect, historic – voyage across the
Atlantic.

30 30
SAT Determining Central Ideas, Themes, and Meaning: Effective Reading

Effective Reading: Sample Paragraph


NEW MAIN IDEA (SLOW
Critics argue that since the Apollo DOWN): NASA programs
missions, little substantial progress has have recently been the
been made in space exploration. NASA subject of ridicule.
Line missions have been criticized as ineffective
5 and costly, and more than one respected
DETAILS (SPEED UP)
pundit has suggested that the U.S.
about the ridicule that
government drastically reduce funding for NASA has received.
the International Space Station and other
NASA ventures. Against this backdrop of
10 hostility, policymakers would do well to
remember that trailblazing scientific NEW MAIN IDEA (SLOW
exploration has been the subject of DOWN): Columbus faced
misplaced scrutiny throughout history. skepticism similar to
what NASA is currently
Centuries before the founding of NASA,
facing.
15 Christopher Columbus faced similar
skepticism in funding his maiden – and in
retrospect, historic – voyage across the
Atlantic.

31 31
SAT Determining Central Ideas, Themes, and Meaning: Passage #1

Questions 1-10 are based on the following But there is also a new flexibility associated with
passage. 45 the electronic media which is at odds with the very
idea of standardization and regulation.
In the following passage, an educator questions the Even where print appears on the screen, it is
importance of standardization in spelling. not fixed; it moves, it can be changed. There isn’t
the same sense of a finished product, or of a fixed
Line Correct spelling has been held up as a measure 50 meaning. Electronic “publications” have an air of
of literacy (or illiteracy). Those who have taken the fluidity, fleetingness, ephemeralness about them.
same license as Shakespeare and spelled their Pluralism and diversity are much more the current
name in a variety of ways have been labeled as values, than standardization.
5 nonstandard, or as character-deficient. So, for example, when I come up with my own
That is not the only side effect that there has 55 creative representations of words on my e-mail,
been: countless members of society have had their those whom I am contacting don’t think that I am
confidence eroded – and their writing and reading ignorant or reprehensible. Neither the medium nor
efforts dampened – by the trap of correct spelling. the form is seen as definitive or authoritative as
10 It isn’t too much to suggest that the obsession with the printed page used to be. Those who are
standardization (and the punishment of 60 connecting to my messages are likely to assume
nonstandard or “incorrect” forms) has contributed that I am not a good typist, or that there is no
to ratings on illiteracy. With so many mistakes to spellchecker on my e-mail – if they notice my
be made, some people are just too frightened to try nonstandard offerings at all.
15 to write or read. Yet these “mistakes” didn’t even I have no doubt that they will not only work out
exist in Shakespeare’s era. 65 what my message means, but that I will provide
Spelling has definitely been used throughout them with some entertainment in the process.
the last century of print to make moral judgments Besides, if there are people who do object, they
about individuals and the state of society. It was have the solution close at hand. Let them use their
20 not uncommon – prior to spellcheckers – to find own spellchecker to standardize the message. But
that job applicants would be eliminated on the 70 it is their problem, not mine, if they cannot cope
grounds of a spelling error, for example. To be with the pluralism.
unable to spell was a measure of ignorance; to People spelled creatively before print, and no
neglect to look up a word in a dictionary and find doubt they will again after the values and mindset
25 the correct form was considered a measure of of standardization have begun to recede. It could
carelessness, slothfulness and irresponsibility – all 75 be that in the not-too-distant future, when the
of which were disqualifications for the job. population once again spells a word the way it
The conventional wisdom now is that young sounds (as they work on their e-mail or electronic
people don’t spell as well as their parents; that bulletin boards), that scholars will look back on the
30 computers and spellcheckers mean that they don’t print period, and standardization, as the
keep the rules in their heads any more, but are 80 aberration in communication. Then they will see
content to access a computer program. This is fine, mindlessness not in those who defied the spelling
as far as most young people are concerned; they conventions, but in those who insisted that such
want to be able to do it, not to know it. But it is arbitrary mechanics should ever have been
35 often taken as a sign of moral decadence by the committed to memory, and then used as a standard
older generation, rather than as evidence that the 85 of worthiness.
information medium is changing.
There are many reasons that standardization in
spelling is not as important today; it’s partly
40 because of spellcheckers that the same energy is
not put into drilling the young to memorize all the
rules. They simply don’t need to learn them off by
heart any more, if they are working on a computer.

32 32
SAT Determining Central Ideas, Themes, and Meaning: Solving Questions

Solving Reading Questions on the SAT: There are four steps to solving reading
questions on the SAT.

1) Cover the answer choices. Do not look at the answer choices!

2) Anticipate an answer to the question. How would YOU answer the question? Use the
notes you took while reading!

3) Reveal the answers and then eliminate answers that are clearly incorrect. Answers
are clearly incorrect because they do not align with your anticipation!

4) Make a decision among whatever choices remain. Avoid answers with absolute
language and/or non-relevant information.

Solving Reading Questions on the SAT: Out of Scope Answers

What makes wrong answers wrong can generally be reduced to a few words – the words
that make a wrong answer “Out of scope.”

How to tell if an answer choice is “Out of scope”:

• It brings in subjects that are not related to the passage.

• It talks about something from a part of the passage outside of the part the question is
referring to.

• It is too extreme (unless the author is very extreme).

• It assumes too much about the author’s viewpoint.

33 33
SAT Determining Central Ideas, Themes, and Meaning: Passage #1 Questions

1 5

The passage is written from the perspective Which choice provides the best evidence for the
of someone who is answer to the previous question?
A) actively involved in linguistics research. A) Lines 17-19 (“Spelling…society”)
B) a participant in a recent debate in the B) Lines 28-32 (“The conventional…program”)
field of linguistics. C) Lines 32-34 (“This is …it”)
C) a historian of linguistics. D) Lines 38-42 (“There are…rules”)
D) an advocate for further linguistics
research.
6

2 The author highlights electronic media primarily


in order to
It can be reasonably inferred from the
passage that the author’s feeling towards A) suggest that the standardization rules for
nonstandard spelling is one of printed media do not apply to electronic
media.
A) qualified skepticism. B) distance himself from the past.
B) sincere approval. C) highlight the possible dangers of a
C) professional detachment. nonstandardized world.
D) open hostility. D) predict the re-emergence of a cyclical trend.

3 7

In the passage, the author references Which choice provides the best evidence for the
Shakespeare in order to answer to the previous question?
A) supply a revisionist theory. A) Lines 38-42 (“There are…rules”)
B) disprove a historical misconception. B) Lines 42-43 (“They simply…computer”)
C) provide a broader context. C) Lines 50-53 (“Electronic…standardization”)
D) invoke an illustrious prose stylist. D) Lines 72-74 (“People…recede”)

4 8
According to the passage, why do young The first sentence of the seventh paragraph
people favor nonstandard spelling? (lines 54-57) most directly responds to which of
the following statements?
A) Young people view nonstandard spelling
as an opportunity to rebel against their A) Lines 17-19 (“Spelling…society”)
parents. B) Lines 28-32 (“The conventional…program”)
B) Young people are following the lead of C) Lines 38-42 (“There are…rules”)
linguistics scholars. D) Lines 47-48 (“Even…changed”)
C) The education of young people does not
stress the rules of standard spelling.
D) Young people view the result of spelling
as more important than the method of
spelling.

34 34
SAT Determining Central Ideas, Themes, and Meaning: Passage #1 Questions

As used in line 48, “fixed” most nearly


means
A) stable.
B) corrected.
C) rigged.
D) complete.

10

The author’s attitude toward those who


“cannot cope with the pluralism” (lines 70-
71) is best described as
A) dismissive.
B) compassionate.
C) spiteful.
D) bemused.

35 35
SAT Determining Central Ideas, Themes, and Meaning: Passage #2
Questions 11-20 are based on the following We're here in Washington also to say that the
passage. 50 problem of this war is not just a question of war
and diplomacy. It's part and parcel of everything
This passage is adapted from the Vietnam that we are trying as human beings to
Veterans Against the War Statement by John communicate to people in this country: the
Kerry to the Senate Committee of Foreign question of racism, which is rampant in the
Relations 55 military; and so many other questions also: the use
of weapons; the hypocrisy in our taking umbrage in
And suddenly we are faced with a very the...Geneva Conventions and using that as
sickening situation in this country because there’s justification for continuation of this war, when we
no moral indignation, and if there is it comes from are more guilty than any other body of violations of
Line people who are almost exhausted by their past 60 those Geneva Conventions -- in the use of free-fire
5 indignincies [sic], and I know that many of them zones, harassment interdiction fire, search and
are sitting in front of me. The country seems to destroy missions, the bombings, the torture of
have lied -- lain down and accepted something as prisoners, the killing of prisoners -- accepted policy
serious as Laos, just as we calmly shrugged off by many units in South Vietnam. That's what
the loss of 700,000 lives in Pakistan, the so-called 65 we're trying to say. It's part and parcel of
10 greatest disaster of all times. everything.
But we are here as veterans to say that we An American Indian friend of mine who lives on
think we are in the midst of the greatest disaster of the Indian nation of Alcatraz put it to me very
all times now because they are still dying over succinctly. He told me how as a boy on an Indian
there, and not just Americans, Vietnamese, and we 70 reservation he had watched television and he used
15 are rationalizing leaving that country so that those to cheer the cowboys when they came in and shot
people can go on killing each other for years to the Indians. And then suddenly one day he stopped
come. in Vietnam and he said, "My God, I'm doing to
Americans seem to have accepted the idea that these people the very same thing that was done to
the war is winding down, at least for Americans, 75 my people," -- and he stopped. And that's what
20 and they have also allowed the bodies, which were we're trying to say, that we think this thing has to
once used by a President for statistics to prove that end.
we were winning this war, to be used as evidence
against a man who followed orders and who
interpreted those orders no differently than 11
25 hundreds of other men in South Vietnam.
We veterans can only look with amazement on the The central problem that Kerry describes in
fact that this country has not been able to see that the passage is that
there's absolutely no difference between a ground
troop and a helicopter crew. And yet, people have A) there is no end in sight to the Vietnam
accepted the differentiation fed them by the War.
30
Administration. No ground troops are in Laos, so B) more American soldiers are dying in
it's alright to kill Laotians by remote control. But Vietnam now than during earlier parts
believe me, the helicopter crews fill the same body of the war.
bags and they wreak the same kind of damage on C) the American public has come to accept
the Vietnamese and Laotian countryside as anyone the current situation in Vietnam.
35
else, and the President is talking about allowing D) the United State is imminently close to
that to go on for many years to come. And one can defeat in Vietnam.
only ask if we will really be satisfied when the
troops march in to Hanoi.
40 We are asking here in Washington for some
action, action from the Congress of the United
States of America which has the power to raise and
maintain armies and which by the Constitution
also has the power to declare war. We've come
45 here, not to the President, because we believe that
this body can be responsive to the will of the
people; and we believe that the will of the people
says that we should be out of Vietnam now.

36 36
SAT Determining Central Ideas, Themes, and Meaning: Passage #2 Questions

12 15
Kerry implies that which of the following is a The author italicizes the word “now” in line
relatively recent development? 13 primarily to
A) The understanding that the Vietnam A) highlight the difference between a past
War will not end. situation and a present situation.
B) The involvement of American Indian B) reflect the severity of growing social
soldiers in fighting the Vietnam War. divisions.
C) The belief that certain methods of killing C) question the feasibility of a situation.
enemies are more acceptable than D) stress the urgency of an issue.
others.
D) The ability of the United States
Congress to limit the United States’
16
involvement in the Vietnam War.
As used in line 30, ”fed” most nearly means

13 A) consumed.
B) eaten.
Which choice provides the best evidence for C) spread.
the answer to the previous question? D) told.
A) Lines 11-14 (“But…Americans”)
B) Lines 29-31 (“And yet…administration”)
C) Lines 40-44 (“We are…war”) 17
D) Lines 67-69 (“An American…succinctly”)
As used in line 42, ”raise” most nearly means

A) boost.
B) enhance.
14 C) form.
D) lift.
The second paragraph (lines 11-17) implies
that
A) the lives of people of different 18
nationalities have similar value.
B) the United States is currently acting in a Kerry contends that the situation he
treacherous manner. describes in the passage has become so dire
C) more American soldiers are dying each that
day than Vietnamese soldiers.
A) it has done irreparable harm to the
D) the United States military leaving
United States.
Vietnam will help expedite Vietnam’s
B) it has eroded the principles underpinning
return to stability.
the United States.
C) it has prevented the United States
military from carrying out its assigned
duties.
D) it has caused social divisions which will
last for generations.

37 37
SAT Determining Central Ideas, Themes, and Meaning: Passage #2 Questions

19

Which choice provides the best evidence for


the answer to the previous question?

A) Lines 1-10 (“And suddenly…times”)


B) Lines 18-25 (“Americans…Vietnam”)
C) Lines 40-44 (“We are…war”)
D) Lines 51-64 (“It’s part…Vietnam”)

20

Kerry references his American Indian friend


in the last paragraph (lines 67-77) in order
to

A) illustrate a philosophical difference.


B) convey a changed mindset.
C) highlight a unique camaraderie.
D) symbolize the legacy of a conflict.

38 38
SAT Punctuation

SAT Writing:
Punctuation

Answers on Page 296

39 39
SAT Punctuation: Comma Usage

Punctuation: Comma Usage

A SAT error is to use commas in a way that does not align with their grammatical function.

Identifying Comma Usage Issues on the SAT: If a comma is located in the sentence or
the answer choices, check to see if it is performing one of the four functions of commas:
separating items in a list; pairing with a conjunction to join two complete
sentences; separating an introductory thought, or separating a descriptive
phrase.

1 3

Not everyone knows that much of Bacteria play an important role in


American law is closely based on sewage reduction by consuming human
medieval English law. In fact, one of waste product. Data has shown that
the rules enacted by the English court without bacteria's involvement,
system in the 17th century is still including consuming various other
relevant in the United States in the materials during the water treatment
21st century. process—rivers and oceans would be
much more polluted.
A) NO CHANGE
B) In fact one of A) NO CHANGE
C) In fact: one of B) process: this is
D) In fact, and one of C) process,
D) process;

Teachers commonly work many more


hours than the official time they spend
in school, typically preparing for class,
grading exams; and performing other
tasks outside the formally defined
school day.

A) NO CHANGE
B) exams and performing
C) exams, and performing
D) exams, and to perform

40 40
SAT Punctuation: Colon Usage

Punctuation: Colon Usage

A SAT error is to use colons in a way that does not align with their grammatical function.

Identifying Colon Usage Issues on the SAT: If a colon is located in the sentence or the
answer choices, check to see if it is performing one of the two functions of colons:
introducing a list or joining a phrase with additional details about that phrase.

4 6

Individuals all over the world can tap Grace, on the other hand, decided to
into developments in three areas: climb every tree she could swing
computing, networking, and software herself into. I was enchanted by the
engineering. tiniest thing of all looking: down,
while my husband tried to snap the
A) NO CHANGE perfect sunset, I thought I saw a
B) the areas of: miniature seashell buried in the rock
C) three areas at my feet.
D) three areas;
A) NO CHANGE
B) all: because
C) all looking,
D) all: looking

For Lawrence, the play brought forth


many memories; playing catch with
his dad, watching the sunset over the
ocean, moving away from home to go
to college.

A) NO CHANGE
B) memories,
C) memories:
D) memories

41 41
SAT Punctuation: Contractions

Punctuation: Contractions

A SAT error is to use apostrophes to form contractions whose constituent words do not fit
within the sentence.

Identifying Contraction Issues on the SAT: If a contraction is located in the sentence


or the answer choices, substitute both of the words in the contraction into the
sentence and then check the sentence using your grammar ear.

7 9

The company staked its robotics My dance instructor, whose


division’s future on using artificial attending my audition, has shared a
intelligence. great deal with me about her
experiences at the American Ballet
A) NO CHANGE Theatre.
B) their
C) it’s A) NO CHANGE
D) its’ B) whose’ attending
C) who’s attending
D) whos’ attending,

Because of they’re nutritional


properties, several servings of fruits
and vegetables should be eaten every
day.

A) NO CHANGE
B) its
C) their
D) there

The SAT mixes possessive and contraction uses of apostrophes!

42 42
SAT Punctuation: Dash Usage

Punctuation: Dash Usage

A SAT error is to use dashes in a way that does not align with their grammatical function.

Identifying Dash Usage Issues on the SAT: If a dash is located in the sentence or the
answer choices, check to see if it is performing one of the three functions of dashes:
separating a descriptive phrase, introducing a change in tone, or presenting an
explanation.

10 12

John was very excited to win the two Tom was gentle, but persistent, with
concert tickets – it was the first time Susan – he knew someday he would
he had ever won a radio contest, and make her his wife.
he knew exactly who he would bring.
A) NO CHANGE
A) NO CHANGE B) Susan, and he
B) contest – and C) Susan, he
C) contest. And D) Susan: and
D) contest: and

11

Caroline also was a great tennis


player, that is a story for another
day.

A) NO CHANGE
B) player; but that
C) process: that
D) player – but that

43 43
SAT Punctuation: Possession

Punctuation: Possession

A SAT error is to use apostrophes in a way that does not correctly indicate possession.

Identifying Possession Issues on the SAT: Look for apostrophes in the sentence or in
the answers choices. Possession errors on the SAT include the text missing an
apostrophe when there is possession, having an apostrophe when there is no
possession, or placing an apostrophe in the incorrect location when indicating
possession.

13 15

Early capitalists believed that To this day, the Saturday Evening


unstable markets would recover on Gazette includes horoscopes based on
their own, without intervention on astrological calendars. The
the part of governments’ or similar newspapers editor, Kerry Finsilver,
regulatory agencies. For example, if says, “The information is essential to
too much of a product is produced, its readers who follow their astrological
price will fall. signs.”

A) NO CHANGE A) NO CHANGE
B) governments B) newspapers’s
C) government’s C) newspaper’s
D) government’s’ D) newspapers’

14

Allowing dodgeball at elementary


schools has come under recent
debate: some people argue that
putting children’s safety in peril is
not worth the fun and exercise.

A) NO CHANGE
B) childrens’
C) childrens’s
D) childrens

Careful! Many subjects that sound plural are actually singular.

44 44
SAT Punctuation: Run-on Sentences and Sentence Fragments

Punctuation: Run-on Sentences and Sentence Fragments

A SAT error is to either use incorrect punctuation when joining two complete sentences or
to use incorrect punctuation when joining two sentence fragments.

Identifying Run-on Sentence and Sentence Fragment Issues on the SAT: Look for
two thoughts joined by some punctuation. Two complete sentences can only be joined using
a period, a semi-colon, or a comma joined with a conjunction.

16 18

My therapist tells me that fear is a Lying within the solar system beyond
normal. Legitimate response to the orbit of Neptune are the dust
genuine danger. Easy for her to say. grains of the Kuiper Belt. Cosmic rays
I’ve never known anyone to be bombard the Earth.
attacked by a pantry, but I still feel a
thrill of panic whenever having to Which choice most effectively
reach for that last, forgotten jar of combines the underlined sentences?
mayonnaise.
A) Dust grains located in the Kuiper
A) NO CHANGE Belt, which lies within the solar
B) normal, legitimate system beyond the orbit of
C) normal; legitimate Neptune, resulting in the Earth
D) normally, legitimate being bombarded by cosmic rays.
B) Dust grains located in the Kuiper
Belt, which lies within the solar
system beyond the orbit of
17 Neptune, result in the Earth being
bombarded by cosmic rays.
Many seed savers order endangered C) Located in the Kuiper Belt, which
plants through a small, but growing lies within the solar system beyond
number of organizations that the orbit of Neptune, dust grains
specialize in rare seeds. that bombard the Earth with
cosmic rays.
A) NO CHANGE D) The Earth’s bombardment by
B) small but growing cosmic rays, as a result of dust
C) small; but growing grains located in the Kuiper Belt,
D) small but growing, which lies within the solar system
beyond the orbit of Neptune.

45 45
SAT Punctuation Review

1 5

Wildfires can be devastating, especially Maggie, a 9200-pound Alaskan elephant,


when dry weather conditions turn needs to lose weight: scientists hope that a
vegetation into kindling. Here in specially designed treadmill will be
California: fall means blistering hot Santa beneficial.
Ana winds that blow in from across the
desert. Usually, the winds just wreak A) NO CHANGE
havoc on my allergies. B) weight, scientists
C) weight, as scientists
A) NO CHANGE D) weight however scientists
B) California. Fall
C) California, fall
D) California fall,
6
2
Consumers’ willingness to spend money is
In Shakespeares theater, tragic heroes crucial to the economy; when people spend
were usually kings or other beings higher more money, companies sell more
in the social scale than the audience. products, make more profits, and employ
more people.
A) NO CHANGE
B) Shakespeare’s A) NO CHANGE
C) Shakespeares’ B) Consumer’s
D) Shakespeare’s’ C) Consumers
D) Consumer’s’
3

Just as all living things are made up of 7


cells, every new cell is produced from an
existing cell. The way cells reproduce is by Research has shown that earthquakes are
dividing, this biological process causes one more likely to occur near tectonic plate
cell to becomes two, and so on. fault lines. This activity can be recorded
with a seismograph.
A) NO CHANGE
B) dividing: and this Which choice most effectively combines the
C) dividing, and this sentences at the underlined portion?
D) dividing and, this
A) NO CHANGE
4 B) lines as
C) lines as the activity is
Julie’s new car has few features. Its D) lines, and this activity can be
reliability, colorful paint job, and leather
seats make it the coolest automobile on the
block.

Which choice most effectively combines the


sentences at the underlined portion?

A) features as its
B) features: its
C) features, therefore
D) features, but its

46 46
SAT Foundations of Arithmetic and Algebra

SAT Math:
Foundations of
Arithmetic and Algebra

Answers on Page 299

47 47
SAT Foundations of Arithmetic and Algebra: Terms and Operations

Factor vs. Multiple


Term Definition
Whole Number A number with no fractional or decimal part.
Integer Any whole number (–400, 0, 2, 1000).
Factor An integer that divides evenly into another integer.
Multiple An integer that can be divided by a smaller integer with no remainder.

Prime vs. Composite


Term Definition
Prime A number with only 1 and itself as factors. 1 is NOT a prime number.
Composite A number with factors other than 1 and itself. 1 is NOT a composite number.

Even vs. Odd


Term Definition
Even An integer which divided by 2 yields a whole number (0 is an even number).
Odd An integer which divided by 2 does not yield a whole number.

Additional Terms
Term Definition
Coefficient A number used to multiply a variable.
Constant A number on its own.
Variable An alphabetic character representing a number.

1 2

NO CALCULATOR NO CALCULATOR

Which of the following is a factor of If a is the greatest prime factor of 28


57 ? and b is the greatest prime factor of
84, what is the value of a – b ?
A) 2
B) 7
C) 13
//
D) 19
/ / /

. . . .

0 0 0 0

1 1 1 1

2 2 2 2

3 3 3 3

4 4 4 4

5 5 5 5

6 6 6 6

7 7 7 7

8 8 8 8

9 9 9 9

48 48
SAT Foundations of Arithmetic and Algebra: Exponents

Term Definition Example

Positive powers Multiply the base by itself as many times as the exponent tells you to. 35 =

Put a one over the base and multiply the base by as many times as the
Negative powers 3–5 =
exponent tells you to.

0 as a power Any number raised to the power of zero equals 1. 30 =

1 as a power Any number raised to the power of 1 equals itself. 31 =

1
Powers on fractions Multiply the fraction by itself as many times as the exponent tell you to. ( 3 )3 =

Powers on negative If a negative number is raised to an even power, it becomes positive. If a


(–3)2 =
numbers negative number is raised to an odd power, it stays negative.

Multiplying powers When multiplying similar bases with powers, add the powers together. 33 x 34 =

35
Dividing powers When dividing similar bases with powers, subtract the powers. ( )=
34
Raising a power to
When raising a power to another power, multiply the powers. (35)3 =
another power
The numerator of the fractional power represents the power and the 1
Fractional powers 32 =
denominator of the fractional power represents the root.

3 5

NO CALCULATOR 𝑥𝑞
2
If 2 = 𝑥12 , x > 1, and q + r = 36,
𝑥𝑟
Which real number satisfies (3x)(27) what is the value of q – r ?
= 912 ?

// / / /

. . . .

0 0 0 0
4 1 1 1 1

2 2 2 2
NO CALCULATOR 3 3 3 3

4 4 4 4
4 5 5 5 5

Which of the following is equal to 𝑡 11 6 6 6 6

for all values of t ? 7 7 7 7

8 8 8 8

9 9 9 9

A) 𝑡4
B) 𝑡11
11
C) 𝑡4
4
D) 𝑡11

49 49
SAT Foundations of Arithmetic and Algebra: Absolute Value

Foundations of Arithmetic and Algebra: Absolute Value

An absolute value is a number’s distance from zero. As such, absolute values are
never negative.

Identify: Any problem that uses absolute value bars …

Set Up: Create two equations and solve for the two values of the given variable.

6 8

Which of the following expressions is NO CALCULATOR


equal to 0 for some value of x ?
For what value of n is |n – 4| + 1
A) 4 – 𝑥 + 4 equal to 0 ?

B) 𝑥 – 4 + 4 A) 0
B) 1
C) 𝑥 + 4 + 4 C) 4
D) There is no such value of n.
D) 𝑥 + 4 – 4

If |3x – 8| < 4, then which of the


following CANNOT be a value of x ?

5
A) 3
B) 2
C) 3
D) 4

50 50
SAT Foundations of Arithmetic and Algebra: Percentage

Foundations of Arithmetic and Algebra: Percentage

A percentage is a number or ratio expressed as a fraction of 100.

Identify: Questions that involves percentages or give “before and after numbers.”

Set Up: To determine percent change, plug any elements you know into the percent
change formula.

Percent Change Formula


Final – Original
Percent Change = x 100%
Original

9 10

A local comedian earns $80 at a Timothy goes to the store to buy


performance where 25 people attend. himself a new suit, and he finds that
The performer uses 32% of the money there is a 16% off sale going on.
earned to pay the costs involved in Additionally, Timothy has a 25%
putting on the performance. The rest discount coupon. If Timothy buys the
of the money earned is the suit that is on sale and also uses his
performer’s profit. What is the profit coupon, what will his total discount
the performer makes at a be?
performance where 25 people attend?
A) 31%
A) $25.60 B) 37%
B) $32.00 C) 41%
C) $54.40 D) 45%
D) $62.80

11

The plans for a rectangular building


were altered by decreasing the length
of the building by 50 percent and
decreasing its width by p percent. If
these alterations decreased the area of
the building by 80 percent, what is the
value of p ?

51 51
SAT Foundations of Arithmetic and Algebra Review

1 4

NO CALCULATOR Jennifer is a coach reviewing the previous


soccer season. She recorded that Player A
If (x6) j = x72 , what is the value of j ? scored 60 percent more goals than Player
B did during the season. Based on
Jennifer’s calculations, if Player A had 128
goals, how many goals did Player B have?

A) 51
B) 80
C) 108
D) 205
2

On the surface of the planet Venus, the


force of gravity is 12% less than that of the
force of gravity on the surface of Earth. If
Steven weighs 160 pounds on the surface
of Earth, what would his weight be on the
surface of Venus, in pounds?

A) 19.2
B) 28.8
C) 140.8 5
D) 148
Jason’s dealership sells cars and trucks.
Last month, his dealership sold 80 cars
and 54 trucks. This month, the number of
cars sold declined by 20 percent, and the
number of trucks sold increased by 50
3 percent. What was the percent increase in
the total number of the two items sold by
NO CALCULATOR the dealership? (Round your answer to the
nearest tenth.)
3x
5 + 4 > 10 is equivalent to which of the
following expressions?

// / / /
70
A) – 3 < x < 10
. . . .

0 0 0 0

35 1 1 1 1
B) 10 < x <
2 2 2 2 2

3 3 3 3
70
C) x < – 3 and x > 10 4 4 4 4

5 5 5 5

70 6 6 6 6
D) x < –10 and x > 3 7 7 7 7

8 8 8 8

9 9 9 9

52 52
SAT Passage Analysis

SAT Writing:
Passage Analysis

Answers on Page 296

53 53
SAT Class 3: Passage Analysis: Adding Precision

Passage Analysis: Adding Precision

Identifying Passage Analysis: Adding Precision Questions on the SAT: The


question will ask which answer provides the most relevant details. Look for the answer
that aligns with the main idea and provides the most vivid imagery through the use of
nouns, adverbs, and adjectives.

Question 1 Refers to the Paragraph Below

My grandmother’s hesitancy of taking the 1


elevator to the 62nd floor makes sense because
she has a terrible fear of heights – 1 to the Which choice best completes the
point of insisting on living only in single-story description of the grandmother’s fear of
homes. It frustrates my grandfather, but I heights?
understand. After all, you can fall down stairs!
A) NO CHANGE
B) she had a phobia.
C) her fear was irrational.
D) heights really frightened her.

Question 2 Refers to the Paragraph Below

The painting was created to highlight the 2


sharp hues that are seen in nature, so that a
person can, for a while, escape the urban Which choice gives a second supporting
malaise. A pond, for example, is a sharp crystal example that is most similar to the
blue, and 2 a frog is sitting stately on a lily example already in the sentence?
pad.
A) NO CHANGE.
B) there is even a flock of birds soaring in
the sky.
C) a frog is a dark majestic green.
D) a variety of colors is seen in the
artwork.

54 54
SAT Passage Analysis: Introductions and Conclusions

Passage Analysis: Introductions and Conclusions

Identifying Passage Analysis: Introductions and Conclusions Questions on the


SAT: The question will ask what answer best introduces or concludes a given
paragraph or passage. Any introduction or conclusion must connect to the main idea(s)
of the paragraph or passage.

Question 3 Refers to the Paragraphs Below

For many decades, studies have shown a 3


strong link between car crashes and driver
distractions. Driving while using a cell phone has Which choice most effectively sets up the
risen to the top of a long list of possible reasons paragraph?
for loss of control of a vehicle. Many countries
have enacted legislation banning the use of cell A) Today everyone has a cell phone in his
phones while driving. Some businesses are also or her pocket.
developing policies that disallow their employees B) Cell phones are one of the leading
from using cell phones while driving, an action causes of car accidents.
that is also designed to protect the company from C) Cell phones have recently grown in
liability in case of an accident. Some statutes popularity.
require cell phones to be “hands-free,” although D) Increasingly, the popularity of cell
studies indicate that problems will still exist with phone use has come under much
a “hands-free” phone. It is nearly impossible to scrutiny.
maintain a strong level of concentration on the
road while engaged in, say, an intense or
important conversation, whether holding the
phone or not.

Question 4 Refers to the Paragraph Below

Some call it a state-sponsored vice, and others 4


believe it is a regressive and voluntary tax on the
poor. Still millions of people across the United Which choice most effectively concludes
States and around the world line up to play the the paragraph?
lottery and indulge their fantasies of wealth. The
justification for holding state lotteries is that A) NO CHANGE
lottery profits are intended to be spent on B) Only 34 cents of each dollar spent on a
education and other underfunded public entities lottery ticket actually goes into the
such as parks. However, schools have not seen the state budget.
windfalls that lottery proponents promised. Some C) The state lottery is a flawed
school districts report that education bonds have government program.
become harder to pass because of the public D) State lotteries are wrongfully
perception that the schools are getting rich from detrimental to the public.
the lottery. 4 Such exploitation of people’s hopes
should be prohibited.

55 55
SAT Class 3: Passage Analysis: Main Idea Analysis

Passage Analysis: Main Idea Analysis

Identifying Passage Analysis: Main Idea Analysis Questions on the SAT: The
question will ask what meaning will be added or lost if a word or phrase is added,
deleted, or modified, or which choice most effectively develops a main idea. Remember
to align your answer with the main idea(s) of the surrounding text.

Question 5 Refers to the Paragraph Below

I was first introduced to Murano glass 5


during an art history course I took my junior
year of college. Our professor taught us that Which choice most effectively sets up the
Murano was the name of a group of islands in information that follows?
the lagoon of Venice that has been an important
center of glass production since medieval times. A) NO CHANGE
For centuries, Murano glassmakers held tightly B) led to increased notoriety.
to their trade secrets and were paid well for C) took its toll.
their beautiful pieces. But the inevitable market D) brought about the end of Murano
reaction of cheaper glass 5 increased demand glass.
for the product. The art of making Murano glass
was almost lost. Thankfully, however, Murano
was given a kick-start in the nineteenth century.
By the twentieth century, glassmakers began
playing with traditional forms and collaborating
with progressive artists and designers.

Question 6 Refers to the Paragraph Below


6
Nine times as many Americans fell in the At this point, the writer is considering
farmlands near Antietam Creek as fell on the adding the following true statement:
beaches of Normandy on D-Day, the so-called
“longest day” of World War II. It was in 1862, Approximately 4,000 American
during the Civil War, that General Robert E. soldiers died in combat during the
Lee’s Confederate Army first engaged Union Revolutionary War.
forces on northern soil. According to historians,
when fighting had subsided, over 23,000 soldiers Should the writer make this addition here?
lay dead or wounded. This was more than all the
dead or wounded Americans in the A) Yes, because it reinforces the point
Revolutionary War, War of 1812, Mexican War, that Americans suffered many
and Spanish-American War combined. 6 casualties during the Civil War.
B) Yes, because it provides a logical
transition to the rest of the paragraph.
C) No, because it is irrelevant to the focus
of the paragraph to this point.
D) No, because it does not support the
statements made in the previous
sentence.

56 56
SAT Passage Analysis: Sequence

Passage Analysis: Sequence

Identifying Passage Analysis: Sequence Questions on the SAT: The question will ask
which sequence of sentences within a paragraph, or which sequence of
paragraphs within a passage, is most logical. To deduce proper sequencing, look for a
topic sentence to use as Sentence 1 or look for connections between ideas to link
different sentences.

Question 7 Refers to the Paragraph Below

[1] With great fanfare, Lee and fellow 7


General “Stonewall” Jackson marched their
ragged Army of Northern Virginia across the The writer wants to add the following
Potomac River and through the Frederick town sentence to the paragraph.
square. [2] Soldiers obeyed Lee’s order to refrain
from violence and pillaging for several days, and Lee issued the Proclamation to the
the townspeople maintained tacit compliance People of Maryland to invite the
and sold food, clothes, and shoes to Southern citizens to join the Southern
troops. [3] Lee keenly observed, however, that movement.
while pleasant, the people of Maryland had no
sympathy for the Confederate side. [4] He The best placement for the sentence is
needed a revised plan. immediately

A) before sentence 1.
B) before sentence 2.
C) before sentence 3.
D) before sentence 4.

Question 8 Refers to the Paragraph Below

[1] Some of Gandhi’s nonviolent actions 8


included hunger strikes and quiet movements.
[2] He felt that all people were equal; he himself To make this sentence more logical,
was Indian and knew how it felt to be treated sentence 5 should be placed
badly. [3] He stood up for the lowest class and
lived like the rest of his people. [4] He lived in a A) where it is now.
hut, made his own clothes, and tended to fruit B) after sentence 1.
trees. [5] With these actions he protested the C) after sentence 2.
fact that certain groups of people were treated D) after sentence 3.
worse than others.

57 57
SAT Passage Analysis Review

Question 1 Refers to the Paragraph Below

By the end of World War II, more than 20 1


million Americans transformed backyards,
empty lots, baseball fields, schoolyards, and At this point, the writer wants to further
apartment rooftops into over 20 million Victory reinforce the paragraph’s claim about all
Gardens, which yielded between 9 and 10 Americans participating in the war effort.
million tons of produce. Distributed small-scale Which choice most effectively accomplishes
agriculture provided sustenance to American this goal?
troops and civilians alike; the summer and fall
crops were eaten fresh or dutifully canned for A) Despite the challenging conditions
the winter and spring by amateur gardeners. brought about by World War II,
1 Victory Gardens were their way of taking B) Acting without government oversight,
the fight to the enemy. It wasn’t uncommon for C) As they helped provide needed
them to say “My food is fighting!” resources for American soldiers,
D) For Americans young and old alike,

Question 2 Refers to the Paragraph Below


2
The next day, we walked out to one of the
spectacular view sites overlooking the Grand Which choice provides information that
Canyon. While my husband took more photos of best supports the statement made in the
the sprawling vista, I again looked down at my previous sentence?
feet. This time, there was no doubt. The ground
was covered with dozens of tiny shells and the A) because the ocean is full of interesting
traces of tiny plants, all seemingly fused into the creatures.
surrounding rock, which, strangely, looked like a B) because my imagination often got the
frozen sea bed. I was amazed – surely I was better of me, especially while on
imagining things. 2 There was no sea vacation.
anywhere near here. How could these be shells? C) because we had driven through the
dry, barren Mojave Desert to reach the
Grand Canyon.
D) because the Grand Canyon was a very
popular tourist attraction
Question 3 Refers to the Paragraph Below
3
Some argue that because human stomach
acids are not nearly as potent as those of other Which choice most logically follows from
animals, humans are better suited for digesting the preceding sentence?
plant material. It is our weaker stomach acids
that force humans to cook most of our meat. 3 A) The human stomach secretes up to 3
liters of gastric acid per day.
B) Humans are incapable of eating and
digesting raw meat.
C) Cooking helps to kill bacteria and
makes the flesh more easily digestible.
D) Cooking can be time consuming, but
most humans prefer to eat cooked
meat.

58 58
SAT Passage Analysis Review

Question 4 Refers to the Paragraphs Below

I would be starting college in a month and 4


decided that, instead of living in the stark and
unimaginative dorms, I would live in a historic Which choice best connects the sentence
neighborhood within walking distance of my with the previous paragraph?
school. Some of these grand homes were owned
by single families, but many had been divided A) NO CHANGE
into affordable apartments. This was fortunate B) Finances had been a little tight, and I
for me because I’m sure my parents would never didn’t want to burden my parents.
consider buying me a mansion to live in when I C) I have always enjoyed learning about
attended college! the past, so I knew I would love living
4 It came with a fireplace – complete with a in a home from another era.
mantle and a generous hearth – intricate crown D) We had already looked at one
molding, original light fixtures with crystals, apartment that was absolutely
metalwork, and old-fashioned switches, and a beautiful.
huge bay window. My mom said that this place
was probably beyond their budget, but I thought
it would be worth a try to convince them that
they should rent it for me anyway.

Question 5 Refers to the Paragraph Below

Adam Smith’s “invisible hand” has come to 5


stand for the ability of the market to recover
naturally 5 after apparently disastrous The writer is considering deleting the
circumstances. Smith believed that unstable underlined phrase. Should the phrase be
markets will recover on their own, without kept or deleted?
intervention on the part of the government or
similar regulatory agencies. For example, if too A) Kept, because it is an indication of the
much of a product is produced, its price will fall. market’s strong ability to recover.
This encourages the public to buy the product, B) Kept, because it is descriptive
which reduces supply. This kind of reaction language that helps to set the tone of
eventually leads to stabilization, at the “natural the paragraph.
price” of a good or service. C) Deleted, because it is an unimportant
detail.
D) Deleted, because it is irrelevant to the
focus of the paragraph to this point.

59 59
60 60
SAT Plug and Chug & Backsolving

SAT Math:
Plug and Chug &
Backsolving

Answers on Page 299

61 61
SAT Plug and Chug & Backsolving: Plug and Chug

Plug and Chug & Backsolving: Plug and Chug

Identify: Problems with variable(s) in the question and variable(s) in the answer choices.
The phrase “in terms of” often indicates a plug and chug question.

Set Up: Pick number(s) for the variable(s) and solve the question as though the variable(s)
is/are your number(s). Plug your number(s) into the answer choices and look for the same
value. If more than one answer choice works, pick different number(s) to eliminate the
remaining choice(s).

1 3

If z is an even integer less than zero, NO CALCULATOR


what is the greatest value of 2z – z ?
If j – k < 2, which of the following
A) –1 must be true?
B) –2
C) –3 I. j + k < 2
D) –4 II. j < 5 + k
III. k – j > – 2

A) II only
B) III only
C) I and III
2 D) II and III
NO CALCULATOR

Which of the following equations has


a graph in the xy-plane for which y is
always greater than or equal to −4 ?

A) y= 𝑥 −4
B) y = (x − 2)3
C) y = −x2 + 4
D) y = x2 − 10

62 62
SAT Plug and Chug & Backsolving: Backsolving

Plug and Chug & Backsolving: Backsolving

Identify: Problems with variable(s) in the question and numbers in the answer choices.

Set Up: Plug the answer choices into the question as the variable(s). Look for the answer
that generates the desired value.

4 6

For what value of x is │x – 3│ + 4 = 0 ? NO CALCULATOR


A) –3 If 0 < xy < x < y, which of the
B) –1 following ordered pairs represents a
C) 1 possible set of values for (x, y) ?
D) There is no such value of x.

A) (0, 4)

B) (4, 0)

2 2
5 C) ( 3, 3 )

NO CALCULATOR 1 7
D) ( 8, 8 )

Which ordered pair (x, y) satisfies the


system of equations shown below?

x + 5y = 10
5x + 10y = 35

A) (1, 5)
B) (5, 1)
C) (10, 0)
D) (0, 10)

63 63
SAT Plug and Chug & Backsolving Review

1 3

What are all the possible values of b NO CALCULATOR


such that ab2 = 63, a < 10, b < 10,
and a and b are integers? Which of the following expressions
A) –3, 3 has a positive value for all a and b
B) 1, 3 such that a < 0 and b > 0 ?
C) 1, 9
D) 3 A) a − b

B) ab3

C) a2b2
b2
D) a
2

NO CALCULATOR

Which ordered pair (x, y) satisfies the 4


system of equations shown below?
NO CALCULATOR
10x – 4y = 36
5x + y = 6 a b
+ =x
b a
A) (4, 1) a b
– =y
B) (1, 4) b a
C) (2, −4)
D) (0, 6) In the equations above, if a ≠ 0 and b
≠ 0, which of the following must be
equal to x2 – y2 ?

A) 2
B) 4
C) 2ab
D) 4a2b2

64 64
SAT Translating Word Problems

SAT Math:
Translating Word
Problems

Answers on Page 299

65 65
SAT Translating Word Problems: Translation Table

Translating Word Problems: Translation Table

Use the following chart to assist in translating word problems into algebra.
.

Term Translation
“Each,”
“Groups of (a number),”
“Number of Times,” Variable (x , y, etc.)
“Some”
“Equals,” “Is” =
“And,” “Plus,” “Sum” +
“Difference” –
“Product” x

“For every,” “Per (ratio)” ÷


“Exceed(s),”
“Greater Than” >

“Meet or exceed,”

“Greater Than or Equal To”
“Less Than” <
“Less Than or Equal To” ≤

66 66
SAT Translating Word Problems: Linear Equations

Translating Word Problems: Linear Equations

Identify: A word problem that mentions a linear model. A linear model relates to the
formula y = mx + b, where m is the slope (the rate of change) and b is the y-
intercept (the amount when x = 0).

Set Up: Plug the given information into slope intercept form (y = mx + b) and then
solve for the requested information.

1 3

NO CALCULATOR When a scuba diver submerges in


water to a depth of 15 feet below the
A postal worker delivers to 150 surface, the pressure due to the
homes. The postal worker earns $12 atmosphere and surrounding water
an hour and an additional $15 bonus is 24.2 pounds per square inch. As
when all of the mail is delivered on the scuba diver descends, the
time. If the postal worker delivers all pressure increases linearly. At a
of the mail on time, what expression depth of 23 feet, the pressure is 32.4
could be used to determine how much pounds per square inch. If the
the postal worker earned? pressure increases at a constant rate
as the scuba diver's depth below the
A) 12x + 15, where x is the number of surface increases, which of the
hours. following linear models best
B) 15x + 12, where x is the number of describes the pressure p in pounds
hours. per square inch at a depth of f feet
C) (12 + 15) + x, where x is the below the surface?
number of homes.
D) 12x + (15 + 150), where x is the A) p = 0.976f – 8.825
number of homes. B) p = 0.976f + 8.848
C) p = 1.025f + 8.825
2 D) p = 1.025f – 8.848
The monthly membership fee for a
local gym is $35.00. The cost of free
weights and cardio equipment is
included in the membership fee, but
there is an additional fee of $1.75 for
each use of the basketball court. For
one month, Mindi’s membership and
basketball court fees were $43.75.
How many times did Mindi use the
basketball court that month?

67 67
SAT Translating Word Problems: Non-Linear Equations

Translating Word Problems: Non-Linear Equations

Identify: After translating a word problem equation there is one equation and/or you are
solving for the value of a variable.

Set Up: Follow the below steps after translation (where necessary):

1) Group terms into a single equation with an equals or inequality sign.


2) Plug any information provided in the question into the equation.
3) Solve for the equation and/or for the requested variable.

4 6

To rent a recording studio, Alpha NO CALCULATOR


Records charges a flat $250 fee plus
$25 per hour to use the studio and The gas mileage for Peter’s car is 30
Beta Records charges a flat $100 fee miles per gallon when the car travels
plus $75 per hour to use the studio. If at an average speed of 45 miles per
q represents the number of hours hour. The car’s gas tank has 14
spent using a studio, what is the gallons of gas at the beginning of a
minimum value of q for which Beta’s trip. If Peter’s car travels at an
total charge is equal to or greater average speed of 45 miles per hour,
than Alpha’s total charge? which of the functions f models the
number of gallons of gas remaining
in the tank t hours after the trip
begins?

5 30
A) f(t) = 14 – 45t
James is renting a hotel room for his
vacation to Berlin. The hotel charges 14 –30t
a one-time flat fee of $200 plus $110 B) f(t) = 14 – 45
per day. A tax of 5% is also applied to
the daily cost. Which of the following 45t
C) f(t) = 14 – 30
represents James’s total cost, in
dollars, for renting a hotel for h 17 –45t
D) f(t) = 14 –
days? 30

A) 1.05(110h) + 200h
B) 1.05(310h)
C) 1.05(110h + 200)
D) 1.05(110h) + 200

68 68
SAT Translating Word Problems: System of Equations

Translating Word Problems: System of Equations

Identify: After translating a word problem equation there are multiple equations
and/or inequalities.

Set Up: Follow the below steps (where necessary):

1) Group like terms (variables with the same exponent and/or whole numbers).
2) Add, subtract, or substitute between the given equation.
3) Solve for the equations and/or for the requested variable.

7 8

NO CALCULATOR Stephen participated in a game


involving the flipping of a two-sided
During a given week Janet buys x coin. Each time the coin landed on
loaves of bread costing two dollars heads Stephen would receive two
each and y cartons of eggs costing four dollars, and each time the coin landed
dollar each. If last week Janet bought on tails Stephen would receive five
a total of 6 items and spent 22 dollars, dollars. Stephen was allowed to flip
which of the system of equations the coin 10 times. If after those flips
would yield the number of loaves of Stephen earned 35 dollars, how much
bread Janet bought? of his earnings (in dollars) came from
the times he flipped heads?
A) x+y=6
2x + 4y = 22

B) x + y = 22 9

x + 4y = 6 Tommy and Sully each ordered a


breakfast at a diner. The price of
C) x+y=6 Tommy’s breakfast was s dollars, and
4x + 2y = 22 the price of Sully’s breakfast was $10
more than the price of Tommy’s
D) x + y = 22 breakfast. If Tommy and Sully split
x + 2y = 6 the cost of the breakfasts evenly and
each paid a 30% tip, which of the
following expressions represents the
amount, in dollars, each of them paid?
(Assume there is no sales tax.)

A) 0.6s + 10
B) 1.3s + 6.5
C) 1.3s + 3
D) 2.6s + 1.5

69 69
SAT Translating Word Problems Review

1 3

While preparing to participate in a charity


NO CALCULATOR
bicycle ride, Karen created a training
schedule in which the distance of her
John exercises by doing sit ups, a, in sets
longest ride every week increased by a
of 10 and push ups, b, in sets of 20. If last
constant amount. If Karen’s training
week John did 20 sets of exercise for a
schedule requires that her longest ride in
total of 450 completed movements, which
week 2 is a distance of 10 miles and her
of the system of equations below would
longest ride in week 20 is a distance of 46
yield the number of push ups John
miles, which of the following best describes
completed?
how the distance Karen rides changes
between week 2 and week 20 of her
A) a + b = 450
training schedule?
20a + 10b = 20
A) Karen increases the distance of her
B) a + b = 20
longest ride by 0.5 miles each week.
B) Karen increases the distance of her 20a + 10b = 450
longest ride by 2 miles each week.
C) Karen increases the distance of her C) a + b = 450
longest ride by 5 miles every 2 weeks. 10a + 20b = 20
D) Karen increases the distance of her
longest ride by 3 miles each week. D) a + b = 20
10a + 20b = 450

2 During a town’s annual singing


competition, groups are divided into duos
NO CALCULATOR (groups of two) and trios (groups of three).
In this year’s competition, there were 108
Christina is a member of a gym which singers in a total of 42 groups. How many
charges $45 per month plus $1.50 each of the singers were in duos?
time a member uses the tennis court.
Which of following functions gives
Christina’s cost, in dollars, for a month in
which she uses the tennis court j times?

A) C(j) = 45 + 1.5j
B) C(j) = 45j + 1.5 // / / /

C) C(j) = 45j . . . .

D) C(j) = 46.5j 0 0 0 0

1 1 1 1

2 2 2 2

3 3 3 3

4 4 4 4

5 5 5 5

6 6 6 6

7 7 7 7

8 8 8 8

9 9 9 9

70 70
SAT Translating Word Problems Review

Questions 5 and 6 relate to the Questions 7 and 8 relate to the


following information. following information.

Sir Isaac Newton’s Second Law of Motion Heather and her friends celebrate her
states that the vector sum of the birthday each year at a family
external forces F on an object, in newtons entertainment center. For every dollar spent,
(N), is equal to the mass m of that object, Heather and her friends receive a certain
in kilograms (kg), multiplied by the number of tokens that can be used to play
acceleration vector a of the object, in games. While making the conversion from
meters per second squared (m/s2). dollars to tokens, the entertainment center
charges a flat fee of 2% on each dollar spent.
Dr. Shao calculates that a projectile This charged fee is not converted into tokens.
weighing 0.0018 kg and traveling with an
acceleration of 5.7 x 10^5 m/s2 will have a This year, Heather and her friends
total force of 1026.1 N. purchased 2500 tokens at the entertainment
center for 675 dollars.
5
Newton’s Second Law of Motion can be 7
applied to any object in motion, such as a
bowling ball rolling down a bowling lane or What was the exchange rate, in tokens per
a plane taking off from an airport runway. one dollar? (Round your answer to the
Dr. Shao calculates that a racecar nearest tenth.)
weighing 850,000 grams has a total force
of 40,000 newtons when crossing the finish
line at the end of the first lap. What was
the acceleration of the racecar at the
moment of crossing the finish line, in
meters per second squared? (Round to the 8
nearest hundredth.)
The family entertainment center sells a
prepaid gift card worth 10,000 tokens.
Heather and her friends can buy the prepaid
gift card using dollars at the entertainment
6 center’s exchange rate with no fee, but they
As a race progresses, the racecar loses will lose any tokens left unused on the
fuel. At the end of a later lap, the racecar prepaid card at the end of the day. What is
is calculated to weigh 840 kilograms and the least number of the 10,000 tokens on the
to have an acceleration of 61.72 meters per prepaid gift card Heather and her friends
second squared. At the end of this lap, the must use for the prepaid gift card to be
total force of the racecar is what percent cheaper than buying all of the tokens with
greater than the total force of the racecar the 2% fee? (Round your answer to the
at the end of the first lap. (Note: Ignore the nearest whole number of tokens.)
percent symbol when entering your
answer. Round to the nearest tenth of a
percent.)

71 71
72 72
SAT Command of Evidence

SAT Reading:
Command of Evidence

Answers on Page 294

73 73
SAT Command of Evidence

Command of Evidence

Identifying Command of Evidence Questions on the SAT: Command of Evidence


questions ask for which line(s) “provide(s) the best evidence for the answer to the
previous question.”

On Command of Evidence questions, pick the lines with the main idea that most-closely
aligns with the answer to the previous question.

If unsure, eliminate down to as few choices as possible before choosing an answer.

In the past ten years, the cacophony of the 1


modern world has expanded to include the various
chirps, buzzes, and rings of cell phones. Some Which choice best summarizes the second
Line market research suggests that four out of five adults paragraph (lines 11-28)?
5 carry cell phones, a number that has been steadily
growing. When a call comes through, some phones A) While some young people are involved in
burst into the owner’s favorite song, while others are saving endangered species, many others are
reminiscent of the ring of conventional land-line more interested in ringtones.
telephones. The ringtone selection from which B) It is inevitable that some environmental
10 consumers can personalize their own phones is vast. groups will use deception to increase
One of the newest innovations in the world of awareness among younger generations.
ringtones allows consumers to download the sounds C) Young people tend to place too much
of various threatened or endangered animals to their political significance on their ringtones.
phones. In an effort to inspire and educate a new D) Ringtones are one way younger generations
15 generation of ecologically-minded youth, can be inspired to get involved in the
environmental groups are creating ringtones that environmental movement.
model the clickety-click sound of the Central
American poison arrow dart frog or the goose bump-
raising howl of a Mexican gray wolf. Other choices
20 that environmentally conscious cell phone owners
can select from include the gentle hooting of several 2
North American owls, the melodies of South Which choice provides the best evidence for the
American birds, and the resonating bellow of the answer to the previous question?
Arctic Beluga. Organizations that offer these
25 ringtones for free are optimistic that people will be A) Lines 9-10 (“The ringtone…vast”)
intrigued enough to want to find out more about the B) Lines 11-14 (“One of…phones”)
creatures. Ideally, these ringtone users will seek out C) Lines 19-24 (“Other…Beluga”)
ways to actively help threatened species. D) Lines 24-28 (“Organizations…species”)

74 74
SAT Command of Evidence

Command of Evidence

Identifying Command of Evidence Questions on the SAT: Command of Evidence


questions ask for which line(s) “provide(s) the best evidence for the answer to the
previous question.”

On Command of Evidence questions, pick the lines with the main idea that most-closely
aligns with the answer to the previous question.

If unsure, eliminate down to as few choices as possible before choosing an answer.

The month was September and the place was in someone come out upon it with a quick, buoyant
the neighborhood of Bridgetown, on the island of step, and then she saw him stop and gaze steadily
Barbados. The seventeenth century was not up the river. At this she turned her head, and her
Line seventeen years old, but the girl who walked slowly 45 eyes went out over the beautiful landscape and the
5 down to the river bank was three years its senior. wide terraces rising above each other towards the
She carried a fishing-rod and line, and her name sky.
was Kate Bonnet. She was a bright-faced, quick- It is astonishing how soon after this a young
moving young person, and apparently did not expect man dressed in a brown suit, and very pleasant to
to catch many fish, for she had no basket in which to 50 look upon, came rapidly walking along the river
10 carry away her finny prizes. Nor, apparently, did bank. This was Master Martin Newcombe, a young
she have any bait, except that which was upon her Englishman, not two years from his native land,
hook and which had been affixed there by one of the and now a prosperous farmer on the other side of
servants at her home, not far away. In fact, Mistress the river.
Kate was too nicely dressed and her gloves were too
15 clean to have much to do with fish or bait, but she
seated herself on a little rock in a shady spot not far
from the water and threw forth her line. Then she
gazed about her – a little up the river and a bit down 3
the river. The passage suggests that Kate believes the
20 It was truly a pleasant scene which lay before her proper attitude is one of
eyes. Not half a mile away was the bridge which
gave this English settlement its name, and beyond A) thoughtfulness.
the river were woods and cultivated fields, with here B) wistfulness.
and there a little bit of smoke, for it was growing C) annoyance.
25 late in the afternoon, when smoke meant supper. D) repose.
Beyond all this the land rose from the lower ground
near the river and the sea, in terrace after terrace,
until the upper stretches of its woodlands showed
clear against the evening sky.
30 But Mistress Kate Bonnet now gazed steadily 4
down the stream, beyond the town and the bridge, Which choice provides the best evidence for
and paid no more attention to the scenery than the the answer to the previous question?
scenery did her, although one was quite as beautiful
as the other. This, she believed, was the proper A) Lines 1-3 (“The month…Barbados”)
35 attitude for fishing. There was a bunch of white B) Lines 20-21 (“It was…eyes”)
flowers in the hat of the young girl. It was not a very C) Lines 26-29 (“Beyond…sky”)
large one, and not a very small one, but of such a D) Lines 30-35 (“But…fishing”)
size as might be easily seen from the bridge, had
any one happened to be crossing about that time.
40 And, in fact, as the wearer of the hat and the white
flowers still continued to gaze at the bridge, she saw

75 75
SAT Command of Evidence Review

Questions 1-10 are based on the following passage. the French Revolution, Louis XVI lost his head
and Choiseul his job. The greatest haul of
In the following passage, a scholar discusses the Parthenon sculptures was to be made by British
Parthenon – a Greek temple renowned for its nobleman and diplomat Thomas Bruce,
architectural beauty – and the controversy 50 the Earl of Elgin, who was inadvertently to
surrounding the ancient sculptures known as the impart a new word to the French language:
“Elgin Marbles” that once adorned the temple. “elginisme,” defined as a “form of vandalism
consisting of taking works of art from their
I first saw the Parthenon in 1954 from the deck countries of origin to put them in public or
of an elderly liner named the Nea Hellas as she 55 private collections.”
approached the dock at Piraeus. Distance hid the Elgin’s initial purpose was merely to make
Line temple’s scars, and it had many, for it was almost drawings and plaster casts of the Parthenon
5 two and a half millennia old. It was begun in 447 Marbles to serve as models for English taste. The
BC when Athens was the leader of an alliance to person most responsible for diverting Elgin’s
check the ambitions of the Persian Empire. By 60 project from its original intention was his
mid-century, it was clear that Persia’s chaplain, Philip Hunt. A firman1 from the
expansionism was well in check, and Athens Ottoman government gave Elgin’s party the
10 redirected the revenues of her alliance into a explicit right to measure, draw, and make plaster
building program. The tour guides who lead the casts of the sculptures. But the final
hordes of sightseers around the Acropolis 65 controversial clauses of the firman, inspired by
nowadays like to refer to the Parthenon as the Hunt, instructed Athens authorities not “to make
symbol of Athenian democracy. In fact, it is a any opposition to the taking away of some pieces
15 monument to profitable imperialism. of stone with inscriptions, and figures.”
But great works of art and architecture On the basis of this vague authorization, Elgin
transcend the motives of their founders. Imperial 70 took away 247 feet of the frieze that ran round
Athens became a model for the ancient world and the outside of the cella wall, seventeen figures
remains the heartland of classical culture. The from the pediment, a caryatid from the Porch of
20 Parthenon helped guarantee that. It may not have the Maidens . . . In fact, Hunt contemplated
symbolized democracy, but it did come to symbolize taking away all of this exquisite temple, but the
what the classical heritage was all about. 75 British Navy would have had to contribute a
With the eighteenth century, the rediscovery of frigate, and no ship was to be had. Louis Fauvel’s
Greece began. In 1748, a British society of wagon was the only vehicle in Athens that could
25 noblemen who had visited Italy and acquired an transport heavy sculptures to the port. It was
appreciation of classical architecture commissioned 80 fortunate that the early nineteenth century did
James Stuart, a painter-architect, and Nicholas not have modern moving equipment.
Revett, an architect draughtsman, to visit Greece Elgin was not immune to the profit motive,
and produce drawings of as many of the ancient but he made no money out of the “Elgin Marbles”
30 monuments as they could. Stuart and Revett when he sold them in 1816 to the British
founded the Greek Revival style, which would ride 85 government. He also lost his reputation, for
the full flood of the nineteenth century which Lord Byron2 was partly responsible, for in
romanticism and, in the United States, would the controversy surrounding the removal of the
become virtually the official architecture of the Parthenon sculptures from Athens, Byron’s voice
35 new republic. carried special weight. “Fair Greece! sad relic of
The collectors soon followed. The French were 90 departed worth/Immortal, though no more;
in the forefront. Comte Marie-Gabriel de Choiseul though fallen, great!” he wrote in Childe Harold’s
Gouffler became Louis XVI’s ambassador to Pilgrimage. Byron popularized what was a novel
Turkey, and his agent in Athens was Louis Fauvel, idea at the time, that a country’s art is its
40 who contrived to corner the antiquities market and heritage. Art – if it is old enough – consists not
imported a wagon that was the only conveyance in 95 merely of articles to be bought and sold at
Athens sturdy enough to carry off large pieces of auction. It expresses the spirit of a people.
sculpture. If all had gone smoothly, the “Parthenon The question of the repatriation3 of the
Marbles,” as the Greeks call them, would not be in Marbles began virtually at the same time that
45 the British Museum but in the Louvre. But with Britain decided to purchase them. In 1983, Greek

76 76
SAT Command of Evidence Review

100 Minister of Culture Melina Mercouri held an 1


emotional press conference in London on the
Marbles. “You must understand us . . . [the According to the author, the Greeks’
Marbles] are the symbol and the blood of the Greek motivation for building the Parthenon was to
people,” she said, “This is our history; this is our
105 soul.” A) install their form of government
Lately the Greek argument has adopted a softer throughout the classical world.
tone. Who owns the Marbles does not matter; what B) influence future European artistic
matters is location. In return for giving the movements.
Marbles a home in a new Acropolis Museum, C) glorify national superiority through
110 Greece would pay for replicas to be exhibited in architectural feats.
London, and would provide the British Museum D) commemorate their worship of the
with a permanent rotating collection of Greek art. goddess Athena.
Greece has tacitly recognized that the exhibits
of classical art in the British Museum are good
115 ambassadors for Greece. Moreover the mission of 2
the world’s greatest museums has changed. No
longer are they storehouses of artifacts. Rather, Which choice provides the best evidence for
museum directors are impresarios who mount the answer to the previous question?
splendid exhibitions that draw thousands of
A) Lines 11-14 (“The tour…democracy”)
120 visitors, and travel from museum to museum. Such
B) Lines 17-20 (“Imperial…that”)
exhibitions are international projects and often
C) Line 96 (“It expresses…people”)
profitable ones. The Greek museums and the
D) Lines 132-134 (“Greece…change”)
British Museum could make good partners.
But there is another reason why the Elgin
125 Marbles case will probably soon be resolved. In the
multicultural world of the future, classical art will 3
matter less. The British Museum’s Duveen Gallery
The primary significance of the French
may someday house exhibits from Africa or
Revolution in the context of the passage is
Central America. As the Elgin Marbles become
the fact that it
130 more the “soul of the Greek people,” they will
become less the cultural heritage of the West.
A) was a source of inspiration for Thomas
Greece will not entirely lose the special place it has
Bruce, the Earl of Elgin.
in the affections of the western world, but attitudes
B) was a major turning point in European
will change.
politics.
1an authoritative order, issued by a Turkish sovereign, having C) ended Louis Fauvel’s career.
the force of law D) may be the reason the Elgin Marbles are
2British poet (1788-1824)
in London.
3the act of returning something to the country of its origin

77 77
SAT Command of Evidence Review

4 8

Which choice provides the best evidence for Which choice provides the best evidence for
the answer to the previous question? the answer to the previous question?
A) Lines 23-24 (“With the…began”) A) Lines 23-24 (“With the…began”)
B) Lines 36-43 (“The French…sculpture”) B) Lines 43-45 (“If all…Louvre”)
C) Lines 45-47 (“But with…job”) C) Lines 85-89 (“He also…weight”)
D) Lines 52-54 (“The person…Hunt”) D) Lines 92-94 (“Byron…heritage”)

5
9
The author’s description of the firman
suggests that It can be surmised from the passage that
Elgin’s party
A) Elgin was craftier than the Turkish
rulers suspected. A) profited handsomely from taking the
B) the Turkish rulers did not intend for Marbles.
Elgin to remove as much of the B) received strong opposition from French
Parthenon as he did. authorities.
C) eighteenth-century Athenian decrees C) were lauded as heroes upon their return
were often unclear. to Britain.
D) the original wording of the firman had D) wanted to take more material than just
been altered. the Marbles.

6
10
Which choice provides the best evidence for Which choice provides the best evidence for
the answer to the previous question? the answer to the previous question?
A) Line 36 (“The collectors…followed”) A) Lines 43-45 (“If all…Louvre”)
B) Lines 47-55 (“The greatest…collections”) B) Lines 56-58 (“Elgin’s…taste”)
C) Lines 65-68 (“But the…figures”) C) Lines 73-76 (“In fact…had”)
D) Lines 79-81 (“It was…fortunate”) D) Lines 82-85 (“Elgin…government”)

7
The author characterizes Lord Byron as

A) a defender of Elgin’s actions.


B) ineffectual in resolving the Elgin
Marbles controversy.
C) an advocate of a point of view that was
adopted by others.
D) the leader of a new European artistic
movement.

78 78
SAT Solving Non-Linear Equations

SAT Math:
Solving Non-Linear
Equations

Answers on Page 299

79 79
SAT Solving Non-Linear Equations: Single Equation Questions

Solving Non-Linear Equations: Single Equation Questions

Identify: A question (before or after translation from a word problem) has one equals
sign and/or inequality or you are solving for the value of an equation.

Set Up: Follow the below steps (where necessary):

1) Group like terms (variables with the same exponent and/or whole numbers).
2) Multiply by a common denominator and/or cross-multiply to remove fractions.
3) Solve for the variable.

1 3

NO CALCULATOR 7x + 3
If 6 = z and z = 4, what is the
(4x2y – 2y2 + 5xy2) – (x2y + xy2 – 5y2) value of x ?

Which of the following is equivalent


to the expression above?
4
A) 3x2y2
B) 6xy2 − 2y2 1 2
If 6x + 3y = 8, what is the value of
C) 3x2y + 4xy2
D) 3x2y + 4xy2 + 3y2 2x + 8y ?

The number of cars that a company


expects to sell next year, Cnext year , can 5
be estimated by the number of cars sold
this year, Cthis year , by the equation NO CALCULATOR
below.
Cthis year
If y > 5, which of the following is
Cnext year = Cthis year + 0.5(Cthis year )(1 – ) 1
F equivalent to 1 ?
The constant F in this formula is the + 1
y+4 y+5
number of cars the company’s factory is
able to produce. If the company sold y2 + 9y + 20
420 cars this year and F = 630, how A) 2y + 9
many cars will the company sell two 2y + 9
years from now? (Round your answer to B) y2 + 9y + 20
the nearest whole number.)
C) y2 + 9y + 20

D) 2y + 9

80 80
SAT Solving Non-Linear Equations: Systems of Equations Questions

Solving Non-Linear Equations: Systems of Equations Questions

Identify: There are two or more equals signs and/or inequalities.

Set Up: Follow the below steps (where necessary):

1) Group like terms (variables with the same exponent and/or whole numbers).
2) Add or subtract given equations (if possible).
3) If you cannot add or subtract, then substitute between equations and solve.

6 8

NO CALCULATOR NO CALCULATOR

s = 3.85 + 0.55x Which ordered pair (x, y) satisfies the


i = 1.45 + 0.95x system of equations shown below?

In the equations above, s and i 4x – 2y = 12


represent the cost, in dollars, of 3x + 6y = –6
purchasing a snow shovel and ice
scraper, respectively, x weeks after A) (–6, 2)
November 1 during last winter. What
B) (–2, 2)
was the price of a snow shovel when
it was equal to the price of an ice C) (2, –2)
scraper? (Ignore the dollar sign in D) (4, 2)
your answer.)
9

y=8
y = ax2 + b
7
In the system of equations above, a
NO CALCULATOR and b are constants. Which of the
following represents a range of
3y 4x
If 2x = 6, what is the value of y ? possible real solutions of a and b ?

A) a > 0 and b > 8


B) a > 0 and b < 8
C) a < 0 and b > 8
D) a < 0 and b < 8

81 81
SAT Solving Non-Linear Equations: Rewriting Equations

Solving Non-Linear Equations: Rewriting Equation Questions

Identify: A question that relates to manipulating equations to create new


expressions. These questions will often ask for one variable “in terms of” another.

Set Up: Use algebraic operations and/or substitution to convert the provided
expression into the desired equation. The phrase “in terms of” translates to “solve
for the first variable.” Thus, if a question asks to “solve a in terms of b,” you should solve
the equation for a. The b variable will appear on the right side of the equation.

10 12

NO CALCULATOR NO CALCULATOR

6 1 qq
If = 3g where g ≠ 0 and f ≠ –2, F = k r12 2
2f + 1
what is g in terms of f ? Coulomb’s Law: At a distance, r, the
electrostatic force, F, experienced by
two objects, q1 and q2, can be
A) g = 2f + 1 measured by the above formula,
2 where k is Coulomb’s Law Constant.
B) g =
2f + 1
1 Which of the following equations
C) g = 2 – expresses the square of the distance
f
1 that separates two objects in terms of
D) g = 2 f + 2 the total electrostatic force,
Coulomb’s Law Constant, and the
individual charges of the two objects.
11
1
If x = a2 , where a > 0 and x > 0, Fk
A) r2 = q q
which of the following equations 1 2

gives a in terms of x ? q1q2


B) r2 =
Fk
1 q1q2
A) a = C) r2 =
x kF
1 kq1q2
B) a = x D) r2 =
F
C) a = x

D) a = x4

82 82
SAT Solving Non-Linear Equations Review

1 5

If 9 + 4x = 61, then 3x = ? a2 + b2 = 160


2a = 6b
A) 13
B) 24 If (a, b) is a solution to the system of
C) 39
D) 53 equations above, and b > 0, what is the
b
value of ?
2
2

NO CALCULATOR

If a = 4x2 – 2x, and b = x2 + x + 3, what is


a – 3b in terms of x ? Questions 6 and 7 refer to the following
information.
A) x2 – 5x – 9
B) x2 + 2x + 3 Pythagorean expectation is a formula used
C) 2x2 – 3x + 4 to estimate how many games a baseball
D) 2x2 + 5x + 8 team "should" win based on the number
of runs it scored and allowed during a
season. The Pythagorean expectation
formula for a 162-game season is below:
3 runs scored2
Wins = 162( )
runs scored2 + runs allowed2
If y > 0, what is the solution to the equation
6
3 2
+ =1? According the formula, how many games
y+1 y–1
was a team projected to win in 2013 if it
scored 786 runs and allowed 679 runs?
(Round to the nearest whole number.)

7
4
In 2014 (the following season), the same
team wanted to increase its projected win
NO CALCULATOR
total from 2013 by 10 games (use your
rounded answer from question 6 as the
3b2 + 13 – x = 8 projected win total for 2013). If the team
allowed the same number of runs in 2014 as
If b > 0 and x = 3 in the equation above, it did in 2013, how many additional runs
what is the value of b ? (compared to 2013) would it have to had
scored in 2014 to reach its desired projected
win total? (Round to the nearest whole
number.)

83 83
84 84
SAT Wordiness and Redundancy

SAT Writing:
Wordiness and
Redundancy

Answers on Page 296

85 85
SAT Wordiness and Redundancy: Wordiness

Wordiness and Redundancy: Wordiness

Wordiness occurs when an underlined phrase uses more words than it needs.

Identifying Wordiness on the SAT: Look for the use of passive voice, the presence of
several two or three letter words, or the inclusion of an unnecessary ‘–ing’ verb.

On the SAT, if you are stuck between two or more choices, pick the answer that
has the fewest number of words.

1 3

Marcus Garvey was born in Jamaica The local football team has ardent
in 1887 and went to be in college in fans. These people show up hours
London just before World War I. before the games start and cheer
loudly from kickoff until the final
A) NO CHANGE whistle.
B) would have been in
C) attended A) NO CHANGE
D) went to attend B) fans, who go to the stadium every
week.
C) fans, who greatly love the team.
D) fans—lovers of the team.
2

Tyler enjoyed reading articles about


the economy, the reason being that
he enjoyed being an informed
participant in intellectual debates.

A) NO CHANGE
B) as a result of the reason
C) because
D) DELETE the underlined portion.

86 86
SAT Wordiness and Redundancy: Redundancy

Wordiness and Redundancy: Redundancy

Redundancy occurs when an underlined word or phrase repeats an idea that is


already in the sentence.

Identifying Redundancy on the SAT: Look for common redundancies.

Common redundancies:

• 6:30 p.m. at night


• And also
• Close proximity
• Manually by hand
• Repeat again

4 6

During the camping trip, Tom On June 6th, 1944, over 100,000
remarked that the night sky was members of the Allied forces landed
more clearer than he’d ever seen it in on beaches in Northern France.
his life. Because of the bravery of the
soldiers’ courageous actions, the
A) NO CHANGE Allied army was able to gain a
B) clearer foothold in Europe.
C) as clear
D) DELETE the underlined portion. A) NO CHANGE
B) the brave efforts of so many
soldiers,
C) the brave and courageous actions
of the soldiers,
5
D) the soldiers’ courageous actions,
To monitor the inclement weather,
the school principal requested hourly
weather updates from his assistant
every 60 minutes.

A) NO CHANGE
B) at the top of the hour
C) as clear
D) DELETE the underlined portion.

87 87
SAT Wordiness and Redundancy Review

1 4
Environmental managers say that fire is a I would be starting college in a month and
part of maintaining the proper balance of decided that, instead of living in the stark
underbrush and new-growth trees. and unimaginative dorms, I would live in a
Moreover, they argue that smaller, historic neighborhood within walking
“natural” fires may prevent uncontrolled, distance of my school.
larger blazes. To some, this is just a
rhetorical debate. But to those of us who A) NO CHANGE
live with wildfires, the choice can come B) neighborhood for walking to school.
down to life or death. C) neighborhood, where walking to school
would be convenient.
A) NO CHANGE D) neighborhood by my school that I could
B) us living or dying. walk to.
C) whether we live or die.
D) us being alive afterwards, or not.

2 Emily let her boss know that she needed to


leave work by 4:00 p.m. in the afternoon:
When one freeholder (an individual who she had a dentist appointment that she
owned a piece of real estate) wished and simply could not miss.
desired to sell his land to another
freeholder, there was a good chance that A) NO CHANGE
both of them would be unable to read or B) 4:00 p.m.
write. C) 4:00 p.m. during the evening
D) DELETE the underlined portion
A) NO CHANGE
B) had the hope and desire
C) wished
D) was hopeful and desiring

A surveyor’s job might involve analyzing


the boundaries of private property in order
to resolve disputes. If the study of an
individual’s property indicates that it
covers more land than it was granted
when the deed was created, the
government might recommend mediation
to determine fair compensation.

A) NO CHANGE
B) more land than the property covers in
the present
C) more land than the deed had originally
granted
D) more land than granted by the deed

88 88
SAT The Essay

SAT: The Essay

89 89
SAT SAT Essay Introduction

The “Optional” SAT Essay

Some colleges require the SAT essay. Other colleges do not. Most colleges will accept
scores from the SAT essay even if they do not require it.

You should decide whether to take the SAT essay based on the requirements of the
colleges to which you’re thinking about applying. Colleges’ websites will tell you
whether or not they require the SAT essay.

The Bad News About the SAT Essay

You only have 50 minutes to read and respond to a given passage.

The Good News About the SAT Essay

You only have 50 minutes.

You are not expected to write a masterpiece.

Each grader only spends a few minutes on your essay.

The grade is based on your ability to evaluate how an author makes an argument.

The SAT Essay Prompt

The SAT essay prompt will always relate to a controversial issue, and it will
provide you with a persuasive argument taking one side of that issue. An
example of a prompt is below:

Write an essay in which you explain how Bryan builds an argument to


persuade his audience that the United States should no longer adopt
the gold standard. In your essay, analyze how Bryan uses one or more
of the features listed in the box above (or features of your own choice)
to strengthen the logic and persuasiveness of his argument. Be sure
that your analysis focuses on the most relevant features of the passage.

Your essay should not explain whether you agree with Bryan’s claims,
but rather explain how Bryan builds an argument to persuade his
audience.

90 90
SAT SAT Essay Scoring

SAT Essay Scoring

Students receive a score on a 6-24 scale, which is based upon subscores in three
domains. The SAT essay score does not affect the 400-1600 SAT composite score.

Each of the three domains is scored on a 2-8 scale (two graders give each domain a score
from 1 to 4). The maximum score that a student can receive is 24.

The domains on the SAT essay are:

Reading

Understanding of central ideas, important details, and their interrelationship; accuracy


in representation of the source; use of textual evidence to demonstrate understanding.

Analysis

Evaluation of the author’s use of evidence, reasoning, and/or stylistic and persuasive
elements; support for claims or points made in the response; focus on features of the text
most relevant to addressing the purpose.

Writing

Use of a central thesis; use of effective organization and progression of ideas; use of
varied sentence structures; use of consistent style and tone; command of the conventions
of standard written English.

91 91
SAT Outlining Your SAT Essay

Outlining Your SAT Essay

Use the following steps to outline your essay:

1. Read the essay prompt – The prompt is the same for each essay. The provided text will
change.

2. Read and notate the provided text – Analyze the text for the author’s thesis and how
the author supports that thesis through the use of argumentative techniques.

3. Write an I-B-C - This is your brief outline. It only needs to make sense to you.

I: Introduction – What is the author’s thesis? What are the major argumentative
techniques used to support the thesis?

B: Body – This is the meat of your essay. Jot down two or three argumentative techniques
and how the author used those techniques to support the thesis.

C: Conclusion – A BRIEF wrap-up. Jot down a phrase that ties everything together

4. Write Your Essay – Let your I-B-C notes be your guide. Follow traditional essay structure.

92 92
SAT Argumentative Techniques: Ethos

Argumentative Techniques

There are several categories of argumentative techniques that authors can use on the SAT
essay’s provided text.

• Ethos (the use of the credibility and character of the author)


• Logos (the use of reason)
• Pathos (the use of emotion)
• Figurative Language (the use of figures of speech)

Argumentative Techniques: Ethos

Ethos, or ethical appeal, means convincing by the character and credibility of the
author to support the validity of a thesis.

A writer can demonstrate expertise or establish reputation in several ways:

• Through extensive research.


• Through up-to-date research.
• Through referencing recognized authorities in the field.

An ethical appeal is often indicated by the use of 1st person writing.

Ethos Example

Dolly seems a very ordinary sheep – just an amiable Finn-Dorset ewe – yet as all the
world has acknowledged, if not entirely for the right reasons, she might reasonably claim to
be the most extraordinary creature ever to be born. Mammals are normally produced by the
sexual route: an egg joins with a sperm to form a new embryo. But in 1996 Keith
Campbell and I, with our colleagues at Roslin Institute and PPL, cloned Dolly from a
cell that had been taken from the mammary gland of an old ewe and then grown in culture.
We fused that cultured cell with an egg from yet another ewe to “reconstruct” an embryo
that we transferred into the womb of a surrogate mother, where it developed to become a
lamb. This was the lamb we called Dolly: not quite the first mammal ever to be cloned,
but certainly the first to be cloned from an adult body cell. Her birth overturns one of the
deepest dogmas in all of biology, for until the moment in February 1997 when we made her
existence known through a brief letter in the scientific journal Nature, most scientists
simply did not believe that cloning in such a way, and from such a cell, was possible.

93 93
SAT Argumentative Techniques: Logos

Argumentative Techniques: Logos

Logos, or logical appeal, means convincing by the use of reason to support the
validity of a thesis.

A writer can make a logical appeal in several ways:

• Through the use of cause and effect.


• Through the use of facts and evidence.
• Through the use of examples and precedents (specific examples from the
past).
• Through the use deductive and inductive reasoning.
o Deductive reasoning – To take a general idea and apply it to a specific
case.
 All boys wear the color red. John is a boy. Therefore, John wears
red.
o Inductive reasoning – To derive a general rule from specific cases.
 John, Steve, and Bob all wear red. Therefore, all boys wear red.

Logos Example

Over the long span of history from the founding of the nation in 1776 until
1965, immigration varied widely but averaged around 230,000 a year. This was a
phenomenal flow into a single country, unmatched in world history. It should be noted that
during large parts of that period, the United States – with vast expanses of virtually open
land – was much better able than today to handle 230,000 newcomers annually. Suddenly
in the 1970s and 1980s, at the very time that the majority of Americans were
coming to the conclusion that the U.S. population had grown large enough, immigration
soared above American tradition, averaging more than 500,000 a year. And it ran
around 1 million a year during the 1990s.

94 94
SAT Argumentative Techniques: Pathos

Argumentative Techniques: Pathos

Pathos, or emotional appeal, means convincing by the use of emotion to support the
validity of a thesis.

A writer can use an emotional appeal in several ways:

• Through language choice.


• The use of strong diction.
• The use of juxtaposition (two things placed close together with
contrasting effect).
 “It was the best of times, it was the worst of times, it was the age of
wisdom, it was the age of foolishness.”
• The use of repetition.
• The use of connotation (something suggested or implied by a word or
thing).
 Connotation is a particularly powerful tool in delivering a pathos
argument.

Pathos Example

Fewer adolescences engaging in the study of the maths and sciences is a threat to
society’s future well-being. This is because these pursuits are a foundation of widening
knowledge and improving the quality of life for all of mankind. As more teenagers
around the world shift their interests to the liberal arts, we threaten the emergence of a
less innovative and less productive society, where we cannot guarantee that the
lives of our children will be better than our own.

95 95
SAT Argumentative Techniques: Figurative Language

Argumentative Techniques: Figurative Language

Figurative language is the use of words in figures of speech to support the validity
of a thesis.

A writer can use figurative language in several ways:

• The use of alliteration (repetition of the same initial letter or sound in a group
of words).
• The use of hyperbole (dramatic exaggeration).
• The use of imagery (vivid and descriptive language).
• The use of metaphor (stating a fact through the use of comparison).
• The use of simile (the use of “like” or “as” to compare one object to another)
• The use of personification (human characteristics being given to an animal or
object).

Figurative Language Example

Between us there was, as I have already said somewhere, the bond of the sea. Besides
holding our hearts together through long periods of separation, it had the effect of
making us tolerant of each other’s stories—and even convictions. The Lawyer—the best of
old fellows—had, because of his many years and many virtues, the only cushion on deck,
and was lying on the only rug. The Accountant had brought out already a box of dominoes,
and was toying architecturally with the pieces. Marlow sat cross-legged, leaning against
the mast. He had sunken cheeks, a yellow complexion, a straight back, and, with
his arms dropped, the palms of his hands outwards, resembled an idol. The
Director, satisfied the anchor had good hold, made his way forward and sat down amongst
us. We exchanged a few words lazily. Afterwards there was silence on board the yacht. For
some reason or another we did not begin that game of dominoes. We felt meditative, and fit
for nothing but placid staring.

96 96
SAT Argumentative Techniques: Sample Paragraph

Persuasive Argument: Sample Paragraph

Forget soaring rates of obesity,


bankruptcy, and out-of-wedlock births: by
far the scariest statistic about
contemporary American life is that people
now spend more on Halloween than on any
other “holiday” apart from Christmas. It’s
not that I oppose the wearing of charming
costumes by young children. Nor do I
question the rightness of shoving barrels
of saltwater taffy down young throats.
What gets my sacrificial goat is that
Halloween isn’t about anything. It’s not
about death, or life, or fall, or family, or
patriotic love of country. It is a completely
content-free, sugar-frosted, dark-caped
bacchanal in a society that already, every
day, gives kids license to be content-free,
sugar-frosted, and to dress however
gothically they like.

97 97
SAT Argumentative Techniques: Sample Paragraph

Persuasive Argument: Sample Paragraph

Forget soaring rates of obesity, Pathos: Using fear to


bankruptcy, and out-of-wedlock further the argument.
births: by far the scariest statistic about
contemporary American life is that people
now spend more on Halloween than Logos: Using data to
on any other “holiday” apart from further the argument.
Christmas. It’s not that I oppose the
wearing of charming costumes by young
children. Nor do I question the rightness of
shoving barrels of saltwater taffy down
young throats. What gets my sacrificial Figurative language:
Personification
goat is that Halloween isn’t about
anything. It’s not about death, or life, or
fall, or family, or patriotic love of country.
It is a completely content-free, sugar- Figurative language:
frosted, dark-caped bacchanal in a Imagery
society that already, every day, gives kids
license to be content-free, sugar-frosted,
and to dress however gothically they like.

98 98
SAT SAT Essay Example

SAT Essay Example


50 Minutes——1 Prompt

As you read the passage below, consider how Bryan uses

• evidence, such as facts or examples, to support claims.


• reasoning to develop ideas and to connect claims and evidence.
• stylistic or persuasive elements, such as word choice or appeals to emotion,
to add power to the ideas expressed.

Adapted from former US Representative William Jennings Bryan, “Cross of


Gold.” Originally delivered on July 9, 1896.

I want to suggest this truth, that if the gold standard is a good thing we ought to
declare in favor of its retention and not in favor of abandoning it; and if the gold
standard is a bad thing, why should we wait until some other nations are willing to
help us to let it go?

Here is the line of battle. We care not upon which issue they force the fight. We are
prepared to meet them on either issue or on both. If they tell us that the gold standard
is the standard of civilization, we reply to them that this, the most enlightened of all
nations of the earth, has never declared for a gold standard, and both the parties this
year are declaring against it. If the gold standard is the standard of civilization, why,
my friends, should we not have it? So if they come to meet us on that, we can present
the history of our nation. More than that, we can tell them this, that they will search
the pages of history in vain to find a single instance in which the common people of
any land ever declared themselves in favor of a gold standard. They can find where
the holders of fixed investments have.

Mr. Carlisle said in 1878 that this was a struggle between the idle holders of idle
capital and the struggling masses who produce the wealth and pay the taxes of the
country; and my friends, it is simply a question that we shall decide upon which side
shall the Democratic Party fight. Upon the side of the idle holders of idle capital, or
upon the side of the struggling masses? That is the question that the party must
answer first; and then it must be answered by each individual hereafter. The
sympathies of the Democratic Party, as described by the platform, are on the side of
the struggling masses, who have ever been the foundation of the Democratic Party.
There are two ideas of government. There are those who believe that if you just
legislate to make the well-to-do prosperous, that their prosperity will leak through on
those below. The Democratic idea has been that if you legislate to make the masses
prosperous their prosperity will find its way up and through every class that rests
upon it.

99 99
SAT SAT Essay Example

SAT Essay Example


50 Minutes——1 Prompt
You come to us and tell us that the great cities are in favor of the gold standard. I tell
you that the great cities rest upon these broad and fertile prairies. Burn down your
cities and leave our farms, and your cities will spring up again as if by magic. But
destroy our farms and the grass will grow in the streets of every city in the country.
My friends, we shall declare that this nation is able to legislate for its own people on
every question without waiting for the aid or consent of any other nation on earth, and
upon that issue we expect to carry every single state in the Union.

I shall not slander the fair state of Massachusetts nor the state of New York by saying
that when citizens are confronted with the proposition, “Is this nation able to attend to
its own business?”—I will not slander either one by saying that the people of those
states will declare our helpless impotency as a nation to attend to our own business. It
is the issue of 1776 over again. Our ancestors, when but 3 million, had the courage to
declare their political independence of every other nation upon earth. Shall we, their
descendants, when we have grown to 70 million, declare that we are less independent
than our forefathers? No, my friends, it will never be the judgment of this people.
Therefore, we care not upon what lines the battle is fought. If they say bimetallism is
good but we cannot have it till some nation helps us, we reply that, instead of having a
gold standard because England has, we shall restore bimetallism, and then let
England have bimetallism because the United States have.

If they dare to come out in the open field and defend the gold standard as a good thing,
we shall fight them to the uttermost, having behind us the producing masses of the
nation and the world. Having behind us the commercial interests and the laboring
interests and all the toiling masses, we shall answer their demands for a gold
standard by saying to them, you shall not press down upon the brow of labor this
crown of thorns. You shall not crucify mankind upon a cross of gold.

Write an essay in which you explain how Bryan builds an argument to


persuade his audience that the United States should no longer adopt
the gold standard. In your essay, analyze how Bryan uses one or more
of the features listed in the box above (or features of your own choice)
to strengthen the logic and persuasiveness of his argument. Be sure
that your analysis focuses on the most relevant features of the passage.

Your essay should not explain whether you agree with Bryan’s claims,
but rather explain how Bryan builds an argument to persuade his
audience.

100 100
SAT SAT Essay Brainstorming

Essay Brainstorming

As you read the passage below, consider how Bryan uses

• evidence, such as facts or examples, to support claims.


• reasoning to develop ideas and to connect claims and evidence.
• stylistic or persuasive elements, such as word choice or appeals to emotion,
to add power to the ideas expressed.

Which of the below argumentative techniques does the author use to further his essay?
How are the techniques used in advancing his thesis?

Ethos Logos

Pathos Figurative Language

101 101
SAT SAT Essay I-B-C

Creating an I-B-C and Writing Your Essay

Use your brainstorming to assist in writing your essay. Create an I-B-C outline to organize
your thoughts.

I-B-C: Introduction, Body, Conclusion

Introduction

• What is the author’s THESIS?


• Brief summary of the ARGUMENTATIVE TECHIIQUES used in the essay.

Body Paragraph (Two or three paragraphs)

• A clear TOPIC SENTENCE to begin each paragraph. In the topic sentence,


connect an argumentative technique used in the essay to the author’s THESIS.
• How does the use of this argumentative technique SUPPORT THE AUTHOR’S
THESIS?
• A CLOSING SENTENCE which ties the paragraph together.
• Use SEPARATE PARAGRAPHS for different examples.

Conclusion

• A SUMMARY of the THESIS statement.


• A CLOSING THOUGHT that ties everything together.

102 102
SAT SAT Essay Review #1

SAT Essay Review


50 Minutes——1 Prompt

As you read the passage below, consider how Henry uses

• evidence, such as facts or examples, to support claims.


• reasoning to develop ideas and to connect claims and evidence.
• stylistic or persuasive elements, such as word choice or appeals to emotion,
to add power to the ideas expressed.

Adapted from Patrick Henry’s Speech to the Virginia Convention. Originally


delivered on March 23, 1775.

MR. PRESIDENT: No man thinks more highly than I do of the patriotism, as well as
abilities, of the very worthy gentlemen who have just addressed the House. But
different men often see the same subject in different lights; and, therefore, I hope it
will not be thought disrespectful to those gentlemen if, entertaining as I do, opinions
of a character very opposite to theirs, I shall speak forth my sentiments freely, and
without reserve.

Mr. President, it is natural to man to indulge in the illusions of hope. We are apt to
shut our eyes against a painful truth, and listen to the song of that siren till she
transforms us into beasts. Is this the part of wise men, engaged in a great and arduous
struggle for liberty? Are we disposed to be of the number of those who, having eyes,
see not, and, having ears, hear not, the things which so nearly concern their temporal
salvation? For my part, whatever anguish of spirit it may cost, I am willing to know
the whole truth; to know the worst, and to provide for it.

I have but one lamp by which my feet are guided; and that is the lamp of experience. I
know of no way of judging of the future but by the past. And judging by the past, I
wish to know what there has been in the conduct of the British ministry for the last
ten years, to justify those hopes with which gentlemen have been pleased to solace
themselves, and the House? Is it that insidious smile with which our petition has been
lately received? Trust it not, sir; it will prove a snare to your feet. Suffer not
yourselves to be betrayed with a kiss. Ask yourselves how this gracious reception of
our petition comports with these war-like preparations which cover our waters and
darken our land. Are fleets and armies necessary to a work of love and reconciliation?
Have we shown ourselves so unwilling to be reconciled, that force must be called in to
win back our love? Let us not deceive ourselves, sir. These are the implements of war
and subjugation; the last arguments to which kings resort. I ask, gentlemen, sir, what
means this martial array, if its purpose be not to force us to submission? Can

103 103
SAT SAT Essay Review #1

SAT Essay Review


50 Minutes——1 Prompt
gentlemen assign any other possible motive for it? Has Great Britain any enemy, in
this quarter of the world, to call for all this accumulation of navies and armies?

They tell us, sir, that we are weak; unable to cope with so formidable an adversary.
But when shall we be stronger? Will it be the next week, or the next year? Will it be
when we are totally disarmed, and when a British guard shall be stationed in every
house? Shall we gather strength by irresolution and inaction? Shall we acquire the
means of effectual resistance, by lying supinely on our backs, and hugging the
delusive phantom of hope, until our enemies shall have bound us hand and foot? Sir,
we are not weak if we make a proper use of those means which the God of nature hath
placed in our power. Three millions of people, armed in the holy cause of liberty, and
in such a country as that which we possess, are invincible by any force which our
enemy can send against us. Besides, sir, we shall not fight our battles alone. There is a
just God who presides over the destinies of nations; and who will raise up friends to
fight our battles for us. The battle, sir, is not to the strong alone; it is to the vigilant,
the active, the brave. Besides, sir, we have no election. If we were base enough to
desire it, it is now too late to retire from the contest. There is no retreat but in
submission and slavery! Our chains are forged! Their clanking may be heard on the
plains of Boston! The war is inevitable and let it come! I repeat it, sir, let it come.

It is in vain, sir, to extenuate the matter. Gentlemen may cry, Peace, Peace but there
is no peace. The war is actually begun! The next gale that sweeps from the north will
bring to our ears the clash of resounding arms! Our brethren are already in the field!
Why stand we here idle? What is it that gentlemen wish? What would they have? Is
life so dear, or peace so sweet, as to be purchased at the price of chains and slavery?
Forbid it, Almighty God! I know not what course others may take; but as for me, give
me liberty or give me death!

Write an essay in which you explain how Henry builds an argument to


persuade his audience that the American colonies should wage military
action against Britain. In your essay, analyze how Henry uses one or
more of the features listed in the box above (or features of your own
choice) to strengthen the logic and persuasiveness of his argument. Be
sure that your analysis focuses on the most relevant features of the
passage.

Your essay should not explain whether you agree with Henry’s claims,
but rather explain how Henry builds an argument to persuade his
audience.

104 104
Student
Name:

SAT PRACTICE ESSAY ANSWER SHEET

ESSAY PAGE 1 ESSAY PAGE 1 ESSAY PAGE 1

105
Student
Name:

SAT PRACTICE ESSAY ANSWER SHEET

ESSAY PAGE 2 ESSAY PAGE 2 ESSAY PAGE 2

106
SAT SAT Essay Review #2

SAT Essay Review


50 Minutes——1 Prompt

As you read the passage below, consider how Chisholm uses

• evidence, such as facts or examples, to support claims.


• reasoning to develop ideas and to connect claims and evidence.
• stylistic or persuasive elements, such as word choice or appeals to emotion,
to add power to the ideas expressed.

Adapted from former US Representative Shirley Anita St. Hill Chisholm,


“For the Equal Rights Amendment.” Originally delivered on August 10, 1970.

When a young woman graduates from college and starts looking for a job, she is likely
to have a frustrating and even demeaning experience ahead of her. If she walks into
an office for an interview, the first question she will be asked is, "Do you type?''

There is a calculated system of prejudice that lies unspoken behind that question.
Why is it acceptable for women to be secretaries, librarians, and teachers, but totally
unacceptable for them to be managers, administrators, doctors, lawyers, and Members
of Congress? The unspoken assumption is that women are different. They do not have
executive ability orderly minds, stability, leadership skills, and they are too
emotional.

It has been observed before, that society for a long time discriminated against another
minority, the blacks, on the same basis - that they were different and inferior.

As a black person, I am no stranger to race prejudice. But the truth is that in the
political world I have been far oftener discriminated against because I am a woman
than because I am black.

Prejudice against blacks is becoming unacceptable although it will take years to


eliminate it. But it is doomed because, slowly, white America is beginning to admit
that it exists. Prejudice against women is still acceptable. There is very little
understanding yet of the immorality involved in double pay scales and the
classification of most of the better jobs as "for men only."

More than half of the population of the United States is female. But women occupy
only 2 percent of the managerial positions. They have not even reached the level of
tokenism yet. No women sit on the AFL-CIO council or Supreme Court There have

107 107
SAT SAT Essay Review #2

SAT Essay Review


50 Minutes——1 Prompt
been only two women who have held Cabinet rank, and at present there are none. In
Congress, we are down to one Senator and 10 Representatives.

Considering that there are about 3 1/2 million more women in the United States than
men, this situation is outrageous.

It is true that part of the problem has been that women have not been aggressive in
demanding their rights. This was also true of the black population for many years.
They submitted to oppression and even cooperated with it. Women have done the
same thing. But now there is an awareness of this situation particularly among the
younger segment of the population. As in the field of equal rights for blacks, Spanish-
Americans, the Indians, and other groups, laws will not change such deep-seated
problems overnight, but they can be used to provide protection for those who are most
abused, and to begin the process of evolutionary change by compelling the insensitive
majority to reexamine its unconscious attitudes.

It is for this reason that I wish to introduce today a proposal that has been before
every Congress for the last 40 years and that sooner or later must become part of the
basic law of the land -- the equal rights amendment.

Let me note and try to refute two of the commonest arguments that are offered against
this amendment. One is that women are already protected under the law and do not
need legislation. Existing laws are not adequate to secure equal rights for women.
Sufficient proof of this is the concentration of women in lower paying, menial,
unrewarding jobs and their incredible scarcity in the upper level jobs. If women are
already equal, why is it such an event whenever one happens to be elected to
Congress?

It is obvious that discrimination exists. Women do not have the opportunities that
men do. And women that do not conform to the system, who try to break with the
accepted patterns, are stigmatized as ''odd'' and "unfeminine."

A second argument often heard against the equal rights amendment is that it would
eliminate legislation that many States and the Federal Government have enacted
giving special protection to women and that it would throw the marriage and divorce
laws into chaos.

As for the marriage laws, they are due for a sweeping reform, and an excellent
beginning would be to wipe the existing ones off the books. Regarding special

108 108
SAT SAT Essay Review #2

SAT Essay Review


50 Minutes——1 Prompt
protection for working women, I cannot understand why it should be needed. Women
need no protection that men do not need. What we need are laws to protect working
people, to guarantee them fair pay, safe working conditions, protection against
sickness and layoffs, and provision for dignified, comfortable retirement. Men and
women need these things equally.

Write an essay in which you explain how Chisholm builds an argument


to persuade her audience that the United States should adopt the
Equal Rights Amendment. In your essay, analyze how Chisholm uses
one or more of the features listed in the box above (or features of your
own choice) to strengthen the logic and persuasiveness of her
argument. Be sure that your analysis focuses on the most relevant
features of the passage.

Your essay should not explain whether you agree with Chisholm’s
claims, but rather explain how Chisholm builds an argument to
persuade her audience.

109 109
Student
Name:

SAT PRACTICE ESSAY ANSWER SHEET

ESSAY PAGE 1 ESSAY PAGE 1 ESSAY PAGE 1

110
Student
Name:

SAT PRACTICE ESSAY ANSWER SHEET

ESSAY PAGE 2 ESSAY PAGE 2 ESSAY PAGE 2

111
112 112
SAT Transitions

SAT Writing:
Transitions

Answers on Page 296

113 113
SAT Transitions: Conjunctions

Transitions: Conjunctions

A transition is a word or phrase that connects or joins together words, phrases,


clauses, or sentences.

Conjunctions are the most common type of transition. Conjunctions can indicate agreement
(cause and effect), disagreement (contrast), or the order in which events took place.

Identifying Transition (Conjunction) Issues on the SAT: Look for an agreeing


conjunction joining two disagreeing words, phrases, clauses, or sentences, or a disagreeing
conjunction joining two agreeing words, phrases, clauses, or sentences.

1 3

Landslides can follow heavy rains as Adam Smith wrote, “It is not from the
hillsides are stripped of trees and benevolence of the butcher, the
shrubs. These landslides can pollute brewer, or the baker, that we can
water sources and result in the loss of expect our dinner, but from their
the topsoil necessary to support new regard to their own interest.” On the
growth. Instead, stripped hillsides are other hand, the butcher does not cut
extremely unstable. This can also pose meat because the community desires
a threat to homes in the area. it, but rather because he can earn
money by doing it.
A) NO CHANGE
B) Despite this A) NO CHANGE
C) Nevertheless B) In spite of this
D) Moreover C) In other words
D) Despite this
2

Getting started at the game requires


no specialized expertise: all a person
needs is an internet connection and a
few minutes to create a character.
Becoming an expert (there are
thousands of enhancements to learn
and hundreds of territories to
explore), moreover, requires a larger
commitment.

A) NO CHANGE
B) therefore,
C) however,
D) in short,

114 114
SAT Transitions: Main ideas

Transitions: Main Ideas

A transition is a word or phrase that connects or joins together words, phrases,


clauses, or sentences.

When a transition links sentences within a paragraph, it is essential that the transition
logically connects the main idea of one sentence to the main idea the of sentence before
and/or after it.

Identifying Transition (Main Ideas) Issues on the SAT: Look for a transition that
links two sentences within a paragraph. Does the transition logically connect the idea(s)
within each sentence?

4 6

With the recent reduction of office The internet has billions of webpages
staff, the finance team has been asked which, in total, contain more
to take on additional duties. Working information than all of the world’s
for the company, the accountants are libraries combined. Understanding
now responsible for mailing packages the need to quickly navigate all of
and restocking office supplies. these options, Larry Page and Sergey
Brin founded Google in 1998.
A) NO CHANGE
B) Likewise, A) NO CHANGE
C) For these reasons, B) While studying for their Master’s
D) In addition to creating financial degrees at Stanford,
reports, C) Looking towards the future,
D) DELETE the underlined portion.
5

While television and the internet


provide entertainment options, they
are no match for the experience of
watching a new movie in a theater.
During a rainy weekend or a cold
winter day, the unique sensation of
seeing a film with hundreds of people
is well worth the money.

A) NO CHANGE
B) Especially popular during the
summer,
C) Although it is more expensive than
other entertainment options,
D) During periods of economic growth,

115 115
SAT Transitions Review

1 4

Although computers are used to store People with red hair have contributed a
confidential data of a political, social, or tremendous amount to society. It is thus
economic nature, our dependence on disconcerting that red hair is sometimes
computer technology may signal new risks held in low esteem. There are estimates
to match new possibilities. that red hair will be extinct by 2060.

A) NO CHANGE Which choice most effectively sets up the


B) Because information that follows?
C) Despite the fact that
D) It is true that A) NO CHANGE
B) taken for granted.
C) widely appreciated.
D) at risk of disappearance.
2

The average daytime temperature for


Death Valley hovers around 125°F, though
it has been recorded as high as 134°F. It’s
easy to see how the place earned its name;
temperatures like these make for difficult
travel. Instead, most of the valley’s
winding roads were built in the 1930s and
cannot support high speeds.

A) NO CHANGE
B) Moreover
C) Despite this
D) On the contrary

Some ancient cultures held the belief that


the gods controlled the success of the
yearly harvest. These societies thus placed
a high priority on appeasing the wishes of
their deities. However, it is not surprising
that within these cultures animal
sacrifices and other religious ceremonies
were common during the planting season.

A) NO CHANGE
B) Given that,
C) By contrast,
D) Thereafter,

116 116
SAT Solving Linear Equations

SAT Math:
Solving Linear
Equations

Answers on Page 300

117 117
SAT Solving Linear Equations: Slope-Intercept Form

y
Solving Linear Equations: Slope-
run
Intercept Form (x2, y2)

rise
(x1, y1) slope (m)
Identify: The question will give you at least y-intercept (b)
one point, a slope, and/or a y- or x-
intercept of a line. x
(0 , b)

Set Up: Write down the slope-intercept


form and use it to solve the question. Parallel
lines have the same slope. Perpendicular lines Slope-Intercept Form
have slopes which are negative reciprocals.
y = mx + b
m = slope
b = y-intercept

1 3

What is the x-intercept of the line NO CALCULATOR


that contains the points (–4, 1) and
(–7, 2) in the standard (x, y) Which of the following regions is
coordinate plane? described by the inequalities y < –x + 2
and y > x – 5 ?
A) (–1, 0)
B) (0, –1)
C) (0, 1)
D) (1, 0)

The graph of a line in the xy-plane


has slope 2 and contains the point
(4, 6). The graph of a second line
passes through the points (–1, 1) and A) A
(–5, 9). If the two lines intersect at B) B
the point (a, b), what is the value of C) C
a+b? D) D

A) −1.5
B) −1.25
C) 0.5
D) 0.75

118 118
SAT Solving Linear Equations: Slope

Solving Linear Equations: Slope-Intercept Form Application

Identify: The question asks for an interpretation of an equation in slope-intercept form (y =


mx + b).

Set Up: The slope of a line (m) represents average (arithmetic mean) rate of change.
The y-intercept (b) represents the value of y when x equals 0.

Questions 4 and 5 relate to the 5


following information.
NO CALCULATOR
At a local school, a tenth grader’s
expected Grade Point Average (GPA) Which of the following statements is
can be represented by the equation the best interpretation of the number
y = –0.052x + 4, where x represents 4 in the context of this problem?
the number of absences that a student
has during the school year. A) The estimated GPA for a tenth
grade student with zero absences.
B) The estimated GPA for a tenth
grade student with four absences.
4 C) The estimated decrease in the
GPA for a tenth grade student for
NO CALCULATOR each absence during the school
year.
Which of the following statements is D) The estimated increase in the
the best interpretation of the number GPA for a tenth grade student for
–0.052 in the context of this problem? each absence during the school
year.
A) The estimated GPA for a tenth
grade student with zero absences.
B) The estimated GPA for a tenth
grade student with four absences.
C) The estimated decrease in the
GPA for a tenth grade student for
each absence during the school
year.
D) The estimated increase in the GPA
for a tenth grade student for each
absence during the school year.

119 119
SAT Slope

Solving Linear Equations: Slope y

run
Identify: You will be given two points on a (x2, y2)

rise
line in the xy-coordinate plane and be asked
(x1, y1) slope (m)
about the slope.

Set Up: Write down the slope formula and x


use it to solve for the missing information.

Slope Formula
change in y (rise) y2 − y1
slope (m) = =
change in x (run) x2 − x1

6 8

A line in a coordinate plane includes The graph of the linear function f has
points (2, b) and (12, 6). The slope of intercepts at (0, a) and (b, 0) in the xy-
the line is 0.4. What is the value of b ? plane. If a – b = 0, which of the
following is true about the slope of the
graph of f ?

A) It is positive.
7 B) It is negative.
C) It equals zero.
y
D) It is undefined.

(4, 5)
(–2, 1) (8, y)

x
a O
(–4, –4)
b

In the xy-plane above, line a is parallel


to line b. What is the value of y ?

120 120
SAT Solving Linear Equations Review

Questions 1, 2, and 3 relate to the Based upon the data in the graph, what
following information. would be closest to the expected velocity,
in kilometers/hour (km/hr), of a car that
6 has been traveling for 12 seconds?
5
Time (seconds)

A) 100 km/hr
4
B) 120 km/hr
3 C) 150 km/hr
2 D) 180 km/hr
1
0
0 20 40 60 80

Velocity (kilometers/hour)

The graph above displays the velocity, V, 4


in kilometers per hour, of a car
accelerating for s seconds. NO CALCULATOR

In the standard (x, y) coordinate plane,


1
what is the slope of the line that passes
What does the s-intercept represent in the through the origin and the point (3,8) ?
graph?

A) The initial resting velocity of the car.


B) The total number of kilometers the car
traveled.
C) The total number of seconds the car
traveled.
D) The increase in the velocity for each
second the car accelerated. 5

NO CALCULATOR
2

Which of the following represents the In the system of linear equations below,
relationship between s and V ? a is a constant. If the system has infinite
solutions, what is the value of a ?
1 3
A) s = 15V 8x – 7y = 12
1 15
B) s = V ax – 7 y = 60
15

C) s = 15V + 5

1
D) s = 15V + 5

121 121
122 122
SAT Ratio, Proportion, and Unit Conversion

SAT Math:
Ratio, Proportion, and
Unit Conversion

Important Formulas
Ratio Formula Direct Proportional Relationship
One Part One Part One Part
Ratio = =
Another Part Whole Thing Whole Thing

Probability
Desired Outcomes
Probability =
Total Outcomes

Answers on Page 300

123 123
SAT Ratio, Proportion, and Unit Conversion: Ratios and Proportional Relationships

Ratios and Proportional Relationships

Identify: If you see a question that defines a ratio and/or a proportional relationship.

Set Up: Write down the ratio and/or proportional relationship formulas and fill in
the provided information to solve the question. For proportional relationships, cross
multiply to solve for the unknown variable. Make sure your units are consistent.

Ratio Formula Direct Proportional Relationship


One Part One Part One Part
Ratio = =
Another Part Whole Thing Whole Thing

1 3

The scale on a wilderness map is such Census data for two local towns was
that 0.25 inches represents 0.5 miles. If recently released. The data are shown
a trail measures 2 inches on the map, in the table below.
how many feet long is the actual trail?
Male Female Total
(1 mile = 5280 feet)
City A 2,342 4,523 6,865
City B 4,352 2,123 6,475
A) 1,320 Total 6,694 6,646 13,340
B) 5,280
C) 10,560 Based on the data, how many times as
D) 21,120 likely is it that a randomly selected
person would be a female in City A as
compared to a randomly selected
person being a female in City B?
(Round to the nearest hundredth.)
2
A) 0.50 times as likely
A local restaurant serves soda to B) 0.75 times as likely
customers by the pint. Each week, the C) 1.51 times as likely
restaurant receives a 20 gallon D) 2.01 times as likely
shipment of soda syrup. How many
customers can be served with this
shipment? (1 gallon = 128 ounces; 1
pint = 16 ounces)

124 124
SAT Ratio, Proportion, and Unit Conversion: Ratios and Proportional Relationships

Ratios and Proportional Relationships (Colons)

Identify: If you see a question that defines a ratio and/or proportional relationship
between two or more things expressed with a colon.

Set Up: Represent each number in the ratio as the coefficient of a variable. Each
coefficient represents a portion of the overall total.

Proportion of Overall Total Relationship


An individual coefficent
Proportion =
The sum of the coefficents

4 5

If the degree measures of the angles


of a quadrilateral are in the ratio
2:3:5:6, by how many degrees does
the measure of the largest angle
exceed the measure of the smallest
angle?

Triangle ABC above is isosceles with


AB = AC and BC = 60. The ratio of
DE to DF is 7:8. What is the length
of DC ?

125 125
SAT Ratio, Proportion, and Unit Conversion: Probability

Probability

Identify: The question will mention “probability,” often in reference to data


presented in a chart or graph.

Set Up: List the elements you know, and plug them into the probability formula.

Probability Formula
Desired Outcomes
Probability =
Total Outcomes

6 7

Under 30 Over 30 Total Listen to Do Not Listen to


Male 18 6 24 Podcasts Podcasts
Female 12 4 16 Ages 20 to 24
Total 30 10 40 Ages 25 to 29
Total 126 35
The table above shows the The incomplete table above summarizes
distribution of age and gender for 40 the number of 20-to-24 years olds and
people who entered a contest. If the 25-to-29 year olds who listen to podcasts
contest winner will be selected at according to a recent survey. There are
random, what is the probability that 10 times as many 20-to-24 year olds who
the winner will be either a male
listen to podcasts as there are 20-to-24
under age 30 or a female age 30 or year olds who do not listen to podcasts,
older? and there are two times as many 25-to-
29 year olds who listen to podcasts as
there are 25-to-29 year olds who do not
listen to podcasts. If there is a total of
126 survey respondents who listen to
podcasts and 35 survey respondents
who do not listen to podcasts, which of
the following is closest to the probability
that a person who listens to podcasts
selected randomly is 20-to-24 years old?
(Rounded to the nearest thousandth.)

A) 0.217
B) 0.444
C) 0.556
D) 0.783

126 126
SAT Ratio, Proportion, and Unit Conversion: Unit Conversion

Unit Conversion

Identify: You are being asked a question which asks for a conversion between different
units.

Set Up: Use relevant conversions to cancel units between the provided unit(s) and the
desired unit(s).

8 10

Kathleen runs two miles in 13.5 A typical image taken by Erin’s cell
minutes. If she runs at the same rate, phone camera is 17.5 megabits in size.
which of the following is the closest to Erin can upload her photos to the
the distance she will run in 5,000 internet at a rate of 12 kilobits per
seconds? second for a maximum of 10 hours each
day. If 1 megabit equals 1,024 kilobits,
A) 7.5 miles what is the maximum number of
B) 10.0 miles typical images that Erin can upload to
C) 12.5 miles the internet each day?
D) 15.0 miles

A local box store sells wallpaper in

such large amounts that one roll can

cover up to five walls in Ethan’s


apartment. If one of Ethan’s walls has
2
an area of approximately 1403 square

feet, about how many square feet could

18 rolls of wallpaper cover?

A) 800
B) 2,780
C) 7,650
D) 12,700

127 127
SAT Ratio, Proportion, and Unit Conversion Review

1 4

1 kilogram = 1,000 grams After the first term, each term in a


1,000 milligrams = 1 gram 1
sequence is 2 greater than of the
4
A hardware store sells one type of preceding term. If t is the first term of the
fertilizer in 4-kilogram containers. Based
on the information given in the box above, sequence and t ≠ 0, what is the ratio of the
how many 1-milligram doses are there in second term to the first term?
one 4-kilogram container?
t
A) +2
A) 0.004 4
B) 400 t
C) 40,000 B) 2 + 4
D) 4,000,000
t+2
C)
4t
t+8
D)
4t

A local television station sells time slots


for programs in 15-minute intervals. If the 5
station operates 24 hours per day, every
day of the week, what is the total number On a recent safari, Jessica saw 80 lions
of 15-minute time slots the station can sell
for Friday, Saturday, and Sunday? and 40 tigers. If Jessica only saw lions and
tigers, how many more tigers would she
7
have needed to see for 8 of the total
animals she saw to be tigers?

A) 280
B) 320
3
C) 420
Jack traveled by speedboat at an average D) 520
speed of 45 miles per hour for 4.5 hours
and consumed fuel at a rate of 12 miles per
gallon. How many gallons of fuel did the
speedboat use for the entire 4.5-hour trip?
(Round to the nearest gallon.)

128 128
SAT Words in Context

SAT Reading and Writing:


Words in Context

Answers on Page 295

129 129
SAT Words in Context (Reading)

Words in Context (Reading)

Identifying Words in Context Questions on the SAT: On Words in Context questions,


students are asked to define a word within the context of the surrounding text. No prior
knowledge of the word is necessary.

To solve Words in Context questions, first anticipate a word or tone that fits within the
surrounding text. Then, use your anticipation to eliminate answers that do not align
with your anticipation.

Question 1 Refers to the Paragraph Below

In his 1828 biography of Columbus, 1


Washington Irving postulated that Columbus's
initial failures to procure funding were due to As it is used in the paragraph, “stemmed”
European monarchs’ assumption that the world most nearly means
was flat. However, the maritime navigation of
the time relied on the curvature of the earth, A) caused.
and modern scholars recognize that Columbus's B) produced.
difficulty in obtaining funds stemmed from a C) originated.
different source. In initial meetings, would-be D) diverges.
backers agreed that Columbus's plan was
infeasible because he had vastly underestimated
the size of the westward expanse between
Europe and Asia.

Question 2 Refers to the Paragraph Below

With the eighteenth century, the rediscovery 2


of Greece began. In 1748, a British society of
noblemen who had visited Italy and acquired an As it is used in the paragraph, “flood” most
appreciation of classical architecture nearly means
commissioned James Stuart, a painter-architect,
and Nicholas Revett, an architect draughtsman, A) swell.
to visit Greece and produce drawings of as many B) invasion.
of the ancient monuments as they could. Stuart C) catastrophe.
and Revett founded the Greek Revival style, D) drowning.
which would ride the full flood of the nineteenth
century romanticism and, in the United States,
would become virtually the official architecture
of the new republic.

130 130
SAT Words in Context (Writing)

Words in Context (Writing)

Identifying Words in Context Questions on the SAT: On Words in Context questions,


students are asked to define a word within the context of the surrounding text. No prior
knowledge of the word is necessary.

To solve Words in Context questions, first anticipate a word or tone that fits within the
surrounding text. Then, use your anticipation to eliminate answers that do not align
with your anticipation.

Question 3 Refers to the Paragraph Below

With repetition, a great number of 3


impressions will soon occur, which will associate
themselves with other impressions. These will A) NO CHANGE
often be accompanied by the expression of the B) suggestion
patient’s opinion that they have no meaning or C) expression
are unimportant. According to Freud, this is the D) disclosure
implication of unconscious self-criticism that lies
at the heart of the patient’s neurosis.

Question 4 Refers to the Paragraph Below

Heroes are extraordinary people.


Trailblazers, they dare to go where no one has 4
gone before. When Amelia Earhart became the
A) NO CHANGE
first woman passenger to cross the Atlantic
B) desired
Ocean in an airplane, she also became America’s
C) intimated
national hero, a role she merited for the rest of
D) regarded
her life. Her courage in daring to make the
flight, her honesty in insisting that the pilot
deserved all of the credit, her modesty in
accepting the accolades, or praise, were traits
that caught the fancy of the country and of the
world, as well.

131 131
SAT Words in Context Review

Question 1 Refers to the Paragraph Below

Everyone wanted to see the men on the 1


mountain. Gutzon Borglum, who regarded his
masterpiece as far more than a tourist As it is used in the paragraph, “breathe”
attraction, was no doubt reassured when the most nearly means
phrase “Shrine of Democracy” was coined at the
1930 dedication of the Washington head. “A A) blow.
monument’s dimensions,” said Borglum, “should B) respire.
be determined by the importance to civilization C) send.
of the events commemorated . . . Let us place D) utter.
there, carved high, as close to heaven as we can,
the words of our leaders, their faces, to show
posterity what manner of men they were. Then
breathe a prayer that these records will endure
until the wind and the rain alone shall wear
them away.”

Question 2 Refers to the Paragraph Below

The movement of sound in the atmosphere 2


depends on the properties of the air. This means
that changes in temperature and density can A) NO CHANGE
affect how far away thunder can be heard. B) appear
Sometimes atmospheric conditions limit the C) impact
ability of the sound of thunder to transport. D) carry

Question 3 Refers to the Paragraph Below

The neighbor shrugged and glanced at 3


Ernest Hemingway’s empty house, a comfortable
looking chalet with a big pair of elk horns over A) NO CHANGE
the front door. It is built on a hillside looking B) embellished
down on the Big Wood River, and out across the C) spartan
valley at the Sawtooth Mountains. D) pure
A mile or so away, in a small graveyard at the
north end of town, is Hemingway’s easy grave,
lying in the afternoon shadow of Baldy Mountain
and the Sun Valley ski runs.

132 132
SAT Words in Context Review

Question 4 Refers to the Paragraph Below

As Lebron James has developed as a 4


basketball player, his skills have been compared
to those of Magic Johnson, a former player for As it is used in the paragraph, “vacated”
the Los Angeles Lakers. James, however, has most nearly means
vacated from Johnson in a number of ways, most
notably in his ability to use his strength and A) eliminated.
overall athleticism to score easy baskets. B) evacuated.
C) departed.
D) retired.

Question 5 Refers to the Paragraph Below

There is an old trail many years long that 5


leads back to a land of long ago when Arizona
was the last frontier. This is the trail to the land As it is used in the paragraph,
of the Snake Dance of the Hopi Indians in the “penetrated” most nearly means
high mesa country of northern Arizona. Weird
almost beyond description, it has been A) entered.
celebrated each year since long before the first B) punctured.
white man penetrated that barren land and C) seized.
gazed upon the almost incredible spectacle of D) affected.
half-naked, grotesquely painted desert Indians
dancing with big diamondbacks or deadly
sidewinders held in their mouths.

133 133
134 134
SAT Parallelism

SAT Writing:
Parallelism

Answers on Page 297

135 135
SAT Parallelism

Parallelism

Proper grammar requires that all items in a list or all related clauses be in the same
grammatical form (nouns, verbs, correlative conjunctions, etc.).

Identifying Parallelism Issues on the SAT: Look for two or more items or clauses in a
list.

1 3

Leonardo da Vinci was born in A local civic group is working hard to


Florence in 1452, and many people recruit new members. The group has
say he defined the “Renaissance thus formed a club dedicated to
man.” Da Vinci was a talented performing music in a local park.
painter, sculptor, mathematician, Some pieces don’t fare well with the
architect, and he wrote many books. public (the one attempt at bluegrass
comes to mind), while others have
A) NO CHANGE worked just fine.
B) architect, and he wrote many
books. A) NO CHANGE
C) architect, and writer. B) work
D) architect that has also written C) worked
many books. D) could have worked

On Monday, Steve wakes up and


goes to work. On Wednesday, Steve
heads to the store to buy bread and
eggs. On Friday, Steve drives to the
bank to withdraw money for his rent.

Which choice best maintains the


sentence pattern already established
in the paragraph?

A) NO CHANGE
B) To withdraw money for his rent,
on Friday Steve drives to the
bank.
C) Steve drives the bank on Friday
to withdraw money for his rent.
D) Steve withdraws money from the
bank to pay his rent.

136 136
SAT Parallelism: Correlative Conjunctions

Parallelism: Correlative Conjunctions

Correlative conjunctions are conjunctions that come in pairs, and you have to use both of
them to maintain parallelism in a sentence.

Identifying Parallelism Issues: Correlative Conjunctions on the SAT: Look for the
below conjunctions. Do they occur in a grammatically correct pairing?

between … and
both … and
either … or
neither … nor
not … but
not only … but also

4 5

During the early part of the 21st Research on dinosaur fossils not only
century, companies have increasingly raises questions about the
promoted fitness for employees. In appearance of dinosaurs (feathers?
light of the resulting benefits, not wings? scales?), but also allows us to
only to employee productivity and better classify different types of
again to the bottom line of dinosaurs.
corporations in the form of reduced
health care costs, an expansion of A) NO CHANGE
these programs can be expected. B) and also allows
C) it also allows
A) NO CHANGE D) allowing
B) but it benefits
C) as also to
D) but also to

137 137
SAT Parallelism Review

1 4

In recent years, genetic modifications have Selling old items can be achieved in a
increased crop yields and enhanced the variety of ways: some people have garage
nutritional content of food on the market. sales, others are putting ads in
These developments have both newspapers, and still others choose online
revolutionized food creation and inspired auctions to clean out their basements.
ethical debates about inserting human
engineering into natural processes. A) NO CHANGE
B) they put
A) NO CHANGE C) by putting
B) and also inspired D) put
C) but also inspired
D) will inspire
5
2
The shopping mall’s construction is not
Increases in the stock market result from going according to plan this morning.
a healthy economy and because people are Neither the foreman or the building
confident in the government. materials has arrived on time.

A) NO CHANGE A) NO CHANGE
B) because of the confidence B) Either the foreman or the building
C) confidence materials
D) being confident C) Either the foreman nor the building
materials
D) Neither the foreman nor the building
materials
3

Louis decided that he only had two


options: either to accept the socks he
received for Christmas or return the gift
and get what he truly wanted.

A) NO CHANGE
B) nor return the gift
C) but return the gift
D) or to return the gift

138 138
SAT Average, Median, and Mode

SAT Math:
Average, Median,
and Mode

Important Formulas

Average (Arithmetic Mean) Formula


Sum
Average =
Total Number

Weighted Average Formula


Sum of Weighted Terms
Weighted Average =
Total Number

Answers on Page 300

139 139
SAT Average, Median, and Mode: Average (Arithmetic Mean)

Average, Median, and Mode: Average (Arithmetic Mean)

Identify: If you see the term average (arithmetic mean) in a math question, it is an
average question.

Set Up: Immediately write down the average formula. Plug whatever information is
given in the question into the formula and solve for whatever is missing in the equation.
Repeat as necessary.

Average Formula Weighted Average Formula


Sum Sum of Weighted Terms
Average = Weighted Average =
Total Number Total Number

1 3

John runs the 400 meter dash for his A company receives customer
high school track team. In his first satisfaction ratings on a scale of
five races this year, John ran the 400 integers from 0 to 10, inclusive. On
meter dash in the following times: the first six surveys that the company
52.4 seconds, 54.6 seconds, 53.2 received, the average (arithmetic
seconds, 56.4 seconds, and 55.9 mean) of the ratings was 7.7. What is
seconds. What is the average time, the least rating the company can
in seconds, that Jonathan completed receive on the seventh survey and still
these five races? (Round to the be able to have an average of at least
nearest tenth of a second.) 8 for the first 10 surveys? (Round to
the nearest tenth.)

A local town has elementary schools


and high schools. The seven
elementary schools average 140
students and the two high schools
average 410 students. What is the
average number of students in the
nine schools?

140 140
SAT Average, Median, and Mode: Median and Mode

Average, Median, and Mode: Median and Mode

Identify: If you see the term median or mode, it is a median and mode question.

Set Up: Median is the middle number in a set arranged from least to greatest. Mode
is/are the number(s) that appear(s) most frequently in a set.

4 5

Siblings Survey NO CALCULATOR

Number of Suburb A survey was taken of the value of


Capital City
Siblings Town
boats in a marina, and it was found
0 20 25 that the mean boat value was $85,000
1 35 45 and the median boat value was
2 55 30 $145,000. Which of the following
situations could explain the difference
3 10 20
between the mean and median boat
4+ 5 5 values in the marina?

A researcher chose 250 third graders A) Many of the boats have values
from two local towns and asked each between $85,000 and $145,000.
student how many siblings he or she B) There are a few boats that are
has. The results are shown in the table valued much less than the rest.
above. What is the median number of C) There are a few boats that are
siblings for all students surveyed? valued much more than the rest.
D) It cannot be determined from the
A) 1.5 given information.
B) 2.5
C) 3
D) 3.5

141 141
SAT Average, Median, and Mode Review

3
Questions 1 and 2 relate to the following NO CALCULATOR
information.
In Jessica’s bowling league, the mean
The histogram below shows the distribution of score of all female players is 185, and the
scores on a recent test. mean score of all male players is 149.
Which of the following must be true about
5 the mean score b of the combined group of
4 male and female members of the bowling
Frequency

3 league?
2
A) b < 149
1 B) b = 167
0 C) 149 < b < 185
67 72 83 88 91 94 98 99 D) b > 185
Score

Which of the following is the mean score of 4


the 12 students? (Round to the nearest
tenth.) The histogram below shows the distribution
of the heights of players, in inches, on a
A) 85.9 local basketball team.
B) 88.0
C) 89.2 5
Frequency

D) 90.8 4
3
2
2 1
0
Which of the following is the median score 75 76 77 78 79 80 81 82 83
of the 12 students? Height (inches)

A) 85.5
B) 88.0 What is the average (arithmetic mean)
C) 89.5 height of the members of the basketball
D) 91 team? (Round to the nearest tenth of an
inch.)

142 142
SAT Average, Median, and Mode Review

If a is the average (arithmetic mean) of 5x


and 15, b is the average of 7x and 6, and c
is the average of 12x and 21, what is the
average of a, b, and c in terms of x ?

A) 2x + 6
B) 2x + 14
C) 4x + 7
D) 8x + 4

143 143
144 144
SAT Functions

SAT Math:
Functions

Answers on Page 300

145 145
SAT Functions: f(x)

Functions: f(x)

Identify: Any question that includes function notation with numbers or variables
such as f(4), g(t), or h(6).

Set Up: The number or variable in the parentheses is the item to plug in to the function
wherever you see a variable.

1 3

NO CALCULATOR 4
q(t) = 3t + 4a
If f(x) = 2x2 + 8, what is f(3s) equal
to? In the function above, a is a constant. If
q(6) = 10, what is the value of q(9) ?
A) 12s + 8
B) 12s2 + 8
C) 18s + 8
D) 18s2 + 8

4
2

NO CALCULATOR In the xy-plane, the point (2, 10) lies on


the graph of the function f(x) = 2x2 + dx
A function q satisfies q(4) = 10 and + 6. What is the value of –4d ?
q(3) = 20. A function r satisfies r(3) =
15 and r(20) = 8. What is the value of
r(q(3)) ?

A) 3
B) 8
C) 15
D) 20

146 146
SAT Functions: f(x ± a)

Functions: f(x ± a)

Identify: Any question that includes function notation with a variable AND a
number and/or a second variable in the original function such as f(x – 3), g(t + 4),
or h(a + b)

Set Up: If the original input is an equation (for example, x – 1), first solve for the variable
by setting the two values in parentheses equal to one another and then input the new value
into the equation.

5 7

If the function f is defined by f(x + 2) If the function z is defined by z(Ф +


= 3x + 6, what is the value of f(5) ? 8) = 2Ф – 10, what is the value of
z(20) ?
A) 5
B) 12
C) 15
D) 27

If the function j is defined by j(k – 10)


= 3k2 + 15, what is the value of j(4) ?

147 147
SAT Functions: Graphs

Functions: Graphs

Identify: Problems involving a graph of variables (x) and functions f(x).

Set Up: The input to the function (what is inside the parentheses) is also an x-
coordinate on the graph. The output of the function is the corresponding y-
coordinate on the graph.
y

y = f(x)

y = g(x)
2

x
2

Questions 8, 9, and 10 relate to the above graph

8 10

NO CALCULATOR NO CALCULATOR

The graph of y = f(x) is shown above. Graphs of the functions f and g are
What is f(3) ? shown in the xy-plane above. For which
of the following values of x does f(x) +
g(x) = 7 ?

A) 0
B) 1
9 C) 4
D) 5
NO CALCULATOR

The graph of function g is shown in


the xy-plane above. Which of the
following is equal to 1 ?

A) g(1)
B) g(3)
C) g(5)
D) g(6)

148 148
SAT Functions: Tables

Functions: Tables

Identify: Problems involving a table of variables (x) and functions f(x).

Set Up: Pick a variable (x) from the table and plug it into each of the answer
choice functions. The output value should equal the corresponding f(x) from the table.

11 12

NO CALCULATOR NO CALCULATOR

x 2 4 6 8 x 0 1 2 3
f(x) 0 4 8 12 y 0 2 4 6

The table above gives values of the The table above gives values of the
linear function f(x) for selected linear function f(x) for selected
values of x. Which of the following values of x. Which of the following
defines f(x) ? defines y ?

A) f(x) = x + 2 A) y=x
B) f(x) = x – 2 B) y = x2
C) f(x) = 2x + 2 C) y = 2x
D) f(x) = 2x – 4 D) y=x+1

149 149
SAT Functions Review

1 4
y
NO CALCULATOR
15
If h(x) = 12x + 14, then h(13) =
10
5

–15 –10 –5 O 5 10 15 x
–5
–10
–15

2
The complete graph of the function f is
above. Which of the following are equal to
NO CALCULATOR
15 ?
If f(x – 4) = 3x + 4, then f(5) =
I. f(–15)
II. f(5)
A) 5
III. f(15)
B) 7
C) 23
A) II only
D) 31
B) I and II only
C) II and III only
D) I, II, and III

5
NO CALCULATOR

NO CALCULATOR

q –1 0 1 2
f(q) –4 –2 0 2

The table above gives values of the linear


function f(q) for selected values of q.
Which of the following defines f(q) ?
At what value of x, for –6 ≤ x ≤ 8, does the
function graphed above achieve its A) f(q) = q – 3
maximum value? B) f(q) = q – 2
C) f(q) = 2q
A) –6 D) f(q) = 2q – 2
B) –3
C) 3
D) 8

150 150
SAT Evaluating Quantitative Information

SAT Reading and Writing:


Evaluating Quantitative
Information

Answers on Page 295

151 151
SAT Evaluating Quantitative Information

Evaluating Quantitative Information

Identifying Evaluating Quantitative Information Questions on the SAT: On the


SAT, quantitative information questions assess a student’s ability to understand a chart
or graph and connect it to a passage.

To solve quantitative information questions, there are two things to keep in mind:

1) What is the data about?: This can be determined by answering the question
“What does the header of the graphic say?”

2) What does the data in the graphic show about this topic?: It is
important to look for trends in the data (When one thing increases, another
thing increases; some things are above average and some things are below
average, etc.).

To be correct, an answer must be related to the answers to these questions.

Selected Occupations
Percent Increase in Employment, Projected 2015-2025

Biomedical Engineers

Physical Therapists

Total, All Occupations

0% 10% 20% 30% 40% 50% 60% 70%

The United States Bureau of Labor Statistics 1


forecasts job growth in given occupations,
projecting that the job of physical therapist will Which choice completes the sentence with
be the fastest growing occupation from 2015 to accurate data based on the graph?
2025. 1
A) NO CHANGE
B) projecting that the job of biomedical
engineer will be the fastest growing job
of all occupations from 2015 to 2025.
C) projecting that the job of physical
therapist will grow faster than the job of
biomedical engineer from 2015 to 2025.
D) projecting that the job of physical
therapist will grow faster than the
average of all occupations from 2015 to
2025.

152 152
SAT Evaluating Quantitative Information

Evaluating Quantitative Information

Identifying Evaluating Quantitative Information Questions on the SAT: On the


SAT, quantitative information questions assess a student’s ability to understand a chart
or graph and connect it to a passage.

Questions 2 and 3 refer to the below information

Figure 1 Figure 2
Human Body, Fraction of Mass by Element Human Body, Atomic Percent1

Phosphorous Calcium Phosphorous Calcium


1% 1% Nitrogen <1% <1%
Nitrogen Hydrogen 1%
3% 10%

Carbon
12%
Carbon
18%

Oxygen 24%
Hydrogen 62%
Oxygen 66%

1Atomic percent gives the percentage of one kind of atom relative to the total number of atoms.

2 3

Which choice is supported by the data in Taken together, the two figures suggest
the first figure? that

A) In the human body the number of A) oxygen atoms have a significantly


oxygen atoms is greater than the greater mass than hydrogen atoms.
number of carbon atoms. B) oxygen atoms have a significantly
B) In the human body the fraction of mass greater mass than carbon atoms.
consisting of nitrogen is greater than C) the human body contains a similar
the fraction of mass consisting of amount, in atomic percentage, of carbon
calcium. and hydrogen.
C) In the human body the fraction of mass D) the universe, on average, contains two
consisting of hydrogen is greater than oxygen atoms for every one carbon
the fraction of mass consisting of atom.
oxygen.
D) In the human body the fraction of mass
consisting of carbon is roughly the
same as the fraction of mass consisting
of phosphorous.

153 153
SAT Evaluating Quantitative Information

Evaluating Quantitative Information

Identifying Evaluating Quantitative Information Questions on the SAT: On the


SAT, quantitative information questions assess a student’s ability to understand a chart
or graph and connect it to a passage.

Question 4 refers to the below information

Figure 1: 1860 United States Presidential Election Results

The 1860 United States Presidential Election 4


saw four different candidates win the electoral
votes from at least one state. As the map shows, The writer wants the information in the
Lincoln received the most support in the north passage to correspond as closely as
and in the west; Douglas receive the most possible with the information in the map.
support in the south, and Bell received the most Given that goal, in which sequence should
support in only one state. 4 the names of the three presidential
candidates be listed?

A) NO CHANGE
B) Lincoln, Bell, Breckinridge
C) Lincoln, Breckinridge, Douglas
D) Breckinridge, Lincoln, Douglas

154 154
SAT Evaluating Quantitative Information Review

Questions 1 and 2 refer to the below information

Chemical Composition of Selected Rocks


Percentage of chemical composition
Rock
SiO2 (%) Al2O3 (%) CaO (%) CO2
Average Igneous Rock 59.1 15.3 5.08 0.1
Average Shale 58.1 15.4 3.11 2.6
Average Sandstone 78.3 4.77 5.5 5
Average Limestone 5.19 0.81 42.6 41.5

Ever since humans have learned how to 2


extract the component materials of rocks, they
have taken great interest in understanding the Based on the table and the passage, do the
Line chemical compositions of the rocks found in the data in the table support the author’s
5 Earth’s crust. By the end of the 20th century, proposed pairing of chemicals?
technology had advanced to the point where the
relative percentages of these chemicals could be A) Yes, because for each given rock, the
known to many decimal places. This research percentage of Silicon dioxide is
has routinely demonstrated that within rocks, proportional to the percentage of
10 certain pairs of chemicals are likely to be found Aluminum oxide, and the percentage of
in similar proportions. These pairs of chemicals Calcium oxide is proportional to the
include Silicon dioxide (SiO2) and Aluminum percentage of Carbon dioxide.
oxide (Al2O3), Calcium oxide (CaO) and Carbon B) Yes, because for each given rock, the
15 dioxide (CO2), and Silicon dioxide (SiO2) and percentage of Silicon dioxide is
Carbon dioxide (CO2). proportional to the percentage of
Aluminum oxide, and the percentage of
Silicon dioxide is proportional to the
percentage of Carbon dioxide.
1
C) No, because for each given rock, the
percentage of Silicon dioxide is
Based on the table and the passage, which proportional to the percentage of
choice gives the correct percentage of the Aluminum oxide, but the percentage of
dioxides in an average shale rock? Calcium oxide is not proportional to
the percentage of Carbon dioxide.
A) 59.1% and 15.3% D) No, because for each rock, the
B) 58.1% and 15.4% percentages for neither of the three
C) 58.1% and 2.6% pairs of chemicals are in proportion to
D) 78.3% and 2.6% one another.

155 155
SAT Evaluating Quantitative Information Review

Questions 3 and 4 refer to the below information

Figure 1 Figure 2

Zone A Zone B

20 20
Percentage of weeds killed

Percentage of weeds killed


15 15

10 10

5 5

0 0
Untreated Treated Untreated Treated

Scientists at the local state university ran an 3


experiment to test the effectiveness of a weed
killer recently released to the market. This Which choice offers the most accurate
experiment consisted of testing the weed killer interpretation of the data in the charts?
on two different types of weed. Each type of
weed was contained within a particular zone A) NO CHANGE
(Zone A or Zone B) and a control was set up B) substantially fewer weeds
within each of the zones. As the graphs show, in C) five percent more weeds
Zone A three times as many weeds were killed D) twice as many weeds
on the treated area of Zone A than on the
untreated area of Zone A. 3 This experiment
demonstrates that the new weed killer is 4
effective at killing all types of weeds. 4
Which choice offers the most accurate
interpretation of the data in the charts?

A) NO CHANGE
B) Considering the results of the weeds
treated in Zone B, the results in Zone
A are likely an anomaly.
C) The weeds killed in Zone A also grew
back less quickly than the weeds killed
in Zone B.
D) This result was not replicated on the
weeds treated in Zone B.

156 156
SAT Pronouns

SAT Writing:
Pronouns

Answers on Page 297

157 157
SAT Pronouns: Singular vs. Plural

Pronouns: Singular vs. Plural

Pronouns are words that take the place of nouns. Pronouns are used to reduce the amount
of repetition in a sentence.

A pronoun error on the SAT is to have a singular subject paired with a plural
pronoun or a plural subject paired with a singular pronoun.

Identifying Singular vs. Plural Pronoun Issues on the SAT: Look for an underlined
pronoun in the sentence. To check for singular vs. plural agreement, find the noun
which the pronoun in replacing. Is the noun singular or plural? Does the pronoun
match?

1 3

Each summer we go on a group My doctor says that eating fruits and


camping trip; it is important that each vegetables is important because it
person remembers to bring their own improves sleep and help prevent
supplies because nobody brings extras. disease.

A) NO CHANGE A) NO CHANGE
B) its B) it improve
C) there C) they improve
D) his or her D) they improves

2
Many towns have started to install
cameras on highways to improve
safety. However, some studies indicate
that instead of these cameras
improving safety, they actually
increase the number of accidents.

A) NO CHANGE
B) they actually increases
C) it actually increases
D) it actually increase

158 158
SAT Pronouns: Personal Pronouns

Pronouns: Personal Pronouns

Pronouns are words that take the place of nouns. Pronouns are used to reduce the amount
of repetition in a sentence.

A pronoun error on the SAT is to switch between the types of personal pronouns (1st,
2nd, and 3rd person pronouns).

Identifying Personal Pronoun Issues on the SAT: Look for one or you in the
sentence. A sentence can use one (3rd personal pronoun) or you (2nd person pronoun), but
cannot switch between them.

4 6

If you want to enter the university’s When training for a marathon be


engineering program, one must first careful not to hurt oneself: injuries
demonstrate familiarity with basic can easily be sustained by increasing
concepts from math and physics. your activity too quickly.

A) NO CHANGE A) NO CHANGE
B) you must firstly B) hurt oneself,
C) you must first C) hurt yourself:
D) they must first D) hurt yourself,

In his lecture on vehicle safety, the


instructor emphasized that you must
allow extra time to brake when you
are driving on a wet road.

A) NO CHANGE
B) you must be allowing
C) one must allow
D) we must allow

159 159
SAT Pronouns: Possessive Pronouns

Pronouns: Possessive Pronouns

Pronouns are words that take the place of nouns. Pronouns are used to reduce the amount
of repetition in a sentence.

A pronoun error on the SAT is to replace a possessive noun with a non-


corresponding pronoun.

Identifying Possessive Pronoun Issues on the SAT: Look for pronouns that indicate
possession in the sentence (my, her, your, their). Are they aligning with the subject
correctly?

We, our, and ourselves are only valid when the writer is part of the group being
discussed.

7 9

Although granite countertops create During the weekend of the wedding,


a lovely aesthetic, its delicate nature my parents hosted many of our
makes them much more likely to family members. They definitely
form cracks and require expensive made themselves at home: eating all
repairs. of our snacks and moving our clothes
into the available closet space.
A) NO CHANGE
B) their A) NO CHANGE
C) it’s B) his or her
D) its C) their
D) one’s

Washington D.C. attracts many


tourists because of its fascinating
history and beautiful buildings.

A) NO CHANGE
B) there
C) their
D) it’s

160 160
SAT Pronouns: Pronoun Case

Pronouns: Pronoun Case

Pronouns are words that take the place of nouns. Pronouns are used to reduce the amount
of repetition in a sentence.

A pronoun error on the SAT is to have two pronouns, or a pronoun next to a noun, in
a sentence while misusing one, or both, of the words.

Identifying Pronoun Case Issues on the SAT: Look for two underlined pronouns
or a pronoun next to a noun. To check usage, read the sentence twice and plug in one
subject each time. Use your grammar ear to identify errors.

10 12

Grandmother always told my brother Other African American musicians


and I not to play baseball in the and athletes suffered under the same
house, yet we never listened to her constraints. As a result, Anderson
until the day we broke her favorite traveled to Europe, where her and
antique vase. other American minorities were able
to perform and refine their skills
A) NO CHANGE more freely.
B) us
C) I and my brother A) NO CHANGE
D) my brother and me B) she
C) herself
D) they

11

Both James and Allie wanted to see


the movie, so he and her were willing
to fight the terrible traffic.

A) NO CHANGE
B) he and her were
C) her and him were
D) he and she were

161 161
SAT Pronouns: Unclear Pronoun Subject

Pronouns: Unclear Pronoun Subject

Pronouns are words that take the place of nouns. Pronouns are used to reduce the amount
of repetition in a sentence.

A pronoun error on the SAT is to a have a pronoun without a clear subject.

Identifying Unclear Pronouns Subject Issues on the SAT: Look for an underlined
pronoun. Is the pronoun replacing a noun? Is there vagueness about which noun the
pronoun is replacing?

13 15

Death Valley is located in the east- After adjusting the data, the mortar
central part of California, and is part revealed that the Pyramid must have
of the larger Mojave Desert. It was been built between 3100 B.C. and
named by miners who were heading 2850 B.C., with an average date of
west during the 1849 California gold 2977 B.C. This discovery was
rush. Many miners died trying to controversial because it suggested
cross it. that the structure was built more
than 400 years earlier than most
A) NO CHANGE archeologists originally believed.
B) the valley
C) over A) NO CHANGE
D) OMIT the underlined portion B) these dates
C) it was
D) they

14

Each Saturday, my father and my


grandfather would go to the game
and loudly root for his favorite team.

A) NO CHANGE
B) its
C) my father’s
D) my

162 162
SAT Pronouns Review

1 4

When a teenager moves out of their Because computer science jobs will become
parents’ house to attend college, the so prevalent during our lifetimes, students
parents often feel a mixture of joy and should be required to learn computer
sadness upon realizing that their child has programming before they graduate high
become an adult. school.

A) NO CHANGE A) NO CHANGE
B) its B) one’s
C) they’re C) his or her
D) his or her D) their

2 5

In 1689, France held many settlements When a massive star dies its core collapses
that lay north of the English colonies, but on itself, greatly increasing it’s density.
they also claimed southern land in the
Mississippi River Valley. A) NO CHANGE
B) its
A) NO CHANGE C) the core’s
B) they also claim D) OMIT the underlined portion
C) it also claimed
D) it also claims

When we went to the shelter to choose an


animal, our parents informed us that the
responsibility for taking care of the new
kitten would fall on I and my sister.

A) NO CHANGE
B) my sister and I
C) my sister and me
D) her and I

163 163
164 164
SAT Verbs

SAT Writing:
Verbs

Answers on Page 297

165 165
SAT Verbs: Subject-Verb Agreement

Verbs: Subject-Verb Agreement

Verbs are words that convey actions, occurrences, or states of being.

In proper grammar, singular subjects are paired with singular verbs and plural subjects are
paired with plural verbs.

Identifying Subject-Verb Agreement Issues on the SAT: Look for an underlined verb
in the sentence. To check for subject and verb agreement, place the verb and its
subject directly next to each other, read them together, and check using your
grammar ear.

1 3

Along Berkshire County’s western The book, a compilation of 300


border run the Taconic Mountains, to photographs by 100 photographers,
the east lie the Hoosacs, and offers a glimpse into daily life in the
straddling the Vermont border is the urban centers of Southeast Asia.
Green Mountains.
A) NO CHANGE
A) NO CHANGE B) is offering a glimpse
B) were C) has offered a glimpse
C) are D) offer a glimpse
D) have been

Much of the literature on the Civil


War, particularly those works
released immediately following the
war’s conclusion, are notable for
their portrayal of Robert E. Lee as a
grand anachronism.

A) NO CHANGE
B) are notable for its
C) is notable for their
D) is notable for its

166 166
SAT Verbs: Tense

Verbs: Tense

Verbs are words that convey actions, occurrences, or states of being.

In proper grammar, verb tense must align with the tense of the surrounding text.

Identifying Verb Tense Issues on the SAT: Look for an underlined verb in the sentence.
When checking tense alignment, look for descriptive text (“ending in 1994”) and
the tense of surrounding verbs to indicate the tense of the sentence.

4 6

The Beatles were one of the world’s The documentary of the 2010
most popular musical groups until championship shows the final, tense
internal discord drove the members moments of the game. The rabid fans
apart; the band breaks up in 1970. cheer as the captain walked to the
free throw line to attempt the
A) NO CHANGE winning shots.
B) had broken up
C) broke up A) NO CHANGE
D) breaking up B) had walked
C) would walk
D) walks
5

Tennessee Williams’s drama The


Glass Menagerie is considered a
classic of the American Theatre since
its 1944 debut.

A) NO CHANGE
B) was considered
C) would be considered
D) has been considered

167 167
SAT Verbs Review

1 4

Edward Abbey’s essays earn him praise as The class of students were excited when
a leading American environmentalist Mr. Smith said they would be watching a
during the late 1960s, a period during movie.
which he wrote prolifically.
A) NO CHANGE
A) NO CHANGE B) was excited
B) earned him praise C) is excited
C) have earned him praise D) are excited
D) would earn him praise

5
2
There is an ongoing disagreement between
All cells, even the specialized cells in a those who want to actively manage the
large, multi-cellular creature, have several environment and those who believe that
things in common. For example, all cells humans should take a “hands off”
are composed of approximately 90% water. approach. For example, some people
A thin, protective covering called a advocate active management of forests
membrane enclosing every cell. that include removing trees and brush to
prevent wildfires.
A) NO CHANGE
B) enclose A) NO CHANGE
C) encloses B) includes
D) encloses with C) will include
D) including

Neither her mom nor her dad were


available to drive Becky to her soccer
game, so she arranged to get a ride with
her friend.

A) NO CHANGE
B) was available
C) have been available
D) has been available

168 168
SAT Quadratics and Polynomials

SAT Math:
Quadratics and
Polynomials

Important Formulas
Quadratic Formula

2
–b ± b – 4ac
x=
2a

Answers on Page 301

169 169
SAT Quadratics and Polynomials: Quadratics

Quadratics and Polynomials: Quadratics

Identify: Look for a quadratic equation or binomials (two things in parentheses being
multiplied together).

Set Up: Keep the FOIL rules in mind: First, Outer, Inner, Last.

Look for patterns, like these three commonly tested quadratics:

(x + y)2 = (x +y)(x + y) = x2 + 2xy + y2


(x – y)2 = (x – y)(x – y) = x2 – 2xy + y2
(x + y)(x – y) = x2 – y2

1 3

If x2 – y2 = 117 and x – y = 3, what is NO CALCULATOR


the value of (x + y) ?
If (ax + 3)(bx + 4) = 6x2 + cx + 12 for
all values of x, and a + b = 5, what
are the two possible values for c ?

A) 3 and 4
B) 6 and 12
C) 17 and 18
2
D) 24 and 36
4x4 + 16x2y2 + 16y4

Which of the following is equivalent


to the expression shown above?

A) (2x2 + 4y2)2
B) (2x + 4y)4
C) (4x2 + 2y2)2
D) (4x + 2y)4

170 170
SAT Quadratics and Polynomials: Polynomial Division

Quadratics and Polynomials: Polynomial Division

Identify: A question has a polynomial divided by a second algebraic equation.

Set Up: When dividing, focus on the left-most part of the divisor. This is the part of
the equation that will go into the dividend.

Example:

4 5

NO CALCULATOR NO CALCULATOR

3x – 4 12x2 – 12x + 20
The expression x + 2 is equivalent The expression = 6x –
ax + 2
to which of the following? 44
12 + ax + 2 is true for all values of
10 2
A) 3x – x + 2 x ≠ a, where a is a constant. What is

10 the value of a ?
B) 3x +
x–2
10 A) –12
C) 3 + x + 2 B) –2
C) 2
10 D) 12
D) 3 – x + 2

171 171
SAT Quadratics and Polynomials: Polynomial Factors

Quadratics and Polynomials: Polynomial Factors

Identify: A question references the factor(s) of a polynomial equation.

Set Up: For a number and/or equation to be a factor of a polynomial, it must


divide the polynomial evenly (divide the polynomial with no
remainder).

6 8

For a polynomial f(x), the value of f(5)


x −4 0 4 8
is 16. Which of the following must be
g(x) 6 2 0 8 true about f(x) ?

The function g is defined by a A) x − 5 is a factor of f(x).


polynomial. Some values of x and g(x) B) x + 5 is a factor of f(x).
are shown in the table above. Which C) x − 16 is a factor of f(x).
of the following must be a factor of D) The remainder when f(x) is divided
g(x) ? by x − 5 is 16.

A) x
B) x+4
C) x–4
D) x–8

If the function z has three distinct


zeroes, which of the following could
represent the complete graph of z in
the xy-plane?

A) C)

B) D)

172 172
SAT Quadratics and Polynomials: Polynomial Solving - Quadratic Formula

Quadratics and Polynomials: Polynomial Solving – Quadratic Formula

Identify: A question that requires finding the solutions to a polynomial equation or


asks for the zero of a polynomial function.

Set Up: If possible, convert the equation into the form ax2 + bx + c = 0 and then plug the
given information into the Quadratic Formula.
Quadratic Formula

2
–b ± b – 4ac
x=
2a

A zero (or root) of a polynomial function is a number that, when plugged in for the
variable, makes the function equal to zero.

9 10

NO CALCULATOR NO CALCULATOR

Which of the following is a zero of the x2 + gx = 4b


equation 3x2 – 15x – 42 = 0 ?
In the quadratic equation above, g
A) –4 and b are constants. What are the
B) –2 solutions for x ?
C) 0
D) 2
1 g2 +16b
A) – 2g ± 2

1 g2 + 4b
B) – 2g ± 2

1 g2 + 4b
C) 4g ± 2

1 g2 + 16b
D) 4g ± 2

173 173
SAT Quadratics and Polynomials: Polynomial Solving - Completing the Square

Quadratics and Polynomials: Polynomial Solving – Completing the


Square

Identify: A question which requires finding the solutions to a polynomial equation with
complex factors.

Set Up: Polynomials can be solved by completing the square.

Completing the square steps on the SAT:

Step 1: Divide all terms by the coefficient(s) of x2 and/or y2.

Step 2: Move the number term to the right side of the equation.

Step 3: Take half of the coefficient(s) of the variable term(s), and then square this/these
number(s). Add this/these square(s) to both sides of the equation. Convert the left-hand side to
squared form, and simplify the right-hand side.

Step 4: Rewrite the equation in the desired form (equation of a circle, vertex of a parabola, etc.).

11 12

x2 + y2 + 2x + 4y – 20 = 0 The graph of y = (8x + 8)(x – 2) is a


parabola in the xy-plane. In which of
The equation of a circle in the xy- the following equivalent equations do
plane is shown above. What is the the x- and y-coordinates of the vertex
radius of the circle? of the parabola appear as constants
or coefficients?

1
A) y = 8(x + 2)2 + (–18)

1
B) y = 8(x – 2)2 + (–18)

1 18
C) y = 4(x – 2)2 + (– 4 )

D) y = (x – 4)(x – 2)

174 174
SAT Quadratics and Polynomials: Higher-Order Equations

Quadratics and Polynomials: Higher-Order Equations

Identify: A question will involve the solution to a polynomial formula where the
highest power is greater than 2.

Set Up: If the highest power is even, let x2 = a, solve for a, then replace a with x2 and
solve to get the answers to the original equation. If the highest power is odd, factor out
an x, and then follow the steps for an even power.

A zero (or root) of a polynomial function is a number that, when plugged in for the
variable, makes the function equal to zero. A polynomial of degree n will have n roots,
some of which may be multiple roots (they repeat).

13 14

NO CALCULATOR What are the zeroes of the equation


x5 = 15x3 – 36x ?
x4 = 10x2 – 25
A) x = 0, x = ±2 2, x = ±2
For what real value of x is the
B) x = 0, x = ±2 2, x = ± 3
equation above true?
C) x = 0, x = ±2 3, x = ± 3
A) –5 D) x = 0, x = ±2, x = ±3
B) 0
C) 5
D) 5

175 175
SAT Quadratics and Polynomials Review

1 5

If x2 – y2 = 64 and x + y = 2, what is the NO CALCULATOR


value of (x – y) ?
8x – 6
The expression is equivalent to
x+ 4
which of the following?

38
A) 8 – x + 4

2 38
B) 8 –
x–6
2x2 – 3x – 2 = 0
38
C) 8 +
If a and b are two solutions of the equation x+4
above, and a > b, which of the following is 38
the value of a + b ? D) 8 +
x–6

6
3
x2 + y2 – 10x + 8y = 59
NO CALCULATOR
The equation of a circle in the xy-plane is
If x + 6 is a factor of x2 + bx + b, where b is
shown above. What is the area of the
a constant, what is the value of b ?
circle?

A) 100
B) 1,000
C) 2,500
D) 10,000

In the xy-plane, the graph of function g


has x-intercepts at −5, −2, and 2. Which of
the following could define f ?

A) g(x) = (x − 5)(x − 2)(x + 6)


B) g(x) = (x − 5)(x − 2)2
C) g(x) = (x − 2)(x + 2)(x + 5)
D) g(x) = (x + 2)2(x + 5)

176 176
SAT Analyzing Multiple Texts

SAT Reading:
Analyzing Multiple Texts

Answers on Page 294

177 177
SAT Analyzing Multiple Texts

Approaching Multiple Texts

Identifying and Analyzing Multiple Texts Questions on the SAT: One SAT reading
portion is comprised of two passages which are followed by questions that relate to both
passages.

Approaching Multiple Texts on the SAT: Multiple texts selections on SAT Reading
always share a common theme/topic, with each passage having a different
perspective on that theme/topic. A Venn Diagram can help to organize this information
for making anticipations.

Solving Multiple Texts Questions on the SAT: There are four steps to solving Multiple
Texts questions on the SAT.

1) Cover the answer choices. Do not look at the answer choices!

2) Anticipate an answer to the question. How would YOU answer the question? Use the
notes you took while reading!

3) Eliminate answers that are clearly incorrect. Answers are clearly incorrect because
they do not align with your anticipation!

4) Make a decision among whatever choices remain. Avoid answers with absolute
language and/or non-relevant information.

178 178
SAT Analyzing Multiple Texts

Analyzing Multiple Texts

Identifying and Analyzing Multiple Texts Questions on the SAT: One SAT reading
portion is comprised of two passages which are followed by questions that relate to the
passages.

Analyzing Multiple Texts Strategy: After reading each individual passage, stop and
answer only the questions that relate to that passage. After answering all questions
that relate to only one passage, then answer the questions that refer to both
passages.

Passage 1

Tired of students parading through school 1


sporting baggy scarves, low-rise jeans, and towels Which of the following aspects of the dress
on their heads, high school officials in Stamford, code issue is addressed in Passage 2 but not
Line CT, took action. They replaced their old “anything in Passage 1?
5 goes” dress code with a new policy that, much like
a skimpy outfit, doesn’t leave much to the A) Parents’ opinions
imagination. School officials maintain that the law B) Administrators’ rationale
is on the side of the school; courts have long C) Specific dress code restrictions
supported school dress codes that keep disruptive D) Students’ response
10 or offensive clothing out of classrooms. “As far as
I’m concerned, high school is a place of business. It
needs to be taken seriously,” said Stamford
superintendent, Anthony Mazzullo. “You don’t
wear hats if you’re walking into a corporate 2
15 meeting.”
Passage 2 primarily focuses on
Passage 2
A) the causes underlying the need for dress
codes.
Students at a high school in Vancouver, British
B) a violation of civil rights.
Columbia, are so befuddled by a dress code
C) the differences between American and
imposed on them by school officials that they don’t
Canadian schools’ practices.
know what to wear to school. The wide-ranging
D) the teachers and parents’ failure to
20 code – a team project of teachers and parents
communicate the dress code regulations
aimed at ending racism, drug use, and overt sexual
effectively.
references – left one student to wonder, “We’re not
allowed to wear tight clothes, and we’re not
allowed to wear loose clothes. What are we
25 supposed to wear?” Another student responded
creatively to the vagueness of the code by showing
up for classes wearing only Post-It notes, which did
not violate the wording of the school’s new dress
code.

179 179
SAT Analyzing Multiple Texts

Analyzing Multiple Texts

Identifying and Analyzing Multiple Texts Questions on the SAT: One SAT reading
portion is comprised of two passages which are followed by questions that relate to the
passages.

Analyzing Multiple Texts Strategy: After reading each individual passage, stop and
answer only the questions that relate to that passage. After answering all questions
that relate to only one passage, then answer the questions that refer to both
passages.

Passage 1

Tired of students parading through school 3


sporting baggy scarves, low-rise jeans, and towels The person quoted in lines 10-14 of Passage 1
on their heads, high school officials in Stamford, would most likely respond to the action
Line CT, took action. They replaced their old “anything described in lines 25-29 of Passage 2 by
5 goes” dress code with a new policy that, much like
a skimpy outfit, doesn’t leave much to the A) claiming such an appearance is
imagination. School officials maintain that the law inappropriate to the school environment.
is on the side of the school; courts have long B) resorting to legal action.
supported school dress codes that keep disruptive C) seeking the opinions of parents and
10 or offensive clothing out of classrooms. “As far as teachers.
I’m concerned, high school is a place of business. It D) rewarding the student’s creativity.
needs to be taken seriously,” said Stamford
superintendent, Anthony Mazzullo. “You don’t
wear hats if you’re walking into a corporate
15 meeting.”
4
Passage 2
Unlike the author of Passage 2, the author of
Students at a high school in Vancouver, British Passage 1 does which of the following?
Columbia, are so befuddled by a dress code
imposed on them by school officials that they don’t A) Offers a resolution
know what to wear to school. The wide-ranging B) Quotes an authority
C) Argues a position
20 code – a team project of teachers and parents
aimed at ending racism, drug use, and overt sexual D) Sides with the students
references – left one student to wonder, “We’re not
allowed to wear tight clothes, and we’re not
allowed to wear loose clothes. What are we
25 supposed to wear?” Another student responded
creatively to the vagueness of the code by showing
up for classes wearing only Post-It notes, which did
not violate the wording of the school’s new dress
code.

180 180
SAT Analyzing Multiple Texts Review

These passages present two perspectives on the multimillionaire beachfront property owners. The
issue of sand erosion and beach replenishment in sand replenishment project protects the real estate
California. value of residential and business properties along
45 the shoreline, but does little to profit the ocean
Passage 1 environment and the community at large.
Landlords must make a decision: pass this ballot
Responsibilities pertaining to property and land measure to obtain their sand and keep their renters,
use are agreed upon within a signed, binding or let the ballot measure fail and watch their
contract. While it is fathomable that a disagreement 50 renters dissipate like the sand.
Line with the owner of an adjacent property may occur, a
5 difficulty arises in defining the party liable for Passage 2
necessary maintenance when said disagreement is
over the deterioration of a naturally occurring, Consequentialism refers to those moral theories
communally owned parcel, such as a beach. In which hold that the consequences of a particular
California, the coastal city of Encinitas is slowly action form the basis for any valid moral judgment
10 watching its shoreline wash away. The beach has a about that action. Thus, from a consequentialist
massive sand deficit with over 100,000 cubic yards 55 standpoint, a morally right action is one that
of sand lost on an annual basis. The sand erosion is produces a good outcome, or consequence. In its
a natural occurrence, but beachfront property simplest terms, the end justifies the means. In
owners and the city government are attempting to Encinitas, California, environmentalists and city
15 fight mother nature with a sand replenishment officials agree upon the desired end, but cannot
project. This involves dredging up 2 million cubic 60 agree upon the means to bring it about. At issue is
yards of sand from the ocean bottom at a cost of $25 the sand replenishment project on the November
million. ballot. Because of sand erosion, the bluffs along the
To raise the requisite $25 million, a transient city’s coastline have been collapsing for seven
20 occupancy tax is being proposed on the November years, hurting beachgoers, with one incident
ballot. It would increase the 10% tax on motel rooms 65 resulting in death. In an attempt to resolve this
and other short-term vacation rentals to 13%, crisis, a sand replenishment project is being
dedicating the extra 3% entirely to the sand debated by the general public, but the devil is in
replenishment project. Motel and vacation rental the details. Most agree that the eroding beach
25 owners argue that if vacation renters are asked to needs to be replenished, but to what degree
pay higher fees, it will make vacation rentals 70 depends upon the negative environmental impact
unaffordable and out of reach for many tourists, one is willing to accept. It is the modus operandi of
especially in a stagnant economy. They contend that the project that could lead to varying degrees of
if the renters don’t come to Encinitas, then they’ll environmental degradation. The short-term
30 spend their money on vacations elsewhere, causing incontestable result is that further erosion of the
the regional economy to suffer as well. But it is not 75 70-foot-tall bluffs will be abated, resulting in the
clear how likely this outcome is. Vacationers who preservation of bluff-top homes above and the
are already accustomed to paying $3,000 a week in protection of pedestrians on the beach below. Aside
rent, plus a $300 tax, are unlikely to be dissuaded from the natural ebb and flow of the beachfront,
35 by an extra $90, especially considering that the the man-made causes for the beach’s erosion are
ancillary capital will keep their favorite vacation 80 coastal development, the damming of rivers, sand-
spot in pristine condition year after year. mining operations, jetty construction, harbor
The other alternative would be to share the cost dredging, and seawalls.
of the sand replenishment project through local, The city’s rejoinder is to deposit a substantial
40 state, and federal funds, but that would turn the amount of sand onto the beach once every few
project into a public welfare program for 85 years, combined with the building of artificial reefs

181 181
SAT Analyzing Multiple Texts Review

to slow down sand erosion. This solution concerns 4


environmentalists who are worried about the
movement of sand and the persistent burial of reef It can be reasonably inferred from Passage 1
Line habitat, which supports various kelps and surf that the alternative mentioned in the third
90 grass. Additionally, there are resultant adverse paragraph (lines 38-50) would
impacts on the recreationally and commercially
important invertebrates and fish that utilize those A) be widely approved.
habitats during various life stages. The B) be widely disapproved.
environmentalists’ nostrum to this predicament is C) be favored by renters but not by property
95 to eliminate the man-made causes of the beach’s owners.
erosion. But the resolution to this quandary need D) be favored by government officials but not
not be that stark. A well-designed beach by property owners.
replenishment project avoids beach fill in areas
with sensitive marine resources. The means do not 5
100 mitigate the conflict between environmentalists
and the city, but the end consequence does bring As used in line 71, the phrase “modus operandi”
minimal environmental impact upon wildlife, while refers to
sustaining a safe beach for all to enjoy.
A) the way in which the environment will be
protected.
1 B) the demise of the beaches in Encinitas.
C) the way in which the project will be carried
As used in line 28, “stagnant” most nearly means out.
D) the heavy toll such a project will take on the
A) foul. economy.
B) impure.
C) sluggish.
D) motionless. 6

The author of Passage 2 implies that the


2 movement of sand onto the beach will

Which choice does the author of Passage 1 A) allow for the creation of more natural
explicitly cite as something that influences the habitats.
behavior of vacationers? B) trivialize the danger of beach erosion.
C) not impact the environment in any way.
A) Eroded beaches D) be a better response to the problem than
B) Increased costs trying to eliminate the causes of erosion.
C) Weather
D) The national economy

7
3 Which choice provides the best evidence for the
answer to the previous question?
Which choice provides the best evidence for the
answer to the previous question? A) Lines 51-54 (“Consequentialism…action”)
B) Lines 77-82 (“Aside…seawalls”)
A) Lines 12-16 (“The sand…project”) C) Lines 83-86 (“The city’s…erosion”)
B) Lines 24-28 (“Motel…economy”) D) Lines 99-104 (“The means…enjoy”)
C) Lines 42-46 (“The sand…large”)
D) Lines 47-50 (“Landlords…sand”)

182 182
SAT Analyzing Multiple Texts Review

8 10

The passages can both be best described as In both passages, the authors indicate that
beach replenishment is
A) explanations of a problem.
B) stories of human peril. A) a struggle against nature.
C) illustrations of a commonly held belief. B) an economic crisis.
D) introductions into natural phenomena. C) a battle between government and
citizens.
D) an important civic duty.

Which statement most appropriately 11


describes a distinction between the passages?
Which of the following statements about
A) Passage 1 emphasizes the importance of beach erosion is supported by both passages?
the beach replenishment project, while
Passage 2 does not. A) The responsibility to maintain beaches
B) Passage 1 fails to include facts and lies exclusively with property owners.
figures, while Passage 2 includes such B) The problem of beach erosion will not
data. correct itself naturally.
C) Passage 1 depicts the beach C) The economic impact of beach erosion is
replenishment project as significant, minimal.
whereas Passage 2 does not. D) It is not feasible to move enough sand
D) Passage 1 is centered around the from the ocean to combat the problem.
economic impact of the beach
replenishment project, whereas Passage 2
is not.

183 183
184 184
SAT Prepositions

SAT Writing:
Prepositions

Answers on Page 297

185 185
SAT Prepositions

Prepositions

Prepositions are words that combine with a noun, noun phrase, or pronoun to show its
relationship to another word in the sentence.

Identifying Preposition Issues on the SAT: Look for a noun paired with an improper
preposition. Use your grammar ear to help determine if the preposition is correct.

The use of between and among is frequently assessed on the SAT.

• Between is used when there are two terms.


o The space between the couch and the wall. (two terms)

• Among is used when there are more than two terms .


o The pizza was divided among Alan, Bob, and Chris. (three terms)

1 3

Intense family feuds were common in As many students before him, Alan
the United States during the 19th arrived at college feeling a
century. A particularly well-known combination of nervousness and
dispute was among the Hatfield and excitement.
McCoy families.
A) NO CHANGE
A) NO CHANGE B) By
B) within C) Like
C) throughout D) DELETE the underlined portion.
D) between

Joseph and Emily knew that the


apartment was perfect with him: it
had plenty of space and was located
right by the bus stop.

A) NO CHANGE
B) with them
C) for him
D) for them

186 186
SAT Prepositions Review

1 4

When dropping a piece of sodium into Erik put a lot of effort into planning his
water, it is important to wear goggles and date with Michele, so he was relieved
stand behind a safety shield. This when she responded positively for the
protection is needed because the quiche he made for the occasion.
interaction between the two substances
can cause an explosion. A) NO CHANGE
B) in
A) NO CHANGE C) to
B) between D) with
C) within
D) OMIT the underlined part
5

2 Even though posture has little impact on


one’s ability to govern, politicians should
When a wildfire is burning, firefighters be concerned by their postures. Many
often use a technique called “controlled people will not vote for a person who
burning” to contain the fire and force it to slouches.
burn itself out. Unfortunately, controlled
burning cannot be done on land of dense A) NO CHANGE
underbrush and an overabundance of new- B) about
growth trees. C) for
D) into
A) NO CHANGE
B) where
C) with
D) that’s

There is an ongoing disagreement between


those who want to actively manage the
environment and those who believe in
humans should take a “hands off”
approach.

A) NO CHANGE
B) in the
C) that
D) that the

187 187
188 188
SAT Adjectives and Adverbs

SAT Writing:
Adjectives and Adverbs

Answers on Page 298

189 189
SAT Adjectives and Adverbs

Adjectives and Adverbs

Adjectives are words or phrases that modify (describe) nouns and pronouns.

Adverbs modify (describe) everything but nouns and pronouns (adjectives, verbs,
other adverbs).

Identifying Adverb Issues on the SAT: Look for adjectives describing words or phrases
that are not nouns or pronouns, or for adverbs describing words or phrases that are not
verbs, adjectives, or other adverbs.

1 3

Hoping for a peaceful day at the Sarah and Steve were transfixed by
aquarium, the parents beseeched their the sunset: it was deeply red and it
young son to behave mature like the stretched across the horizon.
older children.
A) NO CHANGE
A) NO CHANGE B) deep red
B) behaves mature C) so deeply red
C) behave maturely D) deeper red
D) behaving maturely

A condition experienced by many


Americans is sleep apnea: a sleep-
related breathing disorder. Sleep apnea
can cause insomnia by repeated
interrupting breathing during the night.
Pauses in breathing might only last a
few seconds and are usually not
remembered.

A) NO CHANGE
B) repeated interruption
C) repeatedly interruption of
D) repeatedly interrupting

190 190
SAT Adjectives and Adverbs Review

1 4

The apartment building, located in an area of The tortoise demonstrated that even if you
the city known for its beautiful murals, offers move very slow, with enough ingenuity you
affordable housing in a cultural diverse can still win the race.
neighborhood.
A) NO CHANGE
A) NO CHANGE B) very slower
B) cultural so diverse C) very slowly
C) cultural; diverse D) very slowest
D) culturally diverse

5
2
Jeffrey was running behind schedule, so he
Ryan tackled the problem deftly, employing had to eat fast and to dress with haste in
the skills he had learned in the armed forces. order to make it on time.

A) NO CHANGE A) NO CHANGE
B) fast B) eat fast and to dress with haste
C) with deftness C) eat quickly and dress hastily
D) with much deftness D) eat fast and to dress hastily

Hoping the family would offer him more work,


the artist painted the woman’s portrait as
meticulous as he could.

A) NO CHANGE
B) to be meticulous
C) with meticulousness
D) meticulously

191 191
192 192
SAT Circles

SAT Math:
Circles

Important Formulas
Area Formula Circumference Formula

Area = r2 Circumference = 2r

Sector Formula Arc Formula


Central Angle Central Angle
Sector = Area( ) Arc = Circumference( )
360 360

Answers on Page 301

193 193
SAT Circles: Area and Circumference

Circles: Area and Circumference

Identify: Any question that provides an area r


and/or a circumference of a circle.

Set Up: Write down the corresponding


formula(s) to find missing information
(often the radius). Repeat as necessary.
Area and Circumference Formulas
Area = r2

Circumference = 2r

1 3

A certain circle has a circumference


of 80 square inches. How long is its
radius, in inches?

In the figure above, point O is the


center of the circle, line segments AB
and BC are tangent to the circle at
points A and C, respectively, and the
2 segments intersect at point B as
shown. If the circumference of the
If circle A has an area of 144, and circle is 100, what is the length of
circle B has a circumference equal to minor arc AC ?
the length of a 180° arc of circle A,
what is the area of circle B ?

A) 12
B) 36
C) 48
D) 72

194 194
SAT Circles: Arc and Sector

Circles: Arc and Sector

Identify: Any question with


a sector, a central angle,
or an arc.

Set Up: Write down the


corresponding formula(s)
Arc Formula Sector Formula
to find missing
information. Repeat as Central Angle Central Angle
Arc = Circumference( 360 ) Sector = Area( 360 )
necessary.

4 6

In a circle with a diameter of 90


inches, what is the length, in inches,
of an arc on the circle with a central
angle measure of 60° ?

A) 15
B) 30
C) 45
D) 60 In the figure above, the circle has
center O and has radius 12. If the
length of arc NM is between 8 and 9,
not inclusive, what is one possible
integer value of x ?
5

In the circle above, segment LM is a


diameter. If the length of arc LNM is
144, what is the length of the radius
of the circle?

195 195
SAT Circles: Chords

Circles: Chords

Identify: Any question with a chord. A chord


is a line segment whose endpoints both lie on a
circle. Chord (Diameter)

Set Up: Use the Pythagorean Theorem to


solve for the missing information.

Pythagorean Theorem
a2 + b2 = c2

7 8

The area of the above circle is 256, the The circle above has a radius of r
distance from point C to point D is 12, inches, and chord CD is parallel to
and AB is the circle’s longest chord. the diameter AB. If the length of CD
What is the distance from point D to
point A ? is 37 of the length of AB, what is the
distance between the chord and the
A) 16
diameter in terms of r ? (Round to
B) 16 2
C) 20 the nearest hundredth.)
D) 20 2
A) 0.43r
B) 0.57r
C) 0.76r
D) 0.90r

196 196
SAT Circles Review

1 4
In a circle with a radius of 20 inches, what
x2 + y2 − 6x − 10y = 30
is the area of a sector of the circle with a
central angle measure of 144° ?
The equation of a circle in the xy-plane is
shown above. What is the area of the
A) 40
circle?
B) 100
C) 144
A) 46
D) 160
B) 64
C) 144
D) 286

80
In a circle with a diameter of  inches,
what is the length, in inches, of an arc of
the circle with a central angle measure of
45° ?

The diagram above shows four identical


circles with a total circumference of 16.
Assuming that each circle is tangent to two
of the other circles in the diagram, what is
the circumference of the smallest circle
that completely contains all four circles?

A) 4
B) 8
C) 4(1 + 2)
D) 32(1 + 2)

197 197
198 198
SAT Triangles

SAT Math:
Triangles

Important Formulas
Area Formula Pythagorean Theorem
1 a2 + b2 = c2
Area = bh
2

Rule of 180

a° c°

a + b + c = 180°

Answers on Page 301

199 199
SAT Triangle Basics

Triangle Basics

1. In all triangles, the biggest side is opposite the biggest angle. The same is true
for the smallest side being opposite the smallest angle.

2. The sum of the interior angles of a triangle always equals 180.

3. The sum of ANY two sides of a triangle MUST be greater than the third side.

1
4. The AREA of any triangle is 2(base)(height) .
height

base

SCALENE ISOSCELES
NO equal sides, NO equal angles TWO equal sides, TWO equal angles


The EQUAL ANGLES
are always opposite the
b° c° EQUAL SIDES

x° x°

EQUILATERAL
THREE equal sides, THREE equal angles

60°
Each angle
is always
60°

60° 60°

Similar Triangles

Similar triangles have the same angle measures, which means that corresponding side
lengths are proportional.
E

B If: Then:
b° a=d
b=e A B C
= =
c =f D E F
a° c° d° f°
A C F
D

200 200
SAT Triangles: Angles (Rule of 180)

Triangles: Angles (Rule of 180)



Identify: If you see a triangle or
intersecting lines, and you are being asked
for an angle measurement.

Set Up: Use the Rule of 180 to solve for the


unknown angle(s). a° c°

Rule of 180
a + b + c = 180°

1 2

If x = 20, and AB = BC = CD then In the figure above, side QS of


what is the value of y ? triangle QRS is on line O. What is q
in terms of z ?
A) 76o
B) 85o A) 100 – z
C) 90o B) 2z
D) 100o C) z–5
D) z+5

201 201
SAT Triangles: Area

Triangles: Area

Identify: If the area of a triangle is


referenced, you are being asked a triangle Height (h)
area question.

Set Up: Immediately write down the


triangle area formula. Plug whatever Base (b)
information is given in the question (triangle
area, base, and/or height) into the formula Triangle Area Formula
and solve for whatever is missing in the 1
equation. Repeat as necessary. Area = 2bh

3 4

What is the area of the triangle In the figure above, ΔTUV is an


shown above? equilateral triangle with sides of
length 6 and total area 9 times the
A) 18 area of equilateral triangle XUY.
B) 18 2 What is the area of trapezoid VWXY ?
C) 36
D) 36 2 A) 3
B) 6 3
C) 12
D) 8 3

202 202
SAT Triangles: Side Lengths (Pythagorean Theorem)

Triangles: Side Lengths


(Pythagorean Theorem)

Identify: If you are solving for the side of a


c
right triangle or need to find the length of b
any diagonal line, you are being asked a right
triangle side length question.

Set Up: Immediately write down the a


Pythagorean Theorem. Plug whatever
information is given in the question (hypotenuse Pythagorean Theorem
and/or legs) into the formula and solve for
whatever is missing in the equation. Repeat as a2 + b2 = c2
necessary.

5 6

In the right triangle above, if y = 12, In pentagon ABCDE above, what is the
what is the value of x? value of y ?

A) 5 A) 57
B) 9 B) 73
C) 13 C) 87
D) 15 D) 95

203 203
SAT Triangles: Special Right Triangles

Triangles: Special Right Triangles

Identify: If you see a right triangle with the angles 30º-60º-90º or 45º-45º-90º.

Set Up: Use special right triangle ratios to solve the problem.

30º-60º-90º 45º-45º-90º

60° 2x 45°
x 2
x x
30° 45°
x 3 x

7 8

In the figure above, RSTV is a square


and TU = 6. What is the area of RSTV ?

In quadrilateral MNOP above, the


bisector of angle MNO is
perpendicular to MO at point Q. If
QM = 10 and PQ = 10, what is the
measure of angle MPQ ?

A) 30º
B) 45º
C) 60º
D) 90º

204 204
SAT Triangles: Similar Triangles

Triangles: Similar Triangles

Identify: If you see triangles with similar angle measures.

Set Up: If triangles have similar angle measures, then their side lengths are
proportional.
.

d° If: Then: And:


a° a=d
B E A B C
C F b=e = = ∆ABC ~ ∆DEF
D E F
c =f
b° c°
e° f°
A
D
9 10

NO CALCULATOR NO CALCULATOR

In the circle shown above, E is the In the figure above, if the legs of
center and lies on AD and CB. Which triangle EFG are parallel to the axes,
of the following statements is true? which of the following could be the
lengths of the sides of ΔEFG ?
A) AEB measures 54°
B) AB is parallel to CD A) 4, 8, and 4 5
C) AB ≈ BE B) 4, 6, and 2 13
D) CD ≈ AE C) 6, 8, and 10
D) 6, 8, and 14

205 205
SAT Triangles Review

1 3

A right triangle has legs of lengths f and g


and a hypotenuse of length h. If f 2 + g2 + h2
= 578, what is the value of h ?

In the polygon above, triangle ABC is an


isosceles right triangle, and the triangle has
an area of 72 square inches. What is the
area of square ACDE, in inches?

2 NO CALCULATOR

In the figure above ∆BCA ~ ∆DCE. Which of


A golfer wants to find a length, x, in feet, of a the following must be true?
sand trap as represented in the sketch
above. The lengths represented by AC, BC, A) AE | DE
DC, and AB on the sketch were measured to B) AE | BA
be 500 feet, 400 feet, 200 feet, and 150 feet, C) AB ∥ DE
respectively. Segments AE and BD intersect D) AE ∥ BD
at C, and ∠CDE and ∠CBA have the same
measure. What is the value of x ?

206 206
SAT Quadrilaterals and Polygons

SAT Math:
Quadrilaterals and
Polygons
Important Formulas
Rhombuses
Area Formula Perimeter Formula Angle Relationship
Area = bh Perimeter = 2b + 2h x° y°
(In a square, b and h are equal) (In a square, b and h are equal)
x + y = 180°
y° x°

Trapezoids
Area Formula Angle Relationship
b1 + b2 x°
Area = ( 2 )h
x + y = 180°

Polygons
Sum of Interior Angles = 180° (n – 2)
n = number of sides

Answers on Page 301

207 207
SAT Quadrilaterals and Polygons: Rhombus Questions

Quadrilaterals and Polygons: Rhombus Questions

Identify: A question references the area, angles, and/or perimeter of a non-trapezoid


quadrilateral.

Set Up: Write down the appropriate formula(s) and/or relationship(s) to find the
missing information.

Area Formula Perimeter Formula Angle Relationship


Area = bh Perimeter = 2b + 2h x° y°
(In a square, b and h are equal) (In a square, b and h are equal)
x + y = 180°
y° x°

1 3

In the diagram above, ΔABE is an


In the above parallelogram, if x = 80,
isosceles right triangle with an area
what is y + 120 ?
of 2. If the sides of the parallelogram
BCDE all have equal length, what is
the area of BCDE ?

2 A) 2 2
B) 4
C) 4 2
D) 8

Erin puts a fence around her


rectangular garden. The garden has
In the figure above, if the area of
a width that is two feet more than
triangle OPL is equal to the area of
four times its length. What is the
the rectangle LMNO, what is the
perimeter of Erin’s fence if the area
length of PN ?
of her garden is 156 square feet?

208 208
SAT Quadrilaterals and Polygons: Trapezoid Questions

Quadrilaterals and Polygons: Trapezoid Questions

Identify: Any question that references the area and/or angles of a trapezoid.

Set Up: Write down the appropriate formula and/or relationship to find the missing
information.

Area Formula Angle Relationship


b1 + b2 x°
Area = ( 2 )h
x + y = 180°

5 7

What is the area of a trapezoid with NO CALCULATOR


bases of length 10 and 20 and a height
of 15 ?

In quadrilateral ABCD above, BC is


parallel to AD, and AB = CD. If BC
6
and AD were each halved and BE was
increased by 300 percent, how would
the area of ABCD change?

A) The area of ABCD would be decreased


by 50 percent.
B) The area of ABCD would not change.
C) The area of ABCD would be
In the above trapezoid, if x = 100,
multiplied by 2.
what is 4y + 50 ?
D) The area of ABCD would be
multiplied by 3.

209 209
SAT Quadrilaterals and Polygons: Polygons

Quadrilaterals and Polygons: Polygons

Identify: Look for regular shapes with more than four sides. Regular shapes have
sides that are all equal and interior (inside) angles that are all equal.

Set Up: For area problems, try to break the polygon into smaller figures such as
triangles. For angle problems, use the Sum of Interior Angles formula.

Sum of Interior Angles Formula


Sum of Interior Angles = 180° (n – 2)
n = number of sides

8 10

In the figure above, what is the sum


of m and n ?

In the figure above, five identical


triangles each share a side with a
9 regular pentagon. The other two
sides of each triangle are equal. If
the area of one of the triangles is 48
square units, and the area of
pentagon ABCDE is 240 square
units, what is the smallest angle
measure of one of the triangles?

The figure above shows a regular A) 48º


hexagon with sides of length z and an B) 50º
equilateral triangle with sides of C) 54º
length d. If the area of the hexagon is D) 60º
18 3 square inches, what is the
height, in inches, of the triangle?

210 210
SAT Polygons and Quadrilaterals Review

1 5

What is the height of a trapezoid with


bases of length 4 and 8 and an area of 60 ?
J
x
(1, 0)

L
2 K

The length of a rectangle is three times its


width. The area of the rectangle is 48 y
square inches. What is the width, in
In the figure above, JK = KL and the
inches, of the rectangle?
coordinates of K are (b, –5). What is the
value of b ?

A) 5
B) 5
C) 0
D) –5
3
The lengths of two opposing sides of a
square are decreased by 40 percent. By
6
what percent would the lengths of the
other two sides have to be increased so
that the area of the new figure (a
rectangle) is the same as the area of the
original square? (Round to the nearest
tenth.)

In the figure above, ABCDEFGH is a


regular octagon whose sides are each 8
4 units long. A and E are the centers of two
large circles, and BC, CD, FG, and GH are
20 diameters of the four small semicircles.
What is the total length of the darkened
outline of the figure?
18
A) 32
B) 36
C) 40
What is the fewest number of rectangles, D) 48
each measuring 9-by-4, that is needed to
completely cover, without overlap, the 20-
by-18 rectangle shown above?

211 211
212 212
SAT Lines and Angles

SAT Math:
Lines and Angles

Important Formulas
Rule of 180


x° z°

x + y + z = 180°

Answers on Page 302

213 213
SAT Intersecting Lines

Lines and Angles: Intersecting


Lines f°
m
Identify: A question has two parallel lines
intersected by a third line.

Set Up: For parallel lines intersected by a n
third line, the formed acute angles are
congruent, the formed obtuse angles are
congruent, and acute and obtuse angles
are supplementary (they add to 180 If m || n, then f + g = 180°
degrees).

1 2

NO CALCULATOR

In the figure above, x║y and b = 45°.


What is the value of 3a ?

In the figure above, l║m. Which of the


following must equal 180 ?

A) k+n+r
B) k+p+s
C) n+p+s
D) n+p+t

214 214
SAT Rule of 180

Lines and Angles: Rule of 180

Identify: If you see angles on a straight


line, and you are being asked for an angle
measurement. x°

Set Up: Use the Rule of 180 to solve for the


unknown information. Rule of 180
x + y + z = 180°

3 4

NO CALCULATOR

In the figure above, XWY measures


10°, WYZ measures 125°, and points
X, Y, and Z are collinear. What is the What is the sum, in terms of x, of the
measure of WXY ? degree measures of the two angles
indicated with arrows in the figure
above?

A) x
B) 4x
C) 180 – x
D) 360 – 4x

215 215
SAT Lines and Angles Review

1 3

Points K, L, and M are on line ℓ in that


NO CALCULATOR
order. N is a point not on line ℓ. The
measure of angle KLN is (3x + 3)º and the
measure of angle NLM is (5x – 7)º. What is
the value of x ?

A) 21.5
B) 22
C) 23
D) 23.5

In the figure above, lines a, b, and c


intersect at a point. If j + k = x + y, which
of the following must be true?

I. j = l
2 II. k = z
III. l = z

A) I and II only
B) I and III only
C) II and III only
D) I, II, and III

In the figure above, if r is parallel to s and


b = 110, then b + d =

A) 140
B) 180
C) 220
D) Cannot be determined

216 216
SAT Word Choice

SAT Writing:
Word Choice

Answers on Page 298

217 217
SAT Word Choice: Frequently Confused Words

Word Choice: Frequently Confused Words

An error on the SAT is to use a word that sounds similar to a word that works in the
context of the sentence, but has a different meaning.

Identifying Frequently Confused Words Issues on the SAT: Look for words that have
homophones which may confuse your grammar ear.

Examples:

Accept vs. Except


• Accept – To receive; to agree or consent
• Except – To take out; to leave out

Affect vs. Effect


• Affect – To influence; to change (verb)
• Effect – A result (noun)

Farther vs. Further


• Farther – Longer (physical distance)
• Further – More; additional; extended (non-physical distance)

1 3

With the increased risk resulting from A noteworthy effect of the Supreme
easily accessible digital resources, Court’s ruling is that the government
schools have required students to be can no longer restrict the amount of
more diligent in siting their sources. money companies donate to Political
Action Committees.
A) NO CHANGE
B) siting his or her sources A) NO CHANGE
C) citing their sources B) An important affect
D) citing his or her sources C) One effect significantly
D) One significant affect
2

Recently, wearing white after labor day


has become more socially excepted
than in years past.

A) NO CHANGE
B) excepted then
C) accepted than
D) accepted then

218 218
SAT Word Choice: Inconsistent Tone and/or Style

Word Choice: Inconsistent Tone and/or Style

An error on the SAT is to use diction/tone/style that is inconsistent with the


surrounding text.

Identifying Inconsistent Tone and/or Style Issues on the SAT: Inconsistencies can
include the use of vague language, overly informal language, colloquialisms, or jargon
(vocabulary of a particular group).

4 6

A big reason for the bill’s passage Although he was supposed to be


was the fear that the invasive species preparing for his oral presentation on
would soon overrun the wetland. the workings of the antecubital fossa,
Tom was temporarily enraptured by
A) NO CHANGE the medial epicondyle and
B) main thing causing the Pronator Teres.
C) huge thing about
D) primary cause of A) NO CHANGE
B) human body
C) deep structures
D) knee
5

While most people agree that


building a new bridge will bring
about fewer traffic headaches, there
is stark disagreement about how to
fund the construction project.

A) NO CHANGE
B) slash traffic delays
C) lead to plummeting commute
times
D) mitigate traffic issues

219 219
SAT Word Choice Review

1 4

After the great winter storm of 1992, While historically the Chinese economy
sidewalks and roadways were slick for a has not had an affect on Western stock
prolonged stretch of time. The newscasters prices, increasing globalization makes this
on all stations warned viewers to precede no longer the case.
with caution.
A) NO CHANGE
A) NO CHANGE B) significant effect
B) precede with caution C) effect
C) proceed in caution D) affected
D) proceed with caution

5
2
Upon arriving at the scene, the firefighter
Even though Americans may hold their found the residences engulfed in flames
noses at lobbyists, interest groups have and the surrounding homes at imminent
considerable influence over public policy. danger.

A) NO CHANGE A) NO CHANGE
B) profoundly distaste B) in imminent
C) dislike C) at eminent
D) unanimously oppose D) in eminent

Considering the company’s declining


profits, it is surprising that more people
are not ticked off at the CEO for taking a
vacation.

A) NO CHANGE
B) shouting out
C) expressing their disgust
D) upset

220 220
SAT Statistics

SAT Math:
Statistics

Important Formulas
Exponential Growth Exponential Growth
(Non-Doubling) (Doubling) Exponential Decay
x
y = C(1 + r)t y = a(2) b y = C(1 – r)t

y = Final Amount y = Final Amount y = Final Amount


C = Initial amount x = Duration C = Initial amount
r = Rate of Change a = Original Amount r = Rate of Change
t = Time b = Doubling Time t = Time

Answers on Page 302

221 221
SAT Statistics: Exponential Growth and Exponential Decay

Statistics: Exponential Growth and Exponential Decay

Identify: A question which references exponential growth (increase in size at a


constantly growing rate) and/or exponential decay (decrease in proportion to the
size of the population).

Set Up: Write down the exponential growth and/or exponential decay formula and
plug in the given information to solve for the unknown quantity.
Exponential Growth (Non-Doubling) Exponential Growth (Doubling) Exponential Decay
x
y = C(1 + r)t y = a(2) b y = C(1 – r)t
y = Final Amount y = Final Amount y = Final Amount
C = Initial amount x = Duration C = Initial amount
r = Rate of Change a = Original Amount r = Rate of Change
t = Time b = Doubling Time t = Time

1 3

The stock price of one share in a Emily opened a savings account at a


certain company is worth $750 today. local bank on January 1, 2005. The
A stock analyst believes that the stock table below gives the total value of
will lose 35 percent of its value each Emily’s account over time.
month for the next seven months. The
analyst uses the equation P = 360(x)m Date Account Value
to model the value, P, of the stock after January 1, 2005 $1,000
m months. What value should the January 1, 2006 $1,050
analyst use for x ? January 1, 2007 $1,102.50
January 1, 2008 $1,157.63
January 1, 2009 $1,215.51
2 January 1, 2010 $1,276.28
A researcher at a local college noticed January 1, 2011 $1,340.10
that a local population of algae spores
doubles in size every 4 months. The Which of the following best describes
population at the beginning of January the relationship between time and the
2015 was estimated to be 100 spores. If value of Emily’s savings account?
P represents the population n months
after January, then which of the A) Increasing linear
following equations represents the B) Decreasing linear
model of the population over time? C) Exponential growth
D) Exponential decay
A) P = 4 + 100n
B) P = 100 + 4n
C) P = 100(2)4n
n
D) P = 100(2) 4

222 222
SAT Statistics: Interpreting Charts and Graphs

Statistics: Interpreting Charts and Graphs

Identify: The question will describe a survey, poll, experiment, or other situation,
using a chart, table, and/or graph.

Set Up: Review the titles of any tables, and the headings of any axes. Make sure that
you are using data from the correct location(s).

4 5

10 1000

Stock Price
Population

900
800
5
700
600
0 500
A B C D E F G
Town

The population of seven towns is Day


shown in the graph above. If the total
population is 37,500,000 what is an Based on the graph above, which of
appropriate label for the vertical axis the following best describes the
of the graph? general trend in the stock price from
Sunday through Saturday?
A) Population (in tens of thousands)
B) Population (in hundreds of A) Sales increased each day from
thousands) Sunday to Saturday.
C) Population (in millions) B) Sales decreased each day from
D) Population (in tens of millions) Sunday to Saturday.
C) Sales increased from Sunday to
Tuesday and then generally
decreased.
D) Sales generally remained steady
from Sunday to Saturday.

223 223
SAT Statistics: Line of Best Fit

Statistics: Line of Best Fit

Identify: A question will reference the line of best fit for a given collection of data
(usually arranged in a scatter plot).

Set Up: The line of best fit is a line on a graph that shows the general direction that a
group of points seems to be heading. The slope (rise over run) of a line of best fit
describes the trend of the collected data.

Questions 6, 7, and 8 relate to the below data 7

Which of the following is the best


1600
1500 interpretation of the slope of the line
1400 of best fit in the context of this
SAT Score

1300 problem?
1200
1100
1000 A) The predicted average increase in
900 SAT score for every day increase of
800
12 14 16 18 20 22 24 26 28 30
studying.
B) The predicted increase in days
Days Studied
studied for each additional average
point scored on the SAT.
Ben works as a SAT tutor on
C) The predicted number of days
weekends during the summer.
studied for a student who receives
The scatterplot above shows the
a score of 850.
relationship between the average SAT
D) The predicted average SAT score
score and the total number of days
for a student who studied for 12
studied for each of Ben’s students.
days.
The line of best fit is also shown.

8
Based on the line of best fit to the
6
data shown, which of the following
Based on the line of best fit to the values is closest to the average
data shown, which of the following is increase in SAT score for each
closest to the average SAT score for a additional day studied?
student who studied for 21 days?
A) 5
A) 1050 B) 15
B) 1150 C) 30
C) 1250 D) 50
D) 1350

224 224
SAT Statistics: Surveys, Research, and Margin of Error

Statistics: Surveys, Research, and Margin of Error

Identify: A question will reference some form of data collection and/or the terms
“margin of error,” “sample/sample error,” or “association.”

Set Up: To decrease the margin of error (the amount of random sampling error) or increase
the association (the strength of the connection between different data points) in the results
of a survey, additional information must align with the population in the original
survey.

9 10

A researcher wanted to know if there is From a large population of people with


an association between scores on poor hearing, 500 participants were
standardized exams and preparation selected at random. Half of the
time for the population of high school participants were randomly assigned to
Juniors in Wyoming. She obtained receive treatment H, and the other half
survey responses from a random did not receive treatment H. The
sample of 500 Wyoming high school resulting data showed that
Juniors and found convincing evidence participants who received treatment H
of a positive association between had significantly improved hearing as
standardized test scores and compared to those who did not receive
preparation time. Another researcher treatment H. Based on the design and
intends to replicate the exact same results of the study, which of the
survey and will attempt to get a following is an appropriate conclusion?
smaller margin of error. Which of the
following samples will most likely A) Treatment H is likely to improve
result in a smaller margin of error for the hearing of people who have poor
the survey? hearing.
B) Treatment H improves hearing
A) 2,500 randomly selected high school better than all other available
juniors from all regions of the treatments.
United States. C) Treatment H will improve the
B) 2,500 randomly selected students hearing of anyone who takes it.
from across all high school grades. D) Treatment H will cause a
C) 750 randomly selected high school substantial improvement in
juniors from Wyoming. hearing.
D) 750 randomly selected high school
juniors from across the world.

225 225
SAT Statistics: Standard Deviation

Statistics: Standard Deviation

Identify: The question or the answers will reference the term “standard deviation.”

Set Up: In a data set, a small standard deviation means that the values in the data set are,
on average, close to the mean of the data set, and a large standard deviation means that the
values in the data set are, on average, further away from the mean.

11 12

Survey A Survey B Class A Class B


Weight Frequency Weight Frequency Height Frequency Height Frequency

160 lbs. 4 160 lbs. 14 64 inches 13 64 inches 2

170 lbs. 8 170 lbs. 4 65 inches 4 65 inches 2

180 lbs. 12 180 lbs. 2 66 inches 3 66 inches 3

190 lbs. 7 190 lbs. 3 67 inches 3 67 inches 13

200 lbs. 5 200 lbs. 15 68 inches 14 68 inches 2

Which of the following is true about the Which of the following is true about the
data shown for these two surveys? data shown for these two classes?

A) The standard deviation of weights in A) The standard deviation of heights in


City A is larger. City A is larger.
B) The standard deviation of weights in B) The standard deviation of heights in
City B is larger. City B is larger.
C) The standard deviation of weights in C) The standard deviation of heights in
City A is the same as that of City B. City A is the same as that of City B.
D) The standard deviation of weights in D) The standard deviation of heights in
these cities cannot be calculated with these cities cannot be calculated with
the data provided. the data provided.

226 226
SAT Statistics Review

1
Questions 3 and 4 relate to the
Number of Horseshoe Crabs

35
30 following information.
25
20 Chelsea runs Chelsea’s Freezee Popsicle
15 Emporium, which sells Freezee Pops
10 during the summertime. The scatterplot
5 below shows the relationship between the
0 temperature, in degrees Fahrenheit, and
the number of Pops sold on a given day
last summer.

Day
120
The line graph above shows the number of 100
horseshoe crabs counted along the ocean
80

Pops Sold
during a given week in Capital City.
60
According to the graph, what was the
greatest decrease in the number of 40
horseshoe crabs counted between two 20
consecutive days? 0
80 82 84 86 88 90 92 94
A) 10 crabs
Temperature (Degrees Fahrenheit)
B) 12 crabs
C) 13 crabs
D) 14 crabs 3

2
Based on the line of best fit for the data
(not shown), which of the following is
closest to the number of Pops sold on a day
800
Total Packages Sold

that is 85 degrees Fahrenheit?


600
A) 55
400
B) 65
200 C) 80
D) 90
0
1 2 3 4 5 6 7
Week
4
Eric recently began working for a tutoring Which linear equation most accurately
company. The graph above shows the total represents the line of best fit for this data
number of tutoring packages that Eric has (not shown)?
sold since he started at the tutoring
company. During his first seven weeks at A) y = 5x – 300
the company, Eric took a week-long B) y = 5x
vacation. Based on the graph, which of the C) y = –5x + 80
following is closest to when Eric took his D) y = –5x
vacation?

A) Immediately after week 1


B) Immediately after week 2
C) Immediately after week 3
D) Immediately after week 4

227 227
SAT Statistics Review

5 7

A radioactive substance decays at an Which scatterplot shows a positive


annual rate of 27 percent. If the initial association that is linear? (Note: A positive
amount of the substance is 550 grams, association between two variables is one in
which of the following functions f models which higher values of one variable
the remaining amount of the substance, in correspond to higher values of the other
grams, b years later? variable, and vice versa.)

A) f(b) = 550(0.73)b A)
B) f(b) = 550(0.27)b
C) f(b) = 0.73(550)b
D) f(b) = 0.27(550)b

B)

A researcher conducted a survey to


determine whether people in a certain city
C)
prefer going to the movies to reading
books. The researcher asked 136 people
who visited a movie theater on a Tuesday,
and 9 people refused to respond. Which of
the following factors makes it least likely
that a reliable conclusion can be drawn
about the movie-watching preferences of
all people in the town?
D)
A) Sample size
B) Population size
C) The number of people who refused to
respond
D) Where the survey was given

228 228
SAT Graphing

SAT Math:
Graphing

Important Formulas

Circle Graphing Formula


Circle Graphing Formula: (x – h)2 + (y – k)2 = r2

Center: (h, k)
Radius: r

Parabola Graphing Formulas

Graphing Formula: y = ax2 + bx + c


Vertex form: y = a(x – h)2 + k, where (h, k) is the vertex

If: a > 0; parabola opens up (u-shaped). If: a < 0; parabola opens down (n-shaped)
b
x-coordinate of the vertex (highest or lowest point of the curve): –2a
A parabola’s “axis of symmetry” passes vertically through the vertex.

Answers on Page 302

229 229
SAT Graphing: Circles

Graphing: Circles
(x, y)

r
Identify: Look for questions that reference the
(h, k)
graphing formula of a circle.
x
Set Up: Plug in the elements from the question
into the circle equation.
Circle Graphing Equation
Circle Equation: (x – h)2 + (y – k)2 = r2

Center: (h, k)
Radius: r

1 3

NO CALCULATOR NO CALCULATOR

What is the diameter of a circle with Which of the following is an equation


the below equation? of a circle in the xy-plane with center
(3, 2) and a radius with endpoint
(x – 4)2 + (y + 5)2 – 121 = 0 (–2, –10) ?

A) (x – 3)2 + (y – 2)2 = 132


B) (x – 2)2 + (y – 3)2 = 132
C) (x + 2)2 + y2 = 13
2
D) x2 + (y + 10)2 = 13
NO CALCULATOR

Dale Street Park recently installed a


new circular merry-go-round. The
merry-go-round has a radius of 288
inches.

Which of the following is the


equation of the circle, in inches,
formed by the outside of the merry-
go-round?

A) x2 + y2 = 24
B) (x + y)2 = 24
C) x2 + y2 = 2882
D) x + y = 2882

230 230
SAT Graphing: Parabolas

Graphing: Parabolas (h, k)

Identify: Look for questions that have a


diagram of a parabola or reference the
graphing formula(s) of a parabola. x

Set Up: Solve the question by using the


relevant parabola equation(s).
Parabola Graphing Equations
Parabola Equation: y = ax2 + bx + c

Parabola Vertex Form: y = a(x – h)2 + k

Vertex: (h, k)

4 5

The graph of y = 2x2 – 4x – 48 is a


parabola in the xy-plane. In which of
the following equivalent equations do
the x- and y-coordinates of the vertex
of the parabola appear as constants or
coefficients?

A) y = 2x2 + 4x + 48
B) y = 2(x – 2) – 48 Which of the following is an
C) y = 2(x – 1)2 + (–50) equivalent form of the equation of the
D) y = (x – 2)(2x – 4) graph shown in the xy-plane above,
from which the coordinates of the
vertex A can be identified as
constants in the equation.

A) y = (x + 2)2 + 2x
B) y = –(x + 2)2 + 2
C) y = (x – 2)2 + 2
D) y = –(x – 2)2 + 2

231 231
SAT Graphing Review

1 3

NO CALCULATOR y = a(x – 6)(x – 10)

What is the diameter of the circle In the quadratic equation above, a is a


represented by the below equation? nonzero constant. The graph of the
equation in the xy-plane is a parabola with
(x + 4)2 + (y + 8)2 = 902 vertex (j, k).Which of the following is equal
to k ?

A) –4a
B) –2a
C) 6a
D) 10a

NO CALCULATOR

Which of the following is an equation of a


circle in the xy-plane with center (0, 8) and
4
a radius with endpoint (4, 14)?

A) (x – 4)2 + (y – 14)2 = 522 The graph of y = (3x + 6) (x – 6) is a


B) x2 + (y – 8)2 = 522 parabola in the xy-plane. In which of the
C) (x – 8)2 + y2 = 52 following equivalent equations do the x-
D) x2 + (y – 8)2 = 52 and y-coordinates of the vertex of the
parabola appear as constants or
coefficients?

A) y = (x – 3)3 + (–48)

48
B) y = 3(x – 3)2 + (– )
4

C) y = 3(x – 2)2 + (–48)

D) y = (x + 6)(3x – 6)

232 232
SAT Interpreting Equations

SAT Math:
Interpreting Equations

Answers on Page 302

233 233
SAT Interpreting Equations: Linear Equations

Interpreting Equations: Linear Equations

Identify: A question asks about an equation model that can be written in the form
y = mx + b.

Set Up: Identify the units represented by x and y (the answer must relate to these
units) and then plug numbers in for x to identify trends in the data.

1 2

NO CALCULATOR NO CALCULATOR

P = 12 + 2.5s l = 2,000 + 40d

The equation above is used to model A city is running a cash lottery.


the relationship between the Each time no winning number
percentage of households that watch a combination is drawn, additional
television show, P, and the length of money is entered into the lottery
the time the show has been on, s, in pool. The equation above can be
seasons. According to the model, what used to model the size of the lottery,
is the meaning of the 2.5 in the l, in dollars, after d drawings with
equation? no winning number combinations.
What does it mean that (2, 2,080) is
A) For every additional season that a solution to this equation?
the show is on, the number of
households watching the television A) During the lottery, 40 dollars
show increases by 2.5 percentage are given away each drawing.
points. B) Each winning lottery jackpot is
B) For every additional 2.5 seasons at least 2,080 dollars.
the show is on, the number of C) The lottery will last for 40
households watching the television drawings.
show increases by 12 percentage D) The lottery jackpot after the
points. second drawing in 2,080 dollars.
C) 2.5 percent of households watched
the first season of the television
show.
D) 2.5 percent of households watched
the first season of the television
show and 12 percent of people
watched the second season of the
television show.

234 234
SAT Interpreting Equations: Non-Linear Equations

Interpreting Equations: Non-Linear Equations

Identify: An equation model that CANNOT be written in the form y = mx + b.

Set Up: Translate the information in question into a formula on your own and
then compare your equation to the one given. If there are two things performing a
task at rates that are different magnitudes (one thing is twice as fast as the other, for
example), then the faster thing is represented by a variable and the slower thing is
represented by a multiple of that variable.

3 4

NO CALCULATOR NO CALCULATOR

1 1 1 1 1 1
12 + 8 = t 12 + 8 = t

John is currently using two computers John is currently using two computers
to help him solve a complicated to help him solve a complicated
equation. One of the computers can equation. One of the computers can
solve the problem in twelve hours and solve the problem in twelve hours and
the other computer can solve the the other computer can solve the
problem in eight hours. The equation problem in eight hours. The equation
above describes the situation. Which above describes the situation. Which of
1 1
of the following describes what 12 the following describes what t
represents in this equation? represents in this equation?

A) The portion of the solving A) The portion of the solving completed


completed by the slower computer. by the faster computer.
B) The portion of the solving B) The portion of the solving completed
completed by the faster computer. by both computers in one hour.
C) The portion of the solving C) The portion of the solving completed
completed by the slower computer by the faster computer in one hour.
in one hour. D) The portion of the solving completed
D) The portion of the solving by the slower computer in one hour.
completed by the faster computer
in one hour.

235 235
SAT Interpreting Equations Review

1 3

NO CALCULATOR NO CALCULATOR

Emily receives a bonus from her company Timothy shovels driveways during the
each time she sells a convertible. This winter. If Timothy’s payment for each
bonus is added to a flat annual salary driveway shoveled is modeled by the
which Emily receives regardless of the equation P = 20 + 10h , where h represents
number of convertibles she sells. Emily’s time spent shoveling, in hours, which of
overall income is modeled by the equation the following statements is true?
I = 750c + 30,000, where c represents the
number of convertibles that Emily sells. A) For every increase in shoveling of ten
hours, Timothy receives 10 more
Which of the following statements is true? dollars in payment.
B) For every increase in shoveling of ten
A) For every convertible that Emily sells, hours, Timothy receives 10 less dollars
she receives an additional 30,000 in payment.
dollars. C) For every increase in shoveling of one
B) Emily must sell 750 convertibles to hour, Timothy receives 10 more dollars
receive a bonus. in payment.
C) Emily will receive a minimum of 750 D) For every increase in shoveling of one
dollars. hour, Timothy receives 10 less dollars
D) For every convertible Emily sells, she in payment.
will receive a bonus of 750 dollars.

NO CALCULATOR
2
1 2 1
+ =
NO CALCULATOR x x 5

A moving company estimates the price of a John and Jim are working together to
move, in dollars, using the expression 500 move a pile of mulch onto their mother’s
+ 20ab, where a is the number of movers flower bed. John can shovel half as fast as
who will be working and b is the total Jim, and together they can complete the
number of hours the move will take using job in 5 hours. The equation above
a builders. Which of the following is the represents the situation described. Which
best interpretation of the number 20 in the of the following describes what the
1
expression? expression x represents in the equation?

A) A minimum of 20 builders will work on A) The time, in hours, it takes Jim to


each job. complete the job alone.
B) The price of every job increases by $20 B) The portion of the job that John would
every hour. complete in one hour.
C) Each builder works 20 hours a day. C) The portion of the job that Jim would
D) The company charges $20 per hour for complete in one hour.
each builder. D) The portion of the job that John would
complete in two hours.

236 236
SAT Advanced Grammar

SAT Writing:
Advanced Grammar

Answers on Page 298

237 237
SAT Advanced Grammar: Affect vs. Effect

Advanced Grammar: Affect vs. Effect

An error on the SAT is to misuse the words affect and effect

Identifying Affect vs. Effect Issues on the SAT: Looks for the word affect or effect
underlined in a sentence.

Affect means “to produce a change” or “to act in a way you do not feel” (used as a verb).
• The weather affected Jeremy’s plans.

Effect means “a result” or “a change that occurred” (used as a noun).


• The weather had no effect on Jeremy’s plans.

When you AFFECT a situation you have an EFFECT on it!

1 3

Despite the potential affect that the When Tom Brady was injured in 2007,
weather would have on their trip, Eric many speculated that this injury would
and Nathan went ahead with their adversely effect his career. However,
plan to hike the Grand Canyon. Brady won the MVP award in 2010
and a fourth championship in 2015.
A) NO CHANGE
B) affect of A) NO CHANGE
C) effect that B) adversely affect
D) effect of C) adverse effect
D) adverse affect

The plummeting oil prices have had a


significant affect on the nation’s
economy.

A) NO CHANGE
B) effect on
C) affecting on
D) effect

238 238
SAT Advanced Grammar: Parallel Comparisons

Advanced Grammar: Parallel Comparisons

An error on the SAT is to set up a comparison (words linked by like, than, or as)
where the items being compared are not of the same type.

Identifying Parallel Comparison Issues on the SAT: Look for a comparison being
made in the sentence. Are the items of the same type?

4 6

As an avid mystery reader, Patrick felt The fur coats of dachshunds, often
that novels by John Grisham were smoother and shorter than wheaten
more enjoyable than other authors. terriers, make dachshunds preferable
to pet owners with allergies.
A) NO CHANGE
B) enjoyable, more so than other A) NO CHANGE
authors. B) The fur coats of dachshunds, which
C) more enjoyable than books by other are often smoother and shorter than
authors. wheaten terriers
D) more enjoyable, particularly when C) The fur coats of dachshunds, often
compared to other authors. smoother and shorter than that of
wheaten terriers
D) The fur coats of dachshunds, often
5 smoother and shorter than those of
wheaten terriers
After her company had relocated her,
Dana was disappointed to learn that
owning a car in Japan is more
expensive than the United States.

A) NO CHANGE
B) cars in the United States
C) owning a car in the United States
D) owning a car

239 239
SAT Advanced Grammar: Misplaced Modifiers

Advanced Grammar: Misplaced Modifiers

An error on the SAT is to have a descriptive word or phrase that is NOT next to what
it is describing.

Identifying Misplaced Modifier Issues on the SAT: Look for a descriptive phrase
that is next to a comma. What is being described (the subject of the sentence) must come
directly after the comma.

7 9

In addition to receiving an initial Excluded from almost every recent


payment for her novel, royalties will be major poetry anthology, colleagues
earned by the author on every copy of have unjustly forgotten Edna St.
her book that is sold. Vincent Millay and academics despise
her.
A) NO CHANGE
B) royalties will be received by the A) NO CHANGE
author B) Edna St. Vincent Millay has been
C) the author will have the ability to unjustly forgotten by colleagues
receive royalties C) It is unjust how Edna St. Vincent
D) the author will earn royalties Millay has been forgotten by
colleagues
D) Edna St. Vincent Millay, unjustly
forgotten by colleagues,
8

They know it as Tommy John Surgery,


the procedure has significant
repercussions for pitchers unlucky
enough to tear their elbow ligaments.

A) NO CHANGE
B) Known as Tommy John Surgery,
C) It is known as Tommy John
Surgery,
D) Tommy John Surgery,

240 240
SAT Advanced Grammar: Noun Agreement

Advanced Grammar: Noun Agreement

An error on the SAT is to have a singular noun representing another plural noun, or
a plural noun representing another singular noun.

Identifying Noun Agreement Issues on the SAT: Look for an underlined noun and
then identify the noun it is representing. Are they both singular? Are they both
plural?.

10 12

Most students attending graduate Providing food, allowing for ample


school in education also work as a exercise, and showing affection are all
teacher while completing their degree an important element in taking care of
programs. a dog.

A) NO CHANGE A) NO CHANGE
B) also will be working as a teacher B) is all an important element
C) also work as teachers C) have all been important elements
D) are also working as teachers D) are all important elements

11

Anyone hoping to become a


professional football player must be
physically strong and possess great
endurance. Furthermore, each player
must be quick learners and adapters in
order to understand the entire
playbook.

A) NO CHANGE
B) a quick learner and adapter
C) both quick learners and adapters
D) possess the ability to learn and
adapt

241 241
SAT Advanced Grammar: Who, Whom, That, Which, and Whose

Advanced Grammar: Who, Whom, That, Which, and Whose

An error on the SAT is to misuse the words who, whom, that, which, and whose.

Identifying Who, Whom, That, Which, and Whose Issues on the SAT: Look for the
word(s) who, whom, that, which, or whose in a sentence. Use the following rules to identify
correct usage.

Who and Whom are used with sentence subjects or objects that are a person.

• Who is used with a sentence subject (We all know who was responsible for the
prank).
• Whom is used with a sentence object (You gave the card to whom?).

That and which are used with sentence subjects or objects that are not a person.

• That is used to introduce a clause that is essential to the sentence.


• Which is used to introduce a clause that is not essential to the sentence.

Whose is used to indicate possession for persons and things (Whose jacket is this?).

13 15

The Royal Proclamation of Pottery-making took root in the


1763 limited North American colonists Edgefield District, an area which rich
that desired to establish settlements clay deposits drew potters during the
west of the Appalachian Mountains. early years of the nineteenth century.

A) NO CHANGE A) NO CHANGE
B) whom B) who’s
C) who C) with
D) which D) whose

14

It is important that you let me know


whom you are inviting to the party.

A) NO CHANGE
B) who
C) whose
D) that

242 242
SAT Advanced Grammar Review

1 4

Dunbar’s breakthrough came at the age of One of the primary causes for the financial
20, when he gave his first public reading crisis of 2008 was the effect of banks
before the Western Association of Writers. granting loans to people who could not pay
The praise he gathered attracted the them back.
attention of other writers, whom began to
take an interest in his career. A) NO CHANGE
B) the affect of
A) NO CHANGE C) an effect of
B) writers, that D) an affect of
C) writers, who
D) writers
5

2 An essential component of the artist’s


process, an artist might use the side of the
By the close of the fourteenth century, the pencil lead to create varied shading.
diet of Vikings in Greenland included more
food taken from the sea than their A) NO CHANGE
Norwegian kinspeople. B) Even though it is a high risk technique
C) To produce the feeling of depth and
A) than their Norwegian kinspeople dimension in a drawing
B) than the diet of their Norwegian D) One type of create method
kinspeople
C) than their kinspeople, who were in
Norway
D) as that of their Norwegian kinspeople

As Simon developed as a playwright, his


plays were often compared to Aeschylus
dating back to BCE 500, a time when the
art form was at a cultural zenith.

A) NO CHANGE
B) the Greek playwright Aeschylus
C) writers of Greek tragedies
D) the works of the Greek tragedian
Aeschylus

243 243
244 244
SAT Geometry in Three Dimensions

SAT Math:
Geometry in Three
Dimensions

Important Formulas
Cone Volume Sphere Volume
1 4
Volume = 𝜋r2 h Volume = 𝜋r3
3 3

Cylinder Surface Area Cylinder Volume

Surface Area = 2𝜋r2 + 2𝜋rh Volume = 𝜋r2 h

Rectangular Prism Surface Area Rectangular Prism Volume

Surface Area = 2wl + 2hl + 2hw Volume = lwh

Rectangular Pyramid Volume Hexagonal Prism Volume


1 3 3
Volume = 3lwh Volume = 2 b2h

Answers on Page 303

245 245
SAT Cones

Cones

Identify: Any question that references the


volume of a cone or the distance from the
apex of a cone to a point along the base.

Set Up: Write down the cone volume


formula to find the missing information for a
volume question. The distance from the
apex of the cone to a point along the base
is the hypotenuse of a right triangle. Cone Volume Formula
1
Volume = 𝜋r2 h
3

1 2

A cone has a height of 60 and a base A


diameter of 20. What is the volume of
this cone?

A) 1,200𝜋
B) 2,000𝜋
C) 3,000𝜋
D) 7,500𝜋

The above cone has a height of 30


inches and a base diameter of 80
inches. What is the distance from A to
B, in inches?

246 246
SAT Cylinders

Cylinders
Identify: Any question that references the
surface area or volume of a cylinder.

Set Up: Write down the appropriate


formula to find the missing information.
Repeat as necessary.
Cylinder Formulas

Volume = 𝜋r2 h

Surface Area = 2𝜋r2 + 2𝜋rh

3 4

A cylinder has a volume of 900𝜋 and a


base diameter of 12. What is the
height of this cylinder?
4 ft

John uses a laundry hamper that is


in the shape of the right circular
cylinder above. If the volume of the
hamper is 64𝜋 cubic feet, what is the
diameter of the base of the hamper,
in feet?

247 247
SAT Rectangular Prisms and Spheres

Rectangular Prisms

Identify: Any question that references the


surface area or volume of a regular
rectangular prism. A cube is a regular
rectangular prism in which all sides have the
same length.

Set Up: Write down the appropriate formula


to find the missing information. Repeat as
Rectangular Prism Formulas
necessary.
Volume = lwh
Surface Area = 2wl + 2hl + 2hw

5 6

The edges of a cube are each 4 feet A large cube has edges that are four
long. What is the surface area, in times as long as those of a small cube.
square feet, of this cube? The volume of the large cube is how
many times the volume of the small
A) 8 cube?
B) 16
C) 64 A) 4
D) 96 B) 16
C) 64
D) 256

248 248
SAT Rectangular Prisms and Spheres

Spheres
Identify: Any question that references the
volume of a sphere.

Set Up: Write down the sphere volume


formula to find the missing information.
Repeat as necessary.

Sphere Volume Formula


4
Volume = 𝜋r3
3

7 8

A local youth soccer league uses a A sphere is inscribed inside of a cube.


soccer ball that has a volume of 288𝜋 If the cube has a length of 50 inches,
cubic inches. What is the diameter of which of the following is closest to
the soccer ball, in inches? the volume of the sphere, in cubic
inches?

A) 5,000𝜋
B) 10,000𝜋
C) 20,000𝜋
D) 50,000𝜋

249 249
SAT Rectangular Pyramids

Rectangular Pyramids

Identify: Any question that references the


volume of a regular rectangular
pyramid or the distance from the apex
of a regular rectangular pyramid to a
point along the base.

Set Up: Write down the appropriate


formula to find the missing information for
a volume question. The distance from the
apex of a rectangular pyramid to a point Rectangular Pyramid Volume Formula
along the base can be found using the
1
hypotenuse of a right triangle. Volume = lwh
3

9 10

A rectangular pyramid has a height A right rectangular pyramid with a


of 15, a length of 60, and a volume of square base has a height that is half
8,100. What is the width of this the area of the base. If the height of
rectangular pyramid? the pyramid is 32 meters, what is the
length of a line, in meters, from the
top of the pyramid to a corner of the
base?

A) 1056
B) 40
C) 2112
D) 60

250 250
SAT Hexagonal Prisms

Hexagonal Prisms

Identify: Any question that references the


volume of a regular hexagonal prism.

Set Up: Either write down the hexagonal prism


volume formula to find the missing information
or break the base of the hexagonal prism into six
equilateral triangles, sum the areas of the six
triangles to find the area of the base, and then
multiply the area of the base by the height.
60° 60°
60° 60° Hexagonal Prism Volume Formula
60°
60° 60° 60° 60° 3 3 2
Volume = bh
60° 60°
60°
60° 60° 2
60° 60°
60° 60°

11 12

A regular hexagonal prism has a


height of 6 inches and a base length
of 3 inches. What is the volume of
this hexagonal prism, in cubic
inches? (Round to the nearest tenth.)

The figure above shows a metal hex


nut with two regular hexagonal faces
and a thickness of 2 cm. The length of
each side of a hexagonal face is 2 cm.
A square hole with a length and
width of 2 cm is drilled through the
nut. The density of the metal is 4.7
grams per cubic cm. What is the mass
of this nut, to the nearest gram?
(Density is mass divided by volume.)

251 251
SAT Geometry in Three Dimensions Review

1 4

The length of the base of a rectangular


prism is 15 cm, and width is 24 cm. If the
height of the rectangular prism is 5 cm,
10 ft
what is its volume in cubic cm?

8 ft

2
Monica currently has two dice of different 6 ft
sizes. If the length of the larger die is four A hut is built from a right circular cone
times greater than the length of the and a right cylinder with internal
smaller die, how many times greater is the measurements represented by the figure
volume of the larger die? above. Of the following, which is closest to
the volume of the hut, in cubic feet?

A) 245
B) 320
C) 456
D) 524

S = 2𝜋𝑟 2 + 2𝜋rh
12 ft

In a cylinder, the radius of the cylinder, r,


and the height of the cylinder, h, are
15 ft related to the surface area S by the
formula above.

15 ft Which of the following expresses the


height of a cylinder in terms of the radius
A new barn was built from a right of the cylinder and its surface area?
rectangular prism and a right rectangular
pyramid with internal measurements
represented by the figure above. What is S
A) –r
2𝜋r
the volume of the barn, in cubic feet?
2𝜋r
B) +r
A) 4,275 S
B) 4,560
C) 2𝜋r – S
C) 6,680
D) 8,400 2𝜋r
D) R +
S

252 252
SAT Advanced Concepts (Math)

SAT Math:
Advanced Concepts

Important Formulas

Compound Interest Formula


r
Compound Interest: A = P(1 + n)nt
A = Amount accumulated
P = Principal
r = Interest rate expressed as a decimal
n = Number of periods per year
t = Number of periods

Answers on Page 303

253 253
SAT Advanced Concepts (Math): Complex Numbers

Advanced Concepts (Math): Complex Numbers (i)

Identify: The question will include the complex number i.

Set Up: Use i = −𝟏 to solve the question. Since i is a radical, i should not appear in the
denominator of a fraction.

1 3

NO CALCULATOR NO CALCULATOR

2+i
For i = −1 , what is the sum (5 – 3i ) 4 – 3i
+ (–8 – 2i) ?
If the expression above is rewritten in
A) –3 – 5i the form a + bi, where a and b are
B) –3 + 5i real numbers, what is the value of a ?
C) 13 – 5i (Note: i = −1)
D) 13 + 5i

Which of the following is equal to


(8 + 3i)(4 – 2i) ? (Note: i = −1)

A) 26 – 4i
B) 38 – 4i
C) 26 + 4i
D) 38 + 4i

254 254
SAT Advanced Concepts (Math): Compound Interest

Advanced Concepts (Math): Compound Interest

Identify: The question will ask about compound interest. Compound interest is an
example of exponential growth.

Set Up: Use the compound interest formula to solve the question.

Compound Interest Formula


r
Compound Interest: A = P(1 + n)nt
A = Amount accumulated
P = Principal
r = Interest rate expressed as a decimal
n = Number of periods per year
t = Number of periods

4 6

NO CALCULATOR Keith and his friend Sarah each found


accounts that earn four percent
Bhavik opened a bank account that interest compounded annually. Sarah
earns 3 percent interest compounded made an initial deposit of $1,000 into
annually. His initial deposit was her account and at the same time
$2,000 and he uses the expression Keith made a deposit of $500 into his
$2,000(x)t to find the value of the account. After seven years, how much
account after t years. What is the value more money will Sarah’s initial
of x in the expression? deposit have earned than Keith’s
initial deposit?

A) $456.12
B) $524.67
5
C) $586.76
Timothy found an account that earns 6 D) $657.97
percent compounded monthly. Timothy
made an initial deposit of $25 into his
account. After 10 years, how much
money will Timothy have in his
account?

A) $45.48
B) $52.84
C) $58.45
D) $71.24

255 255
SAT Advanced Concepts: Trigonometry: Sine, Cosine, and Tangent

Trigonometry: Sine, Cosine, and Tangent

Identify: Look for references to sine, cosine, or tangent in the question.

Set Up: Use the SOH-CAH-TOA relationships to solve the question.

SOH-CAH-TOA
opposite adjacent opposite
sin Θ = cos Θ = tan Θ =
hypotenuse hypotenuse adjacent

7 8

NO CALCULATOR

In ∆XYZ, shown above, P is on YZ,


the length of XZ is 7 yards, and cos z
= 0.6. How many yards long is XP ?

Given the right triangle ABC above,


c
which of the following is equal to ?
b

A) sin B
9
B) tan B
C) tan C In triangle ABC, the measure of ∠A is
D) cos C 90°, AB = 5, and BC = 13. Triangle
XYZ is similar to triangle ABC, where
vertices X, Y and Z correspond to
vertices A, B, and C, respectively, and
each side of triangle XYZ is eight
times the length of the corresponding
side of triangle ABC. What is the
value of tan Y ?

256 256
SAT Advanced Concepts (Math): Unit Circle

Advanced Concepts (Math): Unit Circle

Identify: Look for trigonometric identities (sine,


cosine, or tangent) and radian measures in the
question.

Set Up: Use unit circle values or draw a unit


circle and graph the information provided in the
question to solve the question.

Cosine and sine are co-functions of each other.


This means that complementary angles of cosine and
sine are equal. First number (x-coordinate): Cosine
Second number (y-coordinate): Sine
Tangent = sine/cosine

10 11

NO CALCULATOR NO CALCULATOR

It is given that sin x = a, where x is the


radian measure of an angle and

0 < x < 2. If sin w = –a, which of the
following could NOT be the value of w?

a° 3
A) x – 2
7 
In the above right triangle sin a° = 8. B) x + 2
What is cos (90° − a°) ?
C) x + 

D) x – 2

257 257
SAT Advanced Concepts (Math): Review

1 3

NO CALCULATOR

James opened a bank account that


earns 8 percent interest compounded
quarterly. His initial deposit was $80 a° b°
and he uses the expression $80(a)b to
find the value of the account after 12
years. What is the value of b in the The angles shown above are acute
expression? and sin(a°) = cos(b°). If a = 2z − 32
and b = 4z + 2, what is the value of z ?

NO CALCULATOR

4
In the xy-plane below, O is the center
NO CALCULATOR)
of the circle, and the measure of

∠AOB is radians. What is the value Which of the following is equal to
b
of b ? 
sin(5)?


A) –cos(5)


B) –sin(5)

3
C) cos(10)

7
D) sin(10)

258 258
SAT Advanced Concepts (Math): Review

5 6

8
NO CALCULATOR Suppose 0° < x < 90° and tan x = 11.

What is the value of sin x + cos x ?


Which of the following complex
8 + 3i
numbers is equivalent to ? 19
2 – 5i A)
185
(Note: i = −1)
19
B)
1 34i 370
A) – 21 – 21
7
1 34i C)
B) – 21 + 21 185
7
1 46i D)
C) 29 + 29 370

1 46i
D) 29 – 29

259 259
260 260
SAT Practice Reading Passages

SAT Reading:
Practice Passages

Answers on Page 294

261 261
SAT Practice Reading Passage #1

Reading Test
Practice Passages

Directions

Each passage or pair of passages below is followed by a number of questions. After


reading each passage or pair, choose the best answer to each question based on what is
stated or implied in the passage or passages and in any accompanying graphics (such as a
table or graph).

Questions 1-11 are based on the following main customers in the area shifted to the
passage. European supplier. Few of them, it seems, realize –
25 let alone care – that the stuff comes from Europe.
The following excerpt meditates on trends in In the new mental geography created by the
technology and their effects on business. railroad, humanity mastered distance. In the
mental geography of e-commerce, distance has
Line E-commerce is to the Information Revolution been eliminated. There is only one economy and
what the railroad was to the Industrial Revolution 30 only one market.
– a totally new, totally unprecedented, totally One consequence of this is that every business
unexpected development. And like the railroad 170 must become globally competitive, even if it
5 years ago, e-commerce is creating a new and manufactures or sells only within a local or
distinct boom, rapidly changing the economy, regional market. The competition is not local
society, and politics. 35 anymore – in fact, it knows no boundaries. Every
One example: a mid-sized company in America’s company has to become transnational in the way it
industrial Midwest, founded in the 1920s and now is run. Yet the traditional multinational may well
10 run by the grandchildren of the founder, used to become obsolete. It manufactures and distributes
have some sixty percent of the market in in a number of distinct geographies, in which it is a
inexpensive dinnerware for fast food eateries, 40 local company. But in e-commerce there are
school and office cafeterias, and hospitals within a neither local companies nor distinct geographies.
hundred-mile radius of its factory. China is heavy At the same time, it is not yet clear what kinds
15 and breaks easily, so cheap china is traditionally of goods and services will be bought and sold
sold within a small area. Almost overnight this through e-commerce and what kinds will turn out
company lost more than half of its market. One of 45 to be unsuitable for it. This has been true
its customers, a hospital cafeteria where someone whenever a new distribution channel has arisen.
went “surfing” on the Internet, discovered a Twenty-five years ago it was generally believed
20 European manufacturer that offered china of that within a few decades the printed word would
apparently better quality at a lower price and be dispatched electronically to individual
shipped cheaply by air. Within a few months the 50 subscribers’ computer screens. Subscribers would

262 262
SAT Practice Reading Passage #1
then either read text on their computer screens or 2
download it and print it out. This was the
assumption that underlay the CD-ROM. On the whole, the author’s attitude toward e-
Thus many newspapers and magazines, by no commerce is one of
55 means only in the United States, established
themselves online; few, so far, have become gold A) reconsideration.
mines. But anyone who twenty years ago predicted B) caution.
the business of Amazon.com and C) zeal.
Barnesandnoble.com – that is, that books would be D) acceptance.
60 sold on the Internet but delivered in their heavy,
printed form – would have been laughed off the
podium. Yet Amazon.com and Barnesandnoble.com 3
are in exactly that business, and they are in it
worldwide. The first order for the U.S. edition of The author suggests that, when it comes to
65 my most recent book, Management Challenges for making purchases, consumers are
the 21st Century, came to Amazon.com, and it came
from Argentina. A) primarily concerned with price.
Where to manufacture, where to sell, and how B) culturally ignorant.
to sell it will remain important business decisions. C) cheated by local companies.
70 But in another twenty years they may no longer D) technologically driven.
determine what a company does, how it does it,
and where it does it, so momentous have been the
changes brought about by e-commerce. 4
Which choice provides the best evidence for
Amazon.com Revenue by Year the answer to the previous question?
100
Revenue (in United States dollars)

A) Lines 17-22 (“One…air”)


90 B) Lines 31-34 (“One…market”)
80 C) Lines 37-38 (“Yet…obsolete”)
70 D) Lines 40-41 (“But…geographies”)
60
50
40 5
30 As used in line 46, “channel” most nearly
20 means
10
A) conduct.
0
B) river.
2004 2005 2006 2007 2008 2009 2010 2011 2012 2013 2014
C) pathway.
Year D) model.

1
6
The primary purpose of the passage is to The author highlights online newspapers
A) warn about the encroachment of foreign and magazines to underscore
businesses. A) an alarming global movement.
B) predict the organizational structure of B) an inability to make money in the digital
businesses in the internet age. publishing industry.
C) illuminate the nature of different C) the misguided aspirations of online
technological revolutions. publications capitalism.
D) discuss the growing impact of e- D) an inaccurate prediction of technological
commerce on global industry. trends.

263 263
SAT Practice Reading Passage #1

7 10

Which choice provides the best evidence for The author’s use of the words “what,” “how,”
the answer to the previous question? and “where” in lines 71-72 in the final
paragraph function mainly to
A) Lines 37-38 (“Yet…obsolete”)
B) Lines 47-50 (“Twenty-five…screens”)
A) highlight how e-commerce industries
C) Lines 54-57 (“Thus…mines”)
will conduct business in the future.
D) Lines 57-62 (“But…podium”)
B) indicate that e-commerce companies will
fully replace traditional businesses
within the next 20 years.
C) advocate for the increased regulation of
8 e-commerce.
D) emphasize the revolutionary impact of e-
As used in line 49, “dispatched” most nearly
commerce.
means
A) dismissed.
B) replaced. 11
C) sent.
D) overthrown. According to the graph, which statement is
true about Amazon.com revenue from 2004 to
2014?

9 A) Growth of Amazon.com revenue in 2014


was widely out of proportion to growth of
What function does the discussion of revenue during the five years prior.
Amazon.com and Barnesandnoble.com (lines B) Amazon.com revenue surpassed 80 billion
58-59) serve in the context of the passage? dollars for the first time in 2014.
C) Amazon.com revenue will reach 100
A) It enumerates the first two e-commerce billion dollars in 2015.
companies to have success. D) Amazon.com revenue began a new upward
B) It presents two examples of online trend in 2014.
newspapers and magazines.
C) It highlights two companies that have
thrived in the online environment.
D) It presents an argument against the
apparent benefits of e-commerce.

264 264
SAT Practice Reading Passage #2
Questions 12-21 are based on the following anything if it’s all you have. Maybe the less you
passage. have, the more you are required to boast.
The floor of the Salinas Valley, between the
The following excerpt from a novel describes a ranges and below the foothills, is level because this
child’s impression of the California landscape. 45 valley used to be the bottom of a hundred-mile
inlet from the sea. The river mouth at Moss
Line The Salinas Valley is in Northern California. It Landing was centuries ago the entrance to this
is a long narrow swale between two ranges of long inland water. Once, fifty miles down the
mountains, and the Salinas River winds and twists valley, my father bored a well. The drill came up
up the center until it falls at last into Monterey 50 first with topsoil and then with gravel and then
5 Bay. with white sea sand full of shells and even pieces of
I remember that the Gabilan Mountains to the whalebone. There were twenty feet of sand and
east of the valley were light gay mountains full of then black earth again, and even a piece of
sun and loveliness and a kind of invitation, so that redwood, that imperishable wood that does not rot.
you wanted to climb into their warm foothills 55 Before the inland sea the valley must have been a
10 almost as you want to climb into the lap of a forest. And those things had happened right under
beloved mother. They were beckoning mountains our feet. And it seemed to me sometimes at night
with a brown grass love. The Santa Lucias stood up that I could feel both the sea and the redwood
against the sky to the west and kept the valley forest before it.
from the open sea, and they were dark and 60 I have spoken of the rich years when the
15 brooding – unfriendly and dangerous. I always rainfall was plentiful. But there were dry years
found in myself a dread of west and a love of east. too, and they put a terror on the valley. The water
Where I ever got such an idea I cannot say, unless came in a thirty-year cycle. There would be five or
it could be that the morning came over the peaks of six wet and wonderful years when there might be
the Gabilans and the night drifted back from the 65 nineteen to twenty-five inches of rain, and the land
20 ridges of the Santa Lucias. It may be that the birth would shout with grass. Then would come six or
and death of the day had some part in my feeling seven pretty good years of twelve to sixteen inches
about the two ranges of mountains. of rain. And then the dry years would come, and
From both sides of the valley little streams sometimes there would be only seven or eight
slipped out of the hill canyons and fell into the bed 70 inches of rain. The land cracked and the springs
25 of the Salinas River. In the winter of wet years the dried up and the cattle listlessly nibbled dry twigs.
streams ran full force, and they swelled the river Then the farmers and the ranchers would be filled
until sometimes it raged and boiled, bank full, and with disgust for the Salinas Valley. Some families
then it was a destroyer. The river tore the edges of would sell out for nearly nothing and move away.
the farm lands and washed whole acres down; it 75 And it never failed that during the dry years the
30 toppled barns and houses into itself, to go floating people forgot about the rich years, and during the
and bobbing away. It trapped cows and pigs and wet years they lost all memory of the dry years. It
sheep and drowned them in its muddy brown was always that way.
water and carried them to the sea. Then when the
late spring came, the river drew in from its edges
35 and the sand banks appeared. The Salinas was
only a part-time river. The summer sun drove it
underground. It was not a fine river at all, but it
was the only one we had and so we boasted about it
– how dangerous it was in a wet winter and how
40 dry it was in a dry summer. You can boast about

265 265
SAT Practice Reading Passage #2

12 15

The primary purpose of the passage is to Which choice provides the best evidence for
the answer to the previous question?
A) describe a particular landscape
objectively. A) Lines 6-11 (“I remember…mother”)
B) evoke personal responses to a landscape. B) Lines 17-20 (“Where…Lucias”)
C) encourage the reader to visit the Salinas C) Lines 23-25 (“From both…river”)
Valley. D) Lines 41-42 (“Maybe…boast”)
D) champion the protection of a special
portion of the American countryside.

16

The main purpose of the third paragraph is to


13 convey the author’s
The references to “birth” and “death” at the A) ability to recall insignificant events.
end of the second paragraph mainly have B) distaste for the Salinas River.
which effect? C) respect for nature’s destructive potential.
A) They evoke the narrator’s contrasting D) sadness at the frequent loss of farm life.
feelings.
B) They reflect the author’s auspicious
thoughts.
C) They capture the narrator’s existential 17
fear.
D) They reveal the narrator’s longing for The passage suggests that the author views
daylight. his father’s activities as
A) alleviating a drought.
B) demonstrating archaeological theories.
C) unlocking a mystery.
14 D) unearthing the past.
The passage indicates the author’s
relationship with nature
18
A) inspires a religious connection to the
earth.
Which choice provides the best evidence for
B) causes a desire to return to the east.
the answer to the previous question?
C) evokes a childlike feeling.
D) influences the way the author perceives A) Lines 31-33 (“It trapped…sea”)
the world. B) Lines 48-49 (“Once…well”)
C) Lines 55-56 (“Before…forest”)
D) Lines 63-66 (“There…grass”)

266 266
SAT Practice Reading Passage #2

19 21

The description of the “piece of redwood” The author’s use of the word “forgot” (line 76)
(lines 53-54) implies that the author suggests that, in relation to the “people” (line
76), the author
A) desires to live a long life.
B) laments the loss of the great ancient A) has a greater capacity for memory.
forest. B) appreciated the river more.
C) values the redwood more than other C) was more bound to the land.
trees. D) views life in a larger temporal context.
D) prizes its endurance.

20

As used in line 66, “shout” most nearly


means
A) anger.
B) yell.
C) rejoice.
D) abound.

267 267
SAT Practice Reading Passage #3
Questions 22-32 are based on the following solitary inventor, tinkering in his shop, has been
passage. overshadowed by task forces of scientists in
laboratories and testing fields. In the same fashion,
This passage is adapted from the Farewell Address the free university, historically the fountainhead of
of Dwight Eisenhower (January 17, 1961). 55 free ideas and scientific discovery, has experienced
a revolution in the conduct of research. Partly
A vital element in keeping the peace is our because of the huge costs involved, a government
military establishment. Our arms must be mighty, contract becomes virtually a substitute for
ready for instant action, so that no potential intellectual curiosity. For every old blackboard
Line aggressor may be tempted to risk his own 60 there are now hundreds of new electronic
5 destruction. Our military organization today bears computers.
little relation to that known by any of my The prospect of domination of the nation’s
predecessors in peacetime, or indeed by the scholars by Federal employment, project
fighting men of World War II or Korea. Until the allocations, and the power of money is ever present
latest of our world conflicts, the United States had 65 – and is gravely to be regarded. Yet, in holding
10 no armaments industry. American makers of scientific research and discovery in respect, as we
plowshares could, with time and as required, make should, we must also be alert to the equal and
swords as well. But now we can no longer risk opposite danger that public policy could itself
emergency improvisation of national defense; we become the captive of a scientific technological
have been compelled to create a permanent 70 elite. It is the task of statesmanship to mold, to
15 armaments industry of vast proportions. Added to balance, and to integrate these and other forces,
this, three and a half million men and women are new and old, within the principles of our
directly engaged in the defense establishment. We democratic system - ever aiming toward the
annually spend on military security alone more supreme goals of our free society.
than the net income of all United States
20 corporations. This conjunction of an immense Figure 1
military establishment and a large arms industry Total United States (US) Defense Expenditures:
is new in the American experience. The total Military Research and Development
influence - economic, political, even spiritual – is 90
felt in every city, every State house, every office of 80
Spending (in billion of

25 the Federal government. We recognize the


2016 US dollars)

70
imperative need for this development. Yet we must 60
not fail to comprehend its grave implications. Our 50
toil, resources and livelihood are all involved; so is 40
the very structure of our society. In the councils of 30
30 government, we must guard against the 20
acquisition of unwarranted influence, whether 10
sought or unsought, by the military-industrial 0
complex. The potential for the disastrous rise of 1962 1971 1980 1989 1998 2007 2016
misplaced power exists and will persist. We must
Fiscal Year
35 never let the weight of this combination endanger
our liberties or democratic processes. We should Figure 2
take nothing for granted. Only an alert and US Military Research and Development as
knowledgeable citizenry can compel the proper Percent of Defense Outlay
16%
meshing of the huge industrial and military
40 machinery of defense with our peaceful methods 14%
and goals, so that security and liberty may prosper 12%
together. 10%
Percentage

Akin to, and largely responsible for the


8%
sweeping changes in our industrial-military
posture, has been the technological revolution 6%
45
during recent decades. In this revolution, research 4%
has become central; it also becomes more 2%
formalized, complex, and costly. A steadily 0%
increasing share is conducted for, by, or at the 1962 1971 1980 1989 1998 2007 2016
50 direction of, the Federal government. Today, the Fiscal Year

268 268
SAT Practice Reading Passage #3

22 26

The primary purpose of the passage is to As used in line 27, “grave” most nearly
means
A) discuss the growing influence of military
industry in national affairs. A) deadly.
B) predict the organizational structure of B) inevitable.
businesses in the military-industrial C) intense.
age. D) serious.
C) illuminate the evolution of military
technologies.
27
D) warn about the increasing impact of
military industry on global politics. According to the author, a recent
development is

A) the participation of private industry in


23 military research and development.
B) the use of computing in warfare.
The author’s attitude toward the military- C) the waning influence of amateur
industrial complex is best described as one of innovators.
D) the government placing restrictions on
A) apprehension. private enterprise.
B) ambivalence.
C) appreciation. 28
D) astonishment.
Which choice provides the best evidence for
the answer to the previous question?
A) Lines 20-22 (“This…experience”)
B) Lines 33-34 (“The potential…persist”)
24
C) Lines 50-53 (“Today…fields”)
Which choice provides the best evidence for D) Lines 62-65 (“The prospect…regarded”)
the answer to the previous question?
29
A) Lines 1-2 (“A vital…establishment”)
B) Lines 20-22 (“This…experience”) As used in line 54, “fountainhead” most
C) Lines 33-34 (“The potential…persist”) nearly means
D) Lines 70-74 (“It is…society”)
A) apex.
B) culmination.
C) focus.
D) home.
25
30
The author implies that, when it comes to
military research and development, the According to the data in the figures, since
growing role of corporations is 1962, the greatest United States Defense
expenditure in military research and
A) disastrous. development in total US dollars occurred in
B) a positive development. what year?
C) an issue worth studying.
D) necessary, but unfortunate. A) 1967
B) 1992
C) 1997
D) 2008

269 269
SAT Practice Reading Passage #3

31

Taken together, the two figures suggest that

A) United States military research and


development as a percentage of total
United States Defense outlay has
increased significantly since 1962.
B) the total United States Defense
expenditure on military research and
development had its greatest five-year
increase from 2011 to 2016.
C) the total United States Defense budget
has increased significantly since 1962.
D) there is a direct correlation between the
total United States Defense expenditure
on military research and United States
military research and development as a
percentage of total US Defense outlay.

32

Do the data in the figures provide support for


the author’s claim made in the second
paragraph (lines 43-61)?

A) Yes, because the data provide evidence of


a significant increase in the total United
States Defense expenditure in military
research since 1962.
B) Yes, because the data provide evidence of
a significant increase in the United States
military research and development as a
percentage of total United States Defense
outlay since 1962.
C) No, because the data do not provide
evidence about the overall share of
military research and development
conducted by the federal government.
D) No, because the data do not indicate
whether or not military research and
development expenditures by United
States Defense has increased.

270 270
SAT Practice Reading Passage #4

Questions 33-42 are based on the following us exactly when the universe began.
passage. Despite the many extraordinary
accomplishments of the Hubble Telescope, it has
The passage below discusses the Hubble Space not had an easy life. In 1993, scientists discovered
Telescope. 45 that some of the images returned by Hubble were
fuzzy. Because the telescope was low in Earth’s
Have you ever wondered what a star looks like orbit, it could be serviced by a space shuttle, so a
up-close? Or maybe you’ve dreamed of a close shuttle was employed to fix Hubble’s cameras. This
encounter with the surface of a far away planet. first servicing mission in December 1993 was a
Since 1990, the Hubble Telescope has been offering 50 great success. Several other shuttle missions to
5 scientists and curious amateurs alike data and repair and upgrade Hubble have since been made,
images of stars, planets, and other celestial bodies, each meeting with the same success as the first.
never before seen by the human eye. During its time in space, Hubble has observed
Originally designed in the 1970’s, this far- various parts of the universe. It has provided
reaching telescope was named for scientist Edwin 55 scientists with images that have had an impact on
10 P. Hubble. During the 1920’s, Dr. Hubble outlined several areas of astronomy. For example, observing
a classification system for galaxies, which is still in objects in our own solar system, Hubble provided
use today. He also discovered a relationship spectacular views of Comet Shoemaker-Levy 9’s
between a galaxy’s distance and the speed with collision with Jupiter. Its cameras delivered the
which it is moving. Before the launch of the Hubble 60 first detailed images of Pluto and its moon,
15 Telescope, distances to far-off galaxies were not Charon. The telescope has documented the births
well known. Questions such as how rapidly the and deaths of many stars, as well. It provided the
universe is expanding, and for how long, created first visual proof that cosmic dust encircling young
great controversy. One of the Hubble Telescope’s stars is a common phenomenon. Hubble has also
primary goals is to determine the accuracy of Dr. 65 delivered many colorful pictures of stellar deaths,
20 Hubble’s predictions, as well as to answer many of including the destruction of a massive star called
the questions surrounding space exploration. Supernova 1987A.
Human fascination with space began hundreds Hubble has also managed to probe the central
of years ago. That interest has not waned, and regions of galaxies, crowded with stars, dust, and
today, the Hubble Telescope orbits 375 miles above 70 gas. It has provided evidence that large black holes
25 Earth, working nonstop to unlock the secrets of the make up the centers of many galaxies. These black
universe. It uses state-of-the-art instruments to holes are thought to be created after the death of a
provide amazing views of the universe that cannot large star. Because a black hole has a very strong
be made using ground-based telescopes. The gravitational pull, cosmic dust, planets, and other
reason for this is that, from the ground, we look at 75 objects tend to swirl around it. This creates a
30 stars and other objects in space through the common spiral galaxy, such as our own, the Milky
Earth’s atmosphere. Our atmosphere is full of Way. It is clear that the Hubble Telescope has
clouds, dust, and pollution, which causes made many significant contributions to the study
everything we see to shimmer and shake. This of space. Scientists reviewing Hubble’s data are
makes it very difficult for scientists on the ground 80 getting closer to answering questions related to the
35 to make steady, accurate measurements of objects size and age of the universe. Edwin P. Hubble
moving in space. With assistance from the Hubble would most likely be amazed at the data and
Telescope, astronomers can pinpoint the location of images returned to us by his namesake. Since it is
faraway stars and galaxies, and measure the scheduled to stay in orbit indefinitely, the Hubble
speeds and distances of astronomical objects much 85 Telescope should continue transmitting pictures of
40 more precisely. Astronomers may one day even tell the universe for years to come.

271 271
SAT Practice Reading Passage #4

33 37
The passage is written from the perspective of Which choice provides the best evidence for
someone who is the answer to the previous question?

A) actively involved in conducting A) Lines 12-14 (“He…moving”)


astronomical research. B) Lines 42-44 (“Despite …life”)
B) a participant in a recent debate in the C) Lines 68-70 (“Hubble…gas”)
field of astronomy. D) Lines 81-83 (“Edwin…namesake”)
C) knowledgeable about recent
advancements in astronomical research.
D) an advocate for further astronomical
research. 38
As used in line 39, “astronomical” most
nearly means
34

The passage is best described as A) exorbitant.


B) cosmological.
A) a social commentary on scientific C) giant.
advancements in the twentieth century. D) vast.
B) a detailed account of a technological
improvement in a scientific field.
C) a story of how one individual inspired 39
many others.
D) an introduction to an astronomical The passage mainly suggests which of the
method in a systematic manner. following about the Hubble Telescope’s
maintenance issues?

35 A) They were not so great as to severely


interfere with normal operations.
As used in line 5, “amateurs” most nearly
B) They interfered greatly with the operation
means
of the telescope.
C) Although the cameras were fixed, the
A) part-time workers.
fuzzy images that Hubble produced
B) specialists.
created large setbacks.
C) unskilled laborers.
D) Shuttle missions are not the only viable
D) non-professionals.
means of fixing the telescope’s cameras.

36
40
The passage suggests that Edwin P. Hubble
was Which choice provides the best evidence for
the answer to the previous question?
A) very effective at persuading others to
adopt his ideas. A) Lines 26-28 (“It uses…telescopes”)
B) committed to the exploration of a variety B) Lines 44-46 (“In 1993 …fuzzy”)
of sciences. C) Lines 50-52 (“Several…first”)
C) the most important scientific mind in D) Lines 77-79 (“It is…space”)
regards to discoveries in the field of
astronomy.
D) an instrumental scientist who made
great contributions to our understanding
of the universe.

272 272
SAT Practice Reading Passage #4

41
The function of the fourth paragraph (lines
41-52) is to

A) provide insight into the everyday


workings of Hubble’s repair operations.
B) sharpen the reader’s knowledge of
potential setbacks with the telescope.
C) summarize the history of the Hubble
Telescope.
D) point out the favorable results of repair
missions to Hubble.

42

The examples cited in the last paragraph


(lines 68-86) are primarily drawn from

A) speculation.
B) opinion.
C) historical reference.
D) observation.

273 273
SAT Practice Reading Passage #5

Questions 43-52 are based on the following The other alternative would be to share the cost
passage. of the sand replenishment project through local,
40 state, and federal funds, but that would turn the
These passages present two perspectives on the project into a public welfare program for
issue of sand erosion and beach replenishment in multimillionaire beachfront property owners. The
California. sand replenishment project protects the real estate
value of residential and business properties along
Passage 1 45 the shoreline, but does little to profit the ocean
environment and the community at large.
Responsibilities pertaining to property and land Landlords must make a decision: pass this ballot
use are agreed upon within a signed, binding measure to obtain their sand and keep their renters,
contract. While it is fathomable that a disagreement or let the ballot measure fail and watch their
with the owner of an adjacent property may occur, a 50 renters dissipate like the sand.
5 difficulty arises in defining the party liable for
necessary maintenance when said disagreement is
over the deterioration of a naturally occurring, Passage 2
communally owned parcel, such as a beach. In
California, the coastal city of Encinitas is slowly Consequentialism refers to those moral theories
10 watching its shoreline wash away. The beach has a which hold that the consequences of a particular
massive sand deficit with over 100,000 cubic yards action form the basis for any valid moral judgment
of sand lost on an annual basis. The sand erosion is about that action. Thus, from a consequentialist
a natural occurrence, but beachfront property 55 standpoint, a morally right action is one that
owners and the city government are attempting to produces a good outcome, or consequence. In its
15 fight mother nature with a sand replenishment simplest terms, the end justifies the means. In
project. This involves dredging up 2 million cubic Encinitas, California, environmentalists and city
yards of sand from the ocean bottom at a cost of $25 officials agree upon the desired end, but cannot
million. 60 agree upon the means to bring it about. At issue is
To raise the requisite $25 million, a transient the sand replenishment project on the November
20 occupancy tax is being proposed on the November ballot. Because of sand erosion, the bluffs along the
ballot. It would increase the 10% tax on motel rooms city’s coastline have been collapsing for seven
and other short-term vacation rentals to 13%, years, hurting beachgoers, with one incident
dedicating the extra 3% entirely to the sand 65 resulting in death. In an attempt to resolve this
replenishment project. Motel and vacation rental crisis, a sand replenishment project is being
25 owners argue that if vacation renters are asked to debated by the general public, but the devil is in
pay higher fees, it will make vacation rentals the details. Most agree that the eroding beach
unaffordable and out of reach for many tourists, needs to be replenished, but to what degree
especially in a stagnant economy. They contend that 70 depends upon the negative environmental impact
if the renters don’t come to Encinitas, then they’ll one is willing to accept. It is the modus operandi of
30 spend their money on vacations elsewhere, causing the project that could lead to varying degrees of
the regional economy to suffer as well. But it is not environmental degradation. The short-term
clear how likely this outcome is. Vacationers who incontestable result is that further erosion of the
are already accustomed to paying $3,000 a week in 75 70-foot-tall bluffs will be abated, resulting in the
rent, plus a $300 tax, are unlikely to be dissuaded preservation of bluff-top homes above and the
35 by an extra $90, especially considering that the protection of pedestrians on the beach below. Aside
ancillary capital will keep their favorite vacation from the natural ebb and flow of the beachfront,
spot in pristine condition year after year. the man-made causes for the beach’s erosion are

274 274
SAT Practice Reading Passage #5

80 coastal development, the damming of rivers, sand- 44


mining operations, jetty construction, harbor
dredging, and seawalls. Which choice does the author of Passage 1
The city’s rejoinder is to deposit a substantial explicitly cite as something that influences
amount of sand onto the beach once every few the behavior of vacationers?
85 years, combined with the building of artificial reefs
to slow down sand erosion. This solution concerns A) Eroded beaches
environmentalists who are worried about the B) Increased costs
movement of sand and the persistent burial of reef C) Weather
habitat, which supports various kelps and surf D) The national economy
90 grass. Additionally, there are resultant adverse
impacts on the recreationally and commercially
important invertebrates and fish that utilize those 45
habitats during various life stages. The Which choice provides the best evidence for
environmentalists’ nostrum to this predicament is the answer to the previous question?
95 to eliminate the man-made causes of the beach’s
erosion. But the resolution to this quandary need A) Lines 12-16 (“The sand…project”)
not be that stark. A well-designed beach B) Lines 24-28 (“Motel…economy”)
replenishment project avoids beach fill in areas C) Lines 42-46 (“The sand…large”)
with sensitive marine resources. The means do not D) Lines 47-50 (“Landlords…sand”)
100 mitigate the conflict between environmentalists
and the city, but the end consequence does bring
minimal environmental impact upon wildlife, while
sustaining a safe beach for all to enjoy. 46

It can be reasonably inferred from Passage 1


that the alternative mentioned in the third
paragraph (lines 38-50) would
43

As used in line 28, “stagnant” most nearly A) be widely approved.


means B) be widely disapproved.
C) be favored by renters but not by property
A) foul. owners.
B) impure. D) be favored by government officials but not
C) sluggish. by property owners.
D) motionless.

275 275
SAT Practice Reading Passage #5

47 50
The author of Passage 2 implies that the Which statement most appropriately
movement of sand onto the beach will describes a distinction between the passages?

A) allow for the creation of more natural A) Passage 1 emphasizes the importance of
habitats. the beach replenishment project, while
B) trivialize the danger of beach erosion. Passage 2 does not.
C) not impact the environment in any way. B) Passage 1 fails to include facts and
D) be a better response to the problem than figures, while Passage 2 includes such
trying to eliminate the causes of erosion. data.
C) Passage 1 depicts the beach
replenishment project as significant,
whereas Passage 2 does not.
48 D) Passage 1 is centered around the
economic impact of the beach
Which choice provides the best evidence for replenishment project, whereas Passage 2
the answer to the previous question? is not.

A) Lines 51-54 (“Consequentialism…action”)


B) Lines 77-82 (“Aside…seawalls”) 51
C) Lines 83-86 (“The city’s…erosion”)
D) Lines 99-104 (“The means…enjoy”) In both passages, the authors indicate that
beach replenishment is

A) a struggle against nature.


B) an economic crisis.
49 C) a battle between government and
The passages can both be best described as citizens.
D) an important civic duty.
A) explanations of a problem.
B) stories of human peril.
C) illustrations of a commonly held belief.
D) introductions into natural phenomena. 52

Which of the following statements about


beach erosion is supported by both passages?

A) The responsibility to maintain beaches


lies exclusively with property owners.
B) The problem of beach erosion will not
correct itself naturally.
C) The economic impact of beach erosion is
minimal.
D) It is not feasible to move enough sand
from the ocean to combat the problem.

276 276
SAT Practice Writing Passages

SAT Writing:
Practice Passages

Answers on Page 298

277 277
SAT Practice Writing Passage #1

Writing and Language Test


Sample Passages

Directions

Each passage below is accompanied by a number of questions. For some questions, you will
consider how the passage might be revised to improve the expression of ideas. For other
questions, you will consider how the passage might be edited to correct errors in sentence
structure, usage, or punctuation. A passage or a question may be accompanied by one or more
graphics (such as a table or graph) that you will consider as you make revising and editing
decisions.

Some questions will direct you to an underlined portion of a passage. Other questions will
direct you to a location in a passage or ask you to think about the passage as a whole.

After reading each passage, choose the answer to each question that most effectively improves
the quality of writing in the passage or that makes the passage conform to the conventions of
standard written English. Many questions include a “NO CHANGE” option. Choose that
option if you think the best choice is to leave the relevant portion of the passage as it is.

Questions 1-11 are based on the following passage. 1


Green Roofs
A) NO CHANGE
B) roofs: plant-filled
As evidence that human activity is responsible for
C) roofs; plant-filled
D) roofs – plant-filled
climate change grows stronger, many ecological
innovators are developing new approaches to
finding renewable, clean energy sources. Recently,
2
the concept of green 1 roofs, plant-filled roof top
A) NO CHANGE
gardens – 2 has been receiving widespread B) have been
C) was
attention as one substantial way to confront the D) were

country’s energy problems.

278 278
SAT Practice Writing Passage #1

The benefits of green roofs are 3 notable. A 3


green roof acts as a sponge for rainwater, absorbing A) NO CHANGE
B) exhaustive
the majority of water from a typical C) manifold
rainstorm: the roof only requires three to five D) well-known
4
inches of soil to achieve this level of water
absorption, which also helps minimize the weight of 4

the roof. 5 Although the soil absorbs rainwater, A) NO CHANGE


B) rainstorm; so the roof
drainage and sewage systems have a decreased C) rainstorm, the roof
D) rainstorm; the roof
volume of water pumping through them, keeping
these systems from becoming overburdened. The soil
and plants also help improve air quality, taking in
5
carbon for nourishment and emitting clean oxygen
A) NO CHANGE
back into the atmosphere. B) Because
C) Hence
6 Beyond the relative ease of installation, green D) Yet
roofs also provide excellent insulation. Most
traditional roofing products do a poor or moderate 6
job with both interior and exterior temperature
Which choice best connects the sentence with the
control. Green roofs, 7 moreover, have been previous paragraph?

proven very effective at protecting buildings from A) NO CHANGE


B) In addition to absorbing rainwater and
extreme outdoor temperatures. During a hot pollutants
C) Supplementary to their aesthetic appeal
summer day, the temperature of a vegetated roof can
D) Even though they are more expensive than
be markedly lower than the outdoor air temperature, traditional roofs

often between 25° and 30° cooler. By the same token,


7
a green roof serves as a great insulator for keeping
A) NO CHANGE
the heat inside the structure B) furthermore
C) likewise
D) on the other hand

279 279
SAT Practice Writing Passage #1

8
during the cooler months. 8
At this point, the writer is considering adding the
A well-designed green roof also adds to the
following sentence
aesthetic appeal of any building. Many visually
As a bonus, green roofs also operate as noise
stunning plants thrive in rooftop gardens. During insulators, muffling outdoor traffic and
construction sounds.
the growing season, birds and butterflies will
Should the writer make this addition here?
frequent, and sometimes even build nests in, these
A) Yes, because it further highlights the key role
roof parks. In office buildings, green roofs are nearly of green roofs in protecting the environment.
B) Yes, because it provides an additional example
always accessible, not only for maintenance
of the insulation benefits of green roofs.
purposes, 9 but to allow employees a quiet place C) No, because it unnecessarily repeats
information from earlier in the passage.
to relax and commune with nature. In today’s world D) No, because it disrupts the flow of the
paragraph by supplying irrelevant
of pavement and concrete, these roofs quickly information.

become a welcome respite from the normal workday.


One criticism of green roofs is the additional cost
9
of installation. Not surprisingly, the process of
A) NO CHANGE
installing a green roof is highly specialized, B) and
C) but also
including its support structure, drainage material, D) or
moisture barrier and finally, the cost for soil and
plant material. Since green roofs require a
10
contractor who understands both construction and
landscape design, finding an appropriate installer A) NO CHANGE
B) cost out of this world
may 10 empty your wallet. Renovating an existing C) seem outrageous
D) be costly
building to incorporate a green roof is generally
much more expensive than designing a

280 280
SAT Practice Writing Passage #1

11
new building to include a green roof.
Which choice offers an accurate representation of
Even considering all of these expenses, however,
the data in the chart?
most proponents of green roof construction are
A) NO CHANGE
convinced that the savings on total costs over the B) that the total costs over the lifespan of a green
roof approximately equals those of a
lifespan of a green roof more than make up for the traditional roof
C) that even though the total costs of a green roof
initial investment. 11 A properly installed green are greater than those of a traditional roof over
the green roof’s lifetime, it is still worth the
roof is likely to last 10 to 20 years longer than a investment
D) it is too soon to calculate the costs of owning a
traditional slate or shingled roof, primarily because green versus owning a traditional roof.

the plant barrier blocks harmful UV rays from


penetrating the roof’s internal structures.
Contractors and property owners who invest in
green buildings may also be eligible for tax credits,
bringing the overall net cost of the green roofs down
even further.

Total Cost of Owning a Traditional Roof


and a Green Roof

80,000
70,000
Cost (dollars)

60,000
50,000 Traditional
40,000 Roof
30,000
20,000 Green Roof

10,000
0
Year 2 Year 4 Year 6 Year 8 Year 10

Year of ownership

281 281
SAT Practice Writing Passage #2

Questions 12-22 are based on the following passage.


12

Honey Bee Movements A) NO CHANGE


B) reflect for
–1– C) reflects in
D) reflect in
When a honeybee returns to the hive after a
successful foraging trip, she often performs a

“dance,” a set of movements that 12 reflect in


miniature the details of her trip. The duration and
orientation of the movements in the dance depend
13
on the distance from, and the direction of, the bee’s
A) NO CHANGE
latest foraging site relative to the hive. B) movements: movements
C) movements. Movements
–2– D) movements; movements

An explanation is suggested by consideration of


the behavior of other species that perform so-called
idiothetic 13 movements, movements that reflect

in miniature previous large-scale movements. For


14
example, the io moth performs movements after
The writer is considering deleting the underlined
flight that contain information about the flight’s
phrase. Should the phrase be kept or deleted?
length. 14 Some social stingless bees also “dance”
A) Kept, because it supports the claim that other
after foraging. These dances, like those of the species perform idiothetic movements.
B) Kept, because it provides an example of a
honeybee, embody information about the bee’s most specific type of dance.
C) Deleted, because it contradicts the claims
recent flight, but this information is not used by the made in the rest of the paragraph.
D) Deleted, because it mentions stingless bees,
other stingless bees. However, the dances do blurring the focus of the paragraph.

282 282
SAT Practice Writing Passage #2

excite recruits to go out and search. 15 Likewise, 15


africanized honeybees are known to perform dances
A) NO CHANGE
that contain information about the location of food B) In other words,
C) However,
sources, but the dances do not recruit other bees, D) Yet,

because foraging in this subspecies is an individual


activity. 16 Evidently therefore, bees and other 16

insects may perform information-laden A) NO CHANGE


B) Evidently therefore
movements that are not communicative. C) Evidently, therefore
D) Evidently, therefore,
–3–

Thus, to explain how the function of the


honeybee’s dance has evolved, I postulate, first, that
idiothetic movements evolved in primitive bees, and

served to reinforce the memory of locations recently


visited. Then, as bees evolved group foraging, they
came to rely on odors picked up from successful

foragers as a guide to the location of resources.


Finally, idiothetic behavior took on the new function
of attracting the attention of foragers, making them

aware of the odors picked up by successful


hivemates. Thus, although the dance of the
17
honeybee contains locational information about food
A) NO CHANGE
sources, it is not necessary to assume that bee B) convey
C) emanate
dances 17 carry this information to other bees.
D) transpose

283 283
SAT Practice Writing Passage #2

However, the dance may 18 communicate and 18


share the success of a forager’s most recent flight A) NO CHANGE
B) communicate to share
and draw attention to the odors associated with the C) communicate in order to share
D) communicate
food source.
–4–
19
Professor Karl von Frisch won a Nobel Prize for
A) NO CHANGE
his studies on this behavior, and in particular for B) honeybee’s dance
C) honeybees’s dance
work leading him to conclude that the D) honeybees dances

19 honeybees dance communicates information

about the location of a food source to hivemates, 20

thereby enabling them to find and exploit the same A) NO CHANGE


B) However,
resource. 20 Moreover, there has been persistent C) In addition,
D) Similarly,
debate about this interpretation and today it is
generally believed that honeybees are guided to
21
suitable foraging sites by odor cues, including those
Which choice most logically follows from the
picked up from successful foragers, rather than by preceding information?
locational information contained in the dances of A) NO CHANGE
B) the function of the dance is well on its way to
other foragers. If this is correct, 21 the function of being understood.
C) the previous research conducted on the dance
the dance, which embodies information without is completely without merit.
D) odor cues are an effective way for bees to locate
communicating it to other bees, remains unclear. a food source.

Question 22 asks about the previous passage


as a whole.

284 284
SAT Practice Writing Passage #2

Think about the previous passage as a whole as


you answer question 22.

22

To make the passage most logical, paragraph 4


should be placed

A) where it is now.
B) before paragraph 1.
C) before paragraph 2.
D) before paragraph 3.

285 285
SAT Practice Writing Passage #3

Questions 23-33 are based on the following passage. 23

A) NO CHANGE
Exploration Skeptics
B) decades, NASA programs
In recent 23 decades NASA programs have C) decades; NASA programs
D) decades
been the subject of much ridicule. Critics argue that
since the Apollo missions, little substantial progress

has been made in space exploration. NASA missions


have been criticized as ineffective and costly, and 24

more than one respected pundit 24 had suggested A) NO CHANGE


B) had been suggesting
that the U.S. government drastically reduce funding C) was suggesting
D) has suggested
for the International Space Station and other NASA
ventures. Against this backdrop of hostility,
policymakers would do well to remember that
trailblazing scientific exploration has been the
25
subject of misplaced scrutiny throughout history.

Centuries before the founding of NASA, Christopher A) NO CHANGE


B) squarely
Columbus faced similar skepticism in funding his C) roundly
D) usually
maiden – and in retrospect, historic – voyage across

the Atlantic.
In 1485, Columbus presented his exploration
plans in Portugal, Genoa, and Venice, and was

25 typically refused. Columbus next sought an

286 286
SAT Practice Writing Passage #3

audience with the monarchs Ferdinand II of Aragon 26


and Isabella I of Castile. Queen Isabella referred the A) NO CHANGE
B) so
ambitious explorer’s proposal to a committee, C) and
D) given that
26 yet after much time had passed, these savants
of Spain, like their international counterparts, 27
reported that Columbus's plan was 27 batty. A) NO CHANGE
B) demented
In his 1828 biography of Columbus, Washington
C) cuckoo
Irving postulated that Columbus's initial failures to D) asinine

procure funding were due to European 28


28 monarch’s assumption that the world was flat. A) NO CHANGE
B) monarchs’s
29 Furthermore, the maritime navigation of the C) monarchs’
D) monarchs
time relied on the curvature of the earth, and

modern scholars recognize that Columbus's difficulty


29
in obtaining funds stemmed from a different source.
A) NO CHANGE
30 Would-be backers in initial meetings agreed B) In other words,
C) In addition,
that Columbus had an infeasible plan. It was the D) However,
vast underestimation of the size of the westward
30
expanse between Europe and Asia

After continually lobbying at the Spanish court Which choice most effectively combines the
underlined sentences?
and two years of negotiations, Columbus finally
A) The result was an infeasible plan, the vast
managed to secure funding for his voyage in 1492, underestimation of the westward expanse
between Europe and Asia, to would-be backers.
when Ferdinand intervened on the explorer’s behalf. B) In initial meetings, would-be backers agreed
that Columbus's plan was infeasible because he
Ferdinand later claimed credit for being “the had vastly underestimated the size of the
westward expanse between Europe and Asia.
C) The vast underestimation of the size of the
westward expanse between Europe and Asia
was the infeasible plan to would-be backers.
D) In initial meetings, would-be backers agreed
that Columbus's plan was infeasible; it was the
vast underestimation of the size of the
westward expanse between Europe and Asia.

287 287
SAT Practice Writing Passage #3

principal cause why those islands were discovered.” 31


Columbus himself believed that the Spanish Which choice provides the most logical introduction
to the paragraph?
crown’s motivations for backing him were religious.
A) NO CHANGE
In a letter regarding his prospective voyage, B) Ultimately, comparisons between Columbus
and NASA are only superficial.
Columbus described Isabella and Ferdinand as C) Columbus likely saw his western route as a
sort of final frontier.
“princes who love and promote the Christian faith.” D) DELETE the underlined portion.

He wrote that they had “determined to send him to


the countries of India to see the princes, people, and
32
territories, and to learn their disposition and the

proper method of converting them to the holy faith.” A) NO CHANGE


B) place – assuming
31 Columbus’ voyages were ultimately C) place; assuming
D) place assuming
disappointing. If his detractors had prevailed, the

western world would have been a very different


32 place: assuming that we had eventually
33
discovered America at all. Throughout the centuries,
Which choice concludes the passage with a
whenever someone has proposed a revolutionary restatement of the author’s primary claim?
exploration, be it nautical or astronomical in scope, A) NO CHANGE
B) Because of this, societies should be cautious in
there has always been a crowd ready to dismiss the considering scientific proposals.
C) Yet, people often disguise their true feelings
project as dangerous, foolish, or costly. 33 These about scientific ventures.
D) However, it is only through these ventures
critics have likely prevented numerous scientific that progress is made.
advancements throughout history.

288 288
SAT Practice Writing Passage #4

Questions 34-44 are based on the following passage.


34

Ernest Hemingway: Man and Myth A) NO CHANGE


B) then of
Ernest Hemingway is one of those rare authors C) than as
D) than of
most people know about, whether they have read
him or not, because of his sensational publicity and

personal invective. With his image of ruggedness, 35

confidence, virility, and bravery, he has the A) NO CHANGE


B) best: traveling
distinction of being one of the most famous C) best; traveling
D) best. Traveling
twentieth-century American writers. He has been
regarded less as a writer dedicated to his craft
36
34 then as a man of action who happened to be
Which choice most effectively sets up the examples
afflicted with genius. When he won the Nobel Prize that follow?

in 1954, Time magazine reported the news under A) NO CHANGE


B) He was one of the most innovative writers of
Heroes rather than Books. He wrote about what he
his era,
knew 35 best, traveling, bullfights, libations, C) He caused a lot of controversy,
D) He was willing to put himself into harm’s way,
women, wars, big game hunting, deep-sea fishing,
and courage. 36 He acquired his expertise through
well-reported acts of participation as well as 37

observation, by going to all of the wars of his time, The writer is considering deleting the underlined
portion. Should the writer make this change?
hunting and fishing for great beasts, marrying four
A) Yes, because it provides information that is not
times, occasionally getting into fistfights, and factually accurate.
B) Yes, because it introduces information that is
drinking to excess. 37 Hemingway’s writing has irrelevant at this point in the passage.
C) No, because it provides a logical conclusion to
the paragraph.
D) No, because it provides a specific example in
support of arguments made elsewhere in the
passage.

289 289
SAT Practice Writing Passage #4

been translated into many languages and has been 38


read by millions of people.
Which choice provides the most logical introduction
38 Hemingway’s youth greatly shaped his artistic to the paragraph?

voice. As a young man living in Chicago and bored A) NO CHANGE


B) Hemingway hoped to inspire a generation of
by vainglorious drawing room talk about arts and writers that would adopt his interests.
C) To a considerable degree, Hemingway was
artists, he rejected out of hand the role of the indoor complicit in the formation of his public persona.
D) DELETE the underlined portion.
aesthete. If he were to become a writer, it was going
to be at the opposite pole from Proust and his cork-
lined room! Hemingway had grown up in close
39
contact with the outdoors, and throughout his life he
A) NO CHANGE
pursued the hunting and fishing sports that he had B) inside
C) between
learned from his father. In doing so, Hemingway D) behind

assuredly took some amusement in confounding


public expectations of how a writer should dress and
40
conduct himself. After his father’s suicide, he took on
A) NO CHANGE
the persona of “Papa Hemingway.” It served as a B) But
C) In other words
defense, protecting the more complicated person D) So

39 in the mask. 40 And once the persona took

hold, it did not let go, and as a consequence


41
Hemingway dwindled into a celebrity – a person who
is famous for being famous – 41 who’s A) NO CHANGE
B) who
C) that
D) whose

290 290
SAT Practice Writing Passage #4

personality has been narrowed down to a few 42


instantly recognizable trademarks 42 and A) NO CHANGE
B) with characteristics
characteristics. The process had the unpropitious C) and motifs
effect of confusing Hemingway’s work with his life, D) DELETE the underlined portion.

or rather those components of his life 43 that was

lived in open 44 view; it subordinated his literary


43
accomplishment to his personal renown.
A) NO CHANGE
B) that were
C) those were
D) DELETE the underlined portion

44

A) NO CHANGE
B) view; they
C) view, it
D) view, they

291 291
292 292
SAT Appendix: Answers

Appendix:
Answers

293 293
SAT Reading Answers

Determining Central Command of Evidence Analyzing Multiple Practice Reading


Ideas, Themes, and (Page 73) Texts Passages
Meaning (Page 177) (Page 261)
(Page 27) Command of Evidence
Analyzing Multiple Texts Practice Reading
1) B 1) D Passage Answers
2) B 2) D 1) D
1) D 41) D
3) C 3) D 2) D
2) D 42) D
4) D 4) D 3) A
3) A 43) C
5) C 4) B
Command of Evidence 4) A 44) A
6) A
Review Analyzing Multiple Texts 5) C 45) D
7) C
Review 6) D 46) B
8) A
1) C 7) C 47) D
9) A
2) B 1) C 8) C 48) D
10) A
3) D 2) A 9) C 49) A
11) C
4) C 3) D 10) D 50) D
12) C
5) B 4) B 11) B 51) A
13) B
6) C 5) C 12) B 52) B
14) A
7) C 6) D 13) A
15) D
8) D 7) D 14) D
16) D
9) D 8) A 15) B
17) C
10) C 9) D 16) C
18) B
10) A 17) D
19) D
11) B 18) C
20) B
19) D
20) D
21) D
22) A
23) A
24) C
25) D
26) D
27) C
28) C
29) A
30) D
31) C
32) C
33) C
34) B
35) D
36) D
37) A
38) B
39) A
40) B

294 294
SAT Reading Answers

Words in Context Evaluating


(Page 129) Quantitative
Information
Words in Context (Page 151)

1) C Evaluating Quantitative
Information
2) A
3) C 1) D
4) A 2) B
3) A
Words in Context
4) C
Review

Evaluating Quantitative
1) D
Information Review
2) D
3) C 1) C
4) C 2) D
5) A 3) A
4) D

295 295
SAT Writing Answers

Punctuation Passage Analysis Wordiness and Transitions


(Page 39) (Page 53) Redundancy (Page 113)
(Page 85)
Punctuation: Comma Passage Analysis: Transitions
Usage Adding Precision Wordiness and (Conjunctions)
Redundancy:
1) A 1) A Wordiness 1) D
2) C 2) C 2) C
3) C 1) C 3) C
Passage Analysis: 2) C
Punctuation: Colon Introductions and
3) A Transitions (Main Ideas)
Usage Conclusions

4) A Wordiness and 4) D
3) D Redundancy:
5) C 5) C
4) B Redundancy
6) D 6) A
Passage Analysis: Main 4) B
Punctuation: Transitions Review
Idea Analysis
Contractions 5) D
6) D
7) A 5) C 1) B
8) C 6) A 2) B
Wordiness and
9) C 3) B
Redundancy Review
Passage Analysis:
Sequence 4) D
Punctuation: Dash
Usage 1) A
7) B 2) C
10) B 8) B 3) D
11) D
4) A
12) A
Passage Analysis Review 5) B
Punctuation:
1) D
Possession
2) C
13) B 3) C
14) A 4) D
15) C
5) A

Punctuation: Run-ons
and Sentence
Fragments

16) B
17) B
18) B

Punctuation Review

1) C
2) B
3) C
4) D
5) A
6) A
7) B

296 296
SAT Writing Answers

Parallelism Pronouns (Page 157) Verbs (Page 165) Prepositions


(Page 135) (Page 185)
Pronouns: Singular vs. Verbs: Subject-Verb
Parallelism Plural Agreement Prepositions

1) C 1) D 1) C 1) D
2) A 2) A 2) D 2) D
3) B 3) C 3) A 3) C

Parallelism: Pronouns: Personal Verbs: Tense Prepositions Review


Correlative Pronouns
Conjunctions 4) C
4) C 1) A
5) D
4) D 5) A 2) C
6) D
5) A 6) C 3) C
4) C
Verbs Review
Parallelism Review Pronouns: Possessive 5) B
Pronouns
1) B
1) A
7) B 2) C
2) C
8) A 3) B
3) D
9) C 4) B
4) D
5) B
5) D Pronouns: Pronoun Case

10) D
11) D
12) B

Unclear Pronoun
Subject

13) B
14) C
15) B

Pronouns Review

1) D
2) C
3) C
4) D
5) C

297 297
SAT Writing Answers

Adjectives and Word Choice Advanced Grammar Practice Writing


Adverbs (Page 189) (Page 217) (Page 237) Passages
(Page 277)
Adjectives and Adverbs Word Choice: Frequently Advanced Grammar:
Confused Words Affect vs. Effect Practice Writing
1) C Passage Answers
2) D 1) C 1) C
2) C 2) B 1) D 41) D
3) B
3) A 3) B 2) A 42) D
Adjectives and Adverbs 3) C 43) B
Review Word Choice: Advanced Grammar: 4) D 44) A
Inconsistent Tone and/or Parallel Comparisons 5) B
Style
1) D 6) B
4) C
2) A 4) D 7) D
5) C
3) D 5) D 8) B
6) D
4) C 6) A 9) C
5) C Advanced Grammar: 10) D
Word Choice Review Misplaced Modifiers 11) A
12) C
7) D
1) D 13) B
8) B
2) C 14) A
9) B
3) D 15) A
4) A Advanced Grammar: 16) D
5) B Noun Agreement 17) B
18) D
10) C
19) B
11) B
20) B
12) D
21) A
Advanced Grammar: 22) C
Who, Whom, That, 23) B
Which, and Whose 24) D
25) C
13) C
26) A
14) A
27) D
15) D
28) C
Advanced Grammar 29) D
Review 30) B
31) C
1) C 32) B
2) B 33) D
3) D 34) C
4) A 35) B
5) C 36) A
37) B
38) C
39) D
40) B

298 298
SAT Math Answers

Foundations of Plug and Chug & Translating Word Solving Non-Linear


Arithmetic and Backsolving Problems (Page 65) Equations (Page 79)
Algebra (Page 47) (Page 61)
Linear Equations Single Equation
Terms and Operations Plug and Chug Questions

1) A
1) D 1) B 1) D
2) 5
2) 0 2) A 2) 544
3) C
3) D 3) 3

Exponents 4) 96
Non-Linear Equations
Backsolving 5) A

3) 21 4) 3
4) D Systems of Equations
4) C 5) D
1 5) B 6) C
5) or 0.333 6) 7.15
3 6) D
7) 1
Absolute Value System of Equations
Plug and Chug & 8) C
Backsolving Review 9) C
6) D 7) A
7) D 1) A 8) 10
Rewriting Equations
8) D 2) C 9) B

3) C 10) B
Percentage Translating Word
4) B 11) A
Problems Review
12) D
9) C
1) B
10) B Solving Non-Linear
2) A
11) 60 Equations Review
3) D
4) 36
Foundations of 1) C
Arithmetic and Algebra 5) 47.06
2) A
Review 6) 29.6
3) 5
7) 3.8
4) 6
1) 12 8) 9804
5) 2
2) C
6) 93
3) C
7) 111
4) B
5) 8.2

299 299
SAT Math Answers

Solving Linear Ratio, Proportion, and Average, Median, and Functions (Page 145)
Equations (Page 117) Unit Conversion Mode (Page 139)
(Page 123) Functions: f(x)
Slope-Intercept Form Average (Arithmetic
Ratios and Proportional Mean) 1) D
Relationships 2) B
1) A
1) 54.5 3) 14
2) B
1) D 2) 200 4) 8
3) A
2) 160 3) 4
3) D Functions: f(x ± a)
Slope-Intercept Form
Application Median and Mode
Ratios and Proportional 5) C
Relationships (Colon) 6) 603
4) C 4) A
7) 14
5) A 5) B
4) 90
5) 32 Functions: Graphs
Slope Formula Average, Median, and
Mode Review 8) 1
Probability
6) 2 9) C
1) A 10) D
7) 4
11
6) or 0.55 2) B
8) B 20
3) C Functions: Tables
7) C
Solving Linear 4) 79.5
11) D
Equations Review 5) C
Unit Conversion 12) C

1) A Functions Review
8) C
2) B 9) D
1) 170
10) 24
3) D 2) D
8 3) B
4) or 2.667 Ratio, Proportion, and
3
Unit Conversion Review 4) B
5
5) or 0.625 5) D
8
1) D
2) 288
3) 17
4) D
5) D

300 300
SAT Math Answers

Quadratics and Circles (Page 193) Triangles (Page 199) Quadrilaterals and
Polynomials Polygons (Page 207)
(Page 169) Circles: Area and Triangles: Angles (Rule
Circumference of 180 Rhombus Questions
Quadratics
1) 40 1) D 1) 220

1) 39 2) B 2) D 2) 5

2) A 3) 37.5 3) C
Triangle: Area 4) 64
3) C
Circles: Arc and Sector 3) D Trapezoid Questions
Polynomial Division 4) B
4) A 5) 225
4) D 5) 144 Triangles: Sides Lengths 6) 370
(Pythagorean Theorem)
5) C 6) x = 39, 40, 41, or 42 7) C
5) B
Circles: Chords Polygon Questions
Polynomial Factors 6) D
8) 270
6) C 7) C Triangles: Special Right
Triangles 9) 3
7) A 8) D
10) C
8) D 7) 27
Circles Review Quadrilaterals and
8) B
Quadratic Formula Polygons Review
1) D Triangles: Similar
2) 10 Triangles 1) 10
9) B
3) C 2) 4
10) A 9) B
4) B 3) 66.7
10) B 4) 10
Completing the Square
5) D
Triangles Review 6) B
11) 5
12) B 1) 288
2) 300
Higher-Order 3) 17
Equations 4) C

13) C
14) C

Quadratics and
Polynomials Review

1) 32
3
2) or 1.5
2
36
3) or 7.2
5
4) C

5) A

6) A

301 301
SAT Math Answers

Lines and Angles Statistics (Page 221) Graphing (Page 229) Interpreting
(Page 213) Equations (Page 233)
Statistics: Exponential Graphing – Circles
Intersecting Lines Growth and Exponential Interpreting Linear
Decay Equations
1) 22
1) 405
2) C
2) B 1) 0.65 1) A
3) A
2) D 2) D
Rule of 180 3) C Graphing – Parabolas
Interpreting Non-Linear
3) 115 Statistics: Interpreting Equations
4) C
4) B Charts and Graphs
5) B 3) C
4) C 4) B
Lines and Angles
5) C Graphing Review
Review
Interpreting Equations
Statistics: Line of Best 1) 180 Review
1) C
Fit 2) D 1) D
2) C
3) A 2) D
3) B 6) B
4) C 3) C
7) A
4) B
8) C

Statistics: Surveys,
Research, and Margin of
Error

9) C
10) A

Statistics: Standard
Deviation

11) B
12) A

Statistics Review

1) C
2) B
3) B
4) A
5) A
6) D
7) A

302 302
SAT Math Answers

Geometry in Three Advanced Concepts


Dimensions (Page 253)
(Page 245)
Complex Numbers
Cones
1) A
1) B
2) 50 2) B
1
3) or 0.2
Cylinders 5

3) 25 Compound Interest
4) 8
4) 1.03
Rectangular Prisms 5) A
6) D
5) D
6) C Trigonometry: SOH-
CAH-TOA
Spheres
7) C
7) 12
8) 5.6
8) C
12
9) or 2.4
Rectangular Pyramids 5

9) 27 Unit Circle
10) A 7
10) or 0.875
8
Hexagonal Prisms 11) B

11) 140.3
Advanced Concepts
12) 60 Review

Geometry in Three
1) 48
Dimensions Review
2) 3
3) 20
1) 1800
4) C
2) 64
5) C
3) A
6) A
4) B
5) A

303 303
304 304
SAT Appendix: Common Math Formulas

Appendix:
Common Math Formulas

305 305
SAT Appendix: Common Math Formulas
Average Graphing

Sum Circle
Average =
Total Number
(x – h)2 + (y – k)2 = r2

Circles (Non-Graphing) Center: (h, k)


Radius: r
Area Circumference
Parabola
Area = r2 Circumference = 2r
Graphing Formula: y = ax2 + bx + c
Arc Vertex form: y = a(x – h)2 + k, where (h, k) is the vertex
Central Angle
Arc = Circumference( )
360
Quadratics and Polynomials
Sector

Central Angle Quadratic Formula


Sector = Area( )
360
2
–b ± b – 4ac
Compound Interest x=
2a
r
Compound Interest: A = P(1 + )nt
n
Quadrilaterals and Polygons
A = Amount accumulated
P = Principal Rhombus Area Rhombus Perimeter
r = Interest rate expressed as a decimal
n = Compounding per period Area = bh Perimeter = 2b + 2h
t = Number of periods (In a square, b and h are equal) (In a square, b and h are equal)

Trapezoid Area
Exponential Growth Exponential Decay
b1 + b2
Area = ( )h
y = C(1 + r)t y = C(1 – r)t 2

y = Final Amount y = Final Amount Polygon Interior Angles


C = Initial amount C = Initial amount
r = Rate of Change r = Rate of Change Sum of Interior Angles = 180° (n – 2)
t = Time t = Time n = number of sides

Ratio, Probability, and Unit Conversion


Geometry in Three Dimensions
Ratio Formula Direct Proportional Relationship
Cone Volume
1 2 One Part One Part One Part
Volume = 𝜋r h Ratio = =
3 Another Part Whole Thing Whole Thing

Probability
Cylinder Volume Cylinder Surface Area
Desired Outcomes
Probability =
Volume = 𝜋r2h Surface Area = 2𝜋r2 + 2𝜋rh Total Outcomes

Triangles
Rectangular Prism Volume Rectangular Prism Surface Area
Volume = lwh Area Pythagorean Theorem
Surface Area = 2wl + 2hl + 2hw
1
Area = bh a2 + b2 = c2
2
Rectangular Pyramid Volume Sphere Volume
1 4 3 Trigonometry
Volume = lwh Volume = 𝜋r
3 3
opposite adjacent opposite
sin Θ = cos Θ = tan Θ =
hypotenuse hypotenuse adjacent

306 306
SAT Appendix: Table of Contents

Appendix:
Table of Contents

307 307
SAT Appendix: Table of Contents

Introduction to the SAT

Introduction to the SAT: Test Overview (Page 7)

Introduction to the SAT: Content Overview (Page 13)

The Test Zone (Page 23)

SAT Reading

Analyzing Multiple Texts (Page 177)

Command of Evidence (Page 73)

Determining Central Ideas, Themes, and Meaning (Page 27)

SAT Reading and Writing

Evaluating Quantitative Information (Page 151)

Words in Context (Page 129)

SAT Writing

Adjectives and Adverbs (Page 189)

Advanced Grammar (Page 237)

Parallelism (Page 135)


Passage Analysis (Page 53)

Prepositions (Page 185)

Pronouns (Page 157)

Punctuation (Page 39)

Transitions (Page 113)

Verbs (Page 165)

Word Choice (Page 217)

Wordiness and Redundancy (Page 85)

308 308
SAT Appendix: Table of Contents

SAT Math

Advanced Concepts (Page 253)


Average, Median, and Mode (Page 139)
Circles (Page 193)
Foundations of Arithmetic and Algebra (Page 47)
Functions (Page 145)
Geometry in Three Dimensions (Page 245)
Graphing (Page 229)
Interpreting Equations (Page 233)
Lines and Angles (Page 213)
Plug and Chug & Backsolving (Page 61)
Quadratics and Polynomials (Page 169)
Quadrilaterals and Polygons (Page 207)
Ratio, Proportion, and Unit Conversion (Page 123)
Solving Linear Equations (Page 117)
Solving Non-Linear Equations (Page 79)
Statistics (Page 221)
Translating Word Problems (Page 65)
Triangles (Page 199)

SAT Essay
SAT Essay (Page 89)

SAT Practice Reading and Writing Passages

Practice Reading Passages (Page 261)


Practice Writing Passages (Page 277)

Appendix

Appendix: Answers (Page 293)


Appendix: Common Math Formulas (Page 305)

309 309
310 310
311 311
312 312

You might also like